Sie sind auf Seite 1von 513

Cambridge IGCSE®

BIOLOGY 0610/02
*0123456789*

Paper 2 Multiple Choice (Extended) For examination from 2020


SPECIMEN PAPER
45 minutes
Additional materials: Multiple choice answer sheet
Soft clean eraser
Soft pencil (type B or HB is recommended)

READ THESE INSTRUCTIONS FIRST

Write in soft pencil.


Do not use staples, paper clips, glue or correction fluid.
Write your name, centre number and candidate number on the answer sheet in the spaces provided unless
this has been done for you.
DO NOT WRITE IN ANY BARCODES.

There are forty questions on this paper. Answer all questions. For each question there are four possible
answers A, B, C and D.
Choose the one you consider correct and record your choice in soft pencil on the separate answer sheet.

Read the instructions on the answer sheet very carefully.

Each correct answer will score one mark. A mark will not be deducted for a wrong answer.
Any rough working should be done in this booklet.
Electronic calculators may be used.

This document consists of 18 printed pages.

© UCLES 2017 1
[Turn over
2

1 The diagram shows some liver cells as they appear under a microscope.

How many cell walls can be seen?

A 0 B 2 C 5 D 10

2 The diagram shows two pathogens, not drawn to the same scale.

pathogen L pathogen M

S
R cytoplasm

What identifies pathogens L and M and structures R and S?

L M R S

A prokaryote virus cell wall protein coat


B prokaryote virus protein coat cell wall
C virus prokaryote cell wall protein coat
D virus prokaryote protein coat cell wall

2
© UCLES 2017 0610/02/SP/20
3

3 The diagram shows a high-power drawing of a plant cell.

The distance between X and Y on the diagram below is 80 mm.

The actual length of the cell between X and Y was 160 µm.

Y
X

What is the magnification of the cell?

A ×50 B ×100 C ×500 D ×1000

4 The diagram shows part of a plant root in the soil. The root is absorbing water.

At which labelled point is the water potential highest?

soil
particle
D
A B C

xylem
root hair cell

3
© UCLES 2017 0610/02/SP/20 [Turn over
4

5 The diagram shows two solutions that are separated by a partially permeable membrane.

partially permeable
membrane

dilute concentrated
solution solution
of of
sucrose sucrose

X Y

In which direction will most water molecules move in relation to their concentration gradient?

A from X to Y against their concentration gradient


B from X to Y down their concentration gradient
C from Y to X against their concentration gradient
D from Y to X down their concentration gradient

6 Which statement describes active transport?

A the movement of ions through proteins in a cell membrane, releasing energy


B the movement of ions through proteins in a cell membrane, using energy
C the movement of water through lipids in a cell membrane, releasing energy
D the movement of water through lipids in a cell membrane, using energy

7 The diagram shows an experiment using an uncooked potato. The skin of the potato was
removed as shown.

concentrated
sugar solution

potato
skin has been
water removed

Which diagram shows the result of the experiment after 24 hours?

A B C D

4
© UCLES 2017 0610/02/SP/20
5

8 One strand of a DNA molecule contains the base sequence AGT.

What is the base sequence on the other strand of the DNA molecule?

A AGT
B GAT
C TAC
D TCA

9 The following can be used to write a word equation for photosynthesis.

1 carbon dioxide and water

2 light and chlorophyll

3 glucose and oxygen

Which shows a correct word equation for photosynthesis?

A 1 → 2 in the presence of 3

B 1 → 3 in the presence of 2

C 2 → 3 in the presence of 1

D 3 → 1 in the presence of 2

10 Which row shows the most likely number of chloroplasts in three types of cell in a leaf?

vascular
epidermis mesophyll
bundle cell

A 0 6 17
B 0 17 0
C 17 6 0
D 17 0 6

5
© UCLES 2017 0610/02/SP/20 [Turn over
6

11 The diagram shows a tooth.

bone

What are the parts labelled 1, 2 and 3?

1 2 3

A dentine enamel pulp


B enamel dentine pulp
C enamel pulp dentine
D pulp dentine enamel

12 What is the function of the lacteal in a villus?

A absorbs the fatty acids


B allows substances to pass from the lumen of the intestine into the villus
C takes oxygen to the villus
D absorbs the glucose and amino acids

13 On a dry, sunny day, how does water vapour move through the stomata of a leaf?

A into the leaf by diffusion


B into the leaf by respiration
C out of the leaf by diffusion
D out of the leaf by respiration

6
© UCLES 2017 0610/02/SP/20
7

14 The diagram shows a capillary and some body cells.

body cell

Q
P capillary wall

How do ions pass from P to Q?

A by diffusion in blood
B by diffusion in tissue fluid
C by osmosis in blood
D by osmosis in tissue fluid

15 These actions may be important in controlling the spread of disease.

1 washing hands after going to the toilet

2 disposing of waste frequently

3 using separate cutting boards for meat and salad

4 disposing of raw sewage into a river

Which would help control the spread of disease?

A 1, 2, 3 and 4
B 1, 2 and 3 only
C 2 and 3 only
D 4 only

16 New-born babies have passive immunity.

Why is this only temporary?

A No memory cells are produced in the baby.


B The antibodies are insufficient in number.
C The antibodies only act in the mother.
D The immunity is not inherited.

7
© UCLES 2017 0610/02/SP/20 [Turn over
8

17 The table shows some of the features of respiration.

Which row is correct for anaerobic respiration?

amount of energy
releases
released per chemical reaction
carbon dioxide
glucose molecule

A high always the same sometimes


B low different in different organisms sometimes
C high different in different organisms always
D low always the same always

18 The diagram shows some apparatus used to investigate respiration. Yeast, warm water and
substance Z were put into a test-tube. After a while, the limewater began to go cloudy.

yeast, warm water limewater


and substance Z

What is substance Z?

A alcohol
B glucose
C nitrogen
D oxygen

19 What happens during the process of breathing out?

external intercostal pressure in the lungs volume of thorax


diaphragm contracts
muscles contract increases increases

A yes yes no yes


B yes no yes yes
C no yes no no
D no no yes no

8
© UCLES 2017 0610/02/SP/20
9

20 The diagram shows a kidney tubule and some of its associated blood vessels.

Which substance is entirely reabsorbed from the fluid at R to the blood at P?

A glucose
B salts
C urea
D water

21 Which of the following can be an effector in a reflex arc?

A a gland
B a light receptor
C the brain
D the spinal cord

9
© UCLES 2017 0610/02/SP/20 [Turn over
10

22 The diagram shows a section through a human eye.

When focusing on a close object at night, what is the state of structures P and Q?

P Q

A contracted tight
B contracted slack
C relaxed tight
D relaxed slack

23 The diagram shows a shoot that has been placed on its side. The shoot begins to grow upwards.

up
P

What causes the shoot to grow upwards?

A increased cell division by meiosis at P


B increased cell division by mitosis at P
C more cell elongation at P than Q
D more cell elongation at Q than P

10
© UCLES 2017 0610/02/SP/20
11

24 Some roots are known to be gravitropic.

Which pair of diagrams show a controlled experiment to find out if these roots are also
phototropic?

light proof light proof light


box box

light

1 2 3 4

A 1 and 2
B 1 and 3
C 2 and 3
D 2 and 4

25 Which feature of sexual reproduction helps a species to evolve?

A Fewer offspring are produced than in asexual reproduction.


B Offspring are the result of the fusion of the nuclei of dissimilar gametes.
C Offspring always inherit advantageous characteristics.
D Offspring produced will always be in a suitable environment.

26 The diagram shows the female reproductive system.

What is the function of the part labelled X?

A gamete production and hormone secretion


B gamete production only
C hormone secretion only
D zygote production and hormone secretion

11
© UCLES 2017 0610/02/SP/20 [Turn over
12

27 The diagram shows the menstrual cycle of a woman during the month of September.

Sunday Monday Tuesday Wednesday Thursday Friday Saturday

1 2 3 4 5 6

7 8 9 10 11 12 13

14 15 16 17 18 19 20

21 22 23 24 25 26 27

28 29 30

key

= ovulation

= menstruation

Why can fertilisation not take place if sperms are released into the vagina on 8th September?

A Sperms are washed out of the female uterus by the menstrual flow.
B Sperms can survive in the female reproductive system for only 3 or 4 days.
C Sperms must be released after ovulation for fertilisation to take place.
D The uterus lining is washed out of the female body during menstruation.

28 Which states a function of the four hormones FSH, LH, progesterone and oestrogen in controlling
the menstrual cycle?

FSH LH progesterone oestrogen

A stimulates the maintains the lining causes an egg to causes the mature
pituitary gland to of the uterus during mature in an ovary egg to be released
release LH the middle part of from the ovary
the menstrual cycle

B causes an egg to causes the mature prepares the uterus stimulates the
mature in an ovary egg to be released to receive a fertilised pituitary gland to
from the ovary ovum release LH

C prepares the uterus stimulates the stops FSH being maintains the lining
to receive a fertilised ovaries to release produced of the uterus during
ovum oestrogen the middle part of
the menstrual cycle

D maintains the lining stops FSH being causes the mature prepares the uterus
of the uterus during produced egg to be released to receive a fertilised
the middle part of from the ovary ovum
the menstrual cycle

12
© UCLES 2017 0610/02/SP/20
13

29 The diagram shows a cell of an organism formed by reduction division. The nucleus contains
20 chromosomes.

What is the diploid number for this organism?

A 10 B 20 C 40 D 46

30 What are the possible blood groups of the offspring of parents who have blood group A and B?

A AB only
B A and B
C A, B and AB
D A, B, AB and O

13
© UCLES 2017 0610/02/SP/20 [Turn over
14

31 The diagram shows a family tree and the inheritance of the ability to taste a certain substance.

The allele for the ability to taste this substance is dominant.

1st generation David Mary Jeanne Mark

2nd generation

key
male ‘taster’ male ‘non-taster’

female ‘taster’ female ‘non-taster’

Which statement about the genotypes of the sisters Mary and Jeanne is correct?

A Mary is heterozygous and Jeanne is homozygous.


B Mary is homozygous and Jeanne is heterozygous.
C They are both heterozygous.
D They are both homozygous.

32 Haemophilia is a sex-linked recessive condition. A haemophiliac man has one son who has
inherited haemophilia and two more sons who have not.

The man’s wife is pregnant again. If this baby is a girl, what is the chance that she will have
haemophilia?

A 0% B 25 % C 50 % D 75 %

33 Antibiotics often do not work as effectively against bacterial infections as they used to.

What change has taken place in the bacteria, and what type of selection has brought this about?

change in the bacteria type of selection

A have become immune to the antibiotic artificial


B have become immune to the antibiotic natural
C have become resistant to the antibiotic artificial
D have become resistant to the antibiotic natural
14
© UCLES 2017 0610/02/SP/20
15

34 The diagram shows an Arctic ecosystem.

polar bear

glaucous gull ringed seal

little auk arctic cod

squid

amphipods shrimps

herbivorous
copepods

phytoplankton

sunlight

How many organisms can act as tertiary consumers in this food web?

A 3 B 4 C 5 D 6

15
© UCLES 2017 0610/02/SP/20 [Turn over
16

35 The diagram shows the circulation of nitrogen in nature.

nitrogen
in the air
3
2

lightning
ammonium organic
nitrate in nitrogen
1 fertiliser NO in plants
legumes

plants NH4+
5
2 in soil

nitrate animals
in soil
nitrite
in soil
4 4

What is correct?

denitrification nitrification nitrogen fixation

A 1 5 3
B 2 5 4
C 3 2 1
D 1 4 3

36 What is the correct match of example to ecological term?

community ecosystem population trophic level

A all lake organisms freshwater lake freshwater shrimps pond weed as


primary producer
B freshwater shrimps all lake organisms pond weed as freshwater lake
primary producer
C freshwater lake pond weed as freshwater shrimps all lake organisms
primary producer
D freshwater shrimps freshwater lake all lake organisms pond weed as
primary producer

16
© UCLES 2017 0610/02/SP/20
17

37 Some rabbits colonised an island for the first time. The graph shows how their population size
changed over the next few years.

population
size

time

What explains the way the size of the rabbit population changed during the exponential (log)
phase?

A birth rate and death rate in equilibrium


B increasing number of rabbits able to reproduce
C increase in the number of predators
D limiting factors begin to take effect

17
© UCLES 2017 0610/02/SP/20 [Turn over
18

38 The diagram shows a bacterium whose plasmid is being used during genetic engineering to
produce human insulin.

section of
DNA cut
and removed

plasmid

What is inserted at P so that the bacterium can produce human insulin, and which enzyme is
used to catalyse the insertion?

A a section of human DNA, using DNA ligase


B a section of human DNA, using restriction enzymes
C a section of human mRNA, using DNA ligase
D a section of human mRNA, using restriction enzymes

39 Modern technology has resulted in the production and use of insecticides.

Which statement is a false description of insecticides?

A Insecticides can affect food chains.


B Insecticides can cause pollution.
C Insecticides improve yields.
D Insecticides target weeds.

40 Which of these measures will help to prevent acid rain?

A avoiding the use of non-recyclable plastics


B filtering sulfur dioxide in power station chimneys
C reducing methane emissions in industry and agriculture
D using alkaline fertilisers on fields

Permission to reproduce items where third-party owned material protected by copyright is included has been sought and cleared where possible. Every
reasonable effort has been made by the publisher (UCLES) to trace copyright holders, but if any items requiring clearance have unwittingly been included, the
publisher will be pleased to make amends at the earliest possible opportunity.

Cambridge Assessment International Education is part of the Cambridge Assessment Group. Cambridge Assessment is the brand name of University of
Cambridge Local Examinations Syndicate (UCLES), which is itself a department of the University of Cambridge.
18
© UCLES 2017 0610/02/SP/20
Cambridge IGCSE®

BIOLOGY 0610/01
*0123456789*

Paper 1 Multiple Choice (Core) For examination from 2020


SPECIMEN PAPER
45 minutes
Additional materials: Multiple choice answer sheet
Soft clean eraser
Soft pencil (type B or HB is recommended)

READ THESE INSTRUCTIONS FIRST

Write in soft pencil.


Do not use staples, paper clips, glue or correction fluid.
Write your name, centre number and candidate number on the answer sheet in the spaces provided unless
this has been done for you.
DO NOT WRITE IN ANY BARCODES.

There are forty questions on this paper. Answer all questions. For each question there are four possible
answers A, B, C and D.
Choose the one you consider correct and record your choice in soft pencil on the separate answer sheet.

Read the instructions on the answer sheet very carefully.

Each correct answer will score one mark. A mark will not be deducted for a wrong answer.
Any rough working should be done in this booklet.
Electronic calculators may be used.

This document consists of 16 printed pages.

© UCLES 2017 19
[Turn over
2

1 The diagram shows a structure found in the human body.

What is this structure an example of?

A an organ
B an organism
C an organ system
D a tissue

2 Which system is used for naming species?

A binomial
B conservation
C dichotomous
D natural selection

20
© UCLES 2017 0610/01/SP/20
3

3 The diagram shows a section through a flower.

petal

Using the key, identify this flower.

1 sepals present ............................................. go to 2


sepals not present ....................................... go to 3
2 stamens attached to petals ......................... A
stamens not attached to petals ................... B
3 stigma above anthers .................................. C
stigma below anthers .................................. D

4 The table shows some features of animals.

Which animal could be a bird?

feature
feathers gills hair wings

A 8 9 9 8 key
B 9 9 8 8 9 = present
C 9 8 8 9 8 = not present
D 8 8 9 9

5 The diagram shows some liver cells as they appear under a microscope.

How many cell walls can be seen?

A 0 B 2 C 5 D 10

21
© UCLES 2017 0610/01/SP/20 [Turn over
4

6 Which row shows the most likely number of chloroplasts in three types of cell in a leaf?

vascular
epidermis mesophyll
bundle cell

A 0 6 17
B 0 17 0
C 17 6 0
D 17 0 6

7 The diagram shows an experiment using an uncooked potato. The skin of the potato was
removed as shown.

concentrated
sugar solution

potato
skin has been
water removed

Which diagram shows the result of the experiment after 24 hours?

A B C D

8 A student draws a diagram of a plant cell.

The diagram is 40 mm in width. The plant cell is 0.02 mm in width.

What is the magnification of the student’s drawing?

A ×0.005

B ×0.08

C ×200

D ×2000

9 On a dry, sunny day, how does water vapour move through the stomata of a leaf?

A into the leaf by diffusion


B into the leaf by respiration
C out of the leaf by diffusion
D out of the leaf by respiration
22
© UCLES 2017 0610/01/SP/20
5

10 The diagram shows two solutions that are separated by a partially permeable membrane.

partially permeable
membrane

dilute concentrated
solution solution
of of
sucrose sucrose

X Y

In which direction will most water molecules move in relation to their concentration gradient?

A from X to Y against their concentration gradient


B from X to Y down their concentration gradient
C from Y to X against their concentration gradient
D from Y to X down their concentration gradient

11 The graph shows how the rate of an enzyme-controlled reaction changes with temperature.

rate of
reaction

temperature X

What describes the shape of the graph within the temperature range marked X?

A The rate of reaction decreases.


B The rate of reaction increases then decreases.
C The rate of reaction reaches a maximum.
D The reaction is occurring at the optimum temperature.

23
© UCLES 2017 0610/01/SP/20 [Turn over
6

12 Six test-tubes were set up at different pH values. They each contained the same quantity of
starch and amylase in water. The table shows the time taken for the reaction to be completed in
each test-tube.

pH 1 3 5 7 9 11

time to be completed / s 95 64 42 5 35 66

At which pH does the amylase work best?

A pH 1 B pH 5 C pH 7 D pH 11

13 What are the chemical elements found in carbohydrates?

A carbon, hydrogen and oxygen only


B carbon, hydrogen, oxygen and nitrogen only
C carbon, hydrogen, oxygen and sulfur only
D carbon, hydrogen, oxygen, nitrogen and sulfur only

14 A protease is added to a cloudy suspension of protein in a test-tube and kept at 37 °C.

After eight minutes, the suspension changes from cloudy to transparent.

Which product, or products, will now be present in the test-tube?

A amino acids
B fatty acids
C glycerol
D simple sugars

15 The following can be used to write a word equation for photosynthesis.

1 carbon dioxide and water

2 light and chlorophyll

3 glucose and oxygen

Which shows a correct word equation for photosynthesis?

A 1 → 2 in the presence of 3

B 1 → 3 in the presence of 2

C 2 → 3 in the presence of 1

D 3 → 1 in the presence of 2

24
© UCLES 2017 0610/01/SP/20
7

16 The photograph shows a section through the leaf of a plant.

Which label shows the position of the cuticle?

B
D

17 What is a symptom of scurvy?

A bleeding gums
B breathlessness
C diarrhoea
D pain in joints

18 What is an appropriate treatment for severe diarrhoea?

A avoiding drinking
B avoiding high-fibre foods
C drinking a dilute solution of salt and glucose
D eating high-fat foods

25
© UCLES 2017 0610/01/SP/20 [Turn over
8

19 The diagram shows a tooth.

bone

What are the parts labelled 1, 2 and 3?

1 2 3

A dentine enamel pulp


B enamel dentine pulp
C enamel pulp dentine
D pulp dentine enamel

20 The diagram shows a human heart and some of its major blood vessels.

Blockage of which labelled part is a cause of coronary heart disease?

26
© UCLES 2017 0610/01/SP/20
9

21 These actions may be important in controlling the spread of disease.

1 washing hands after going to the toilet

2 disposing of waste frequently

3 using separate cutting boards for meat and salad

4 disposing of raw sewage into a river

Which would help control the spread of disease?

A 1, 2, 3 and 4
B 1, 2 and 3 only
C 2 and 3 only
D 4 only

22 The table shows some of the features of respiration.

Which row is correct for anaerobic respiration?

amount of energy
releases
released per chemical reaction
carbon dioxide
glucose molecule

A high always the same sometimes


B low different in different organisms sometimes
C high different in different organisms always
D low always the same always

27
© UCLES 2017 0610/01/SP/20 [Turn over
10

23 The diagram shows some apparatus used to investigate respiration. Yeast, warm water and
substance Z were put into a test-tube. After a while, the limewater began to go cloudy.

yeast, warm water limewater


and substance Z

What is substance Z?

A alcohol
B glucose
C nitrogen
D oxygen

24 The diagram shows someone blowing up a balloon.

How do the proportions of gases in the air inside the balloon compare with the air outside the
balloon?

carbon dioxide oxygen water vapour

A less more more


B less more less
C more less more
D more less less

28
© UCLES 2017 0610/01/SP/20
11

25 Which organ produces urea?

A bladder
B kidney
C liver
D pancreas

26 The diagram shows a neurone carrying an impulse.

direction of impulse

Which row describes the type of neurone and the direction of impulse?

type of neurone direction of impulse

A motor towards the spinal cord


B motor away from the spinal cord
C sensory towards the spinal cord
D sensory away from the spinal cord

27 Which of the following can be an effector in a reflex arc?

A a gland
B a light receptor
C the brain
D the spinal cord

29
© UCLES 2017 0610/01/SP/20 [Turn over
12

28 The diagram shows the female reproductive system.

What is the function of the part labelled X?

A gamete production and hormone secretion


B gamete production only
C hormone secretion only
D zygote production and hormone secretion

29 The diagram shows the menstrual cycle of a woman during the month of September.

Sunday Monday Tuesday Wednesday Thursday Friday Saturday

1 2 3 4 5 6

7 8 9 10 11 12 13

14 15 16 17 18 19 20

21 22 23 24 25 26 27

28 29 30

key

= ovulation

= menstruation

Why can fertilisation not take place if sperms are released into the vagina on 8th September?

A Sperms are washed out of the female uterus by the menstrual flow.
B Sperms can survive in the female reproductive system for only 3 or 4 days.
C Sperms must be released after ovulation for fertilisation to take place.
D The uterus lining is washed out of the female body during menstruation.

30
© UCLES 2017 0610/01/SP/20
13

30 Some roots are known to be gravitropic.

Which pair of diagrams show a controlled experiment to find out if these roots are also
phototropic?

light proof light proof light


box box

light

1 2 3 4

A 1 and 2
B 1 and 3
C 2 and 3
D 2 and 4

31 What term is used for the transfer of a gene from one organism to another?

A artificial selection
B genetic engineering
C mutation
D natural selection

32 In an experiment to investigate the effects of various environmental factors on germination, four


boiling tubes were set up as shown.

In which tube would the seeds germinate most quickly?

A B C D
cotton wool

cress seed
moist moist
moist dry cotton wool cotton wool
cotton wool cotton wool
substance to
water absorb O2 water

25 °C 25 °C 25 °C 5 °C

31
© UCLES 2017 0610/01/SP/20 [Turn over
14

33 The diagram shows some parts of the male reproductive system.

Which part is cut during a vasectomy?

34 The diagram shows a family tree and the inheritance of the ability to taste a certain substance.

The allele for the ability to taste this substance is dominant.

1st generation David Mary Jeanne Mark

2nd generation

key
male ‘taster’ male ‘non-taster’

female ‘taster’ female ‘non-taster’

Which statement about the genotypes of the sisters Mary and Jeanne is correct?

A Mary is heterozygous and Jeanne is homozygous.


B Mary is homozygous and Jeanne is heterozygous.
C They are both heterozygous.
D They are both homozygous.

32
© UCLES 2017 0610/01/SP/20
15

35 The diagram shows an eagle.

Eagles have hooked beaks that help them to ingest food.

What is this an example of?

A adaptation
B assimilation
C nutrition
D variation

36 In which order do organisms occur in the food chain?

A carnivore → herbivore → producer


B herbivore → carnivore → producer
C producer → carnivore → herbivore
D producer → herbivore → carnivore

37 How is energy transferred between organisms in a food chain?

A combustion
B ingestion
C photosynthesis
D respiration

33
© UCLES 2017 0610/01/SP/20 [Turn over
16

38 The diagram shows part of a section through a leaf in daylight.

What substance follows the path of the arrow out of the leaf?

A carbon dioxide
B energy
C oxygen
D water

39 Which factors, if present, may cause a population of animals to decrease?

predation disease

A 9 9
B 9 8
C 8 8
D 8 9

40 Modern technology has resulted in the production and use of insecticides.

Which statement is a false description of insecticides?

A Insecticides can affect food chains.


B Insecticides can cause pollution.
C Insecticides improve yields.
D Insecticides target weeds.

Permission to reproduce items where third-party owned material protected by copyright is included has been sought and cleared where possible. Every
reasonable effort has been made by the publisher (UCLES) to trace copyright holders, but if any items requiring clearance have unwittingly been included, the
publisher will be pleased to make amends at the earliest possible opportunity.

Cambridge Assessment International Education is part of the Cambridge Assessment Group. Cambridge Assessment is the brand name of University of
Cambridge Local Examinations Syndicate (UCLES), which is itself a department of the University of Cambridge.
34
© UCLES 2017 0610/01/SP/20
Cambridge International Examinations
Cambridge International General Certificate of Secondary Education
*0123456789*

BIOLOGY 0610/02
Paper 2 Multiple Choice (Extended) For Examination from 2016
SPECIMEN PAPER
45 minutes
Additional Materials: Multiple Choice Answer Sheet
Soft clean eraser
Soft pencil (type B or HB is recommended)

READ THESE INSTRUCTIONS FIRST

Write in soft pencil.


Do not use staples, paper clips, glue or correction fluid.
Write your name, Centre number and candidate number on the Answer Sheet in the spaces provided unless
this has been done for you.
DO NOT WRITE IN ANY BARCODES.

There are forty questions on this paper. Answer all questions. For each question there are four possible
answers A, B, C and D.
Choose the one you consider correct and record your choice in soft pencil on the separate Answer Sheet.

Read the instructions on the Answer Sheet very carefully.

Each correct answer will score one mark. A mark will not be deducted for a wrong answer.
Any rough working should be done in this booklet.
Electronic calculators may be used.

The syllabus is accredited for use in England, Wales and Northern Ireland as a Cambridge International Level 1/Level 2 Certificate.

This document consists of 18 printed pages.

© UCLES 2014 35
[Turn over
2

1 The diagram shows some liver cells as they appear under a microscope.

How many cell walls can be seen?

A 0 B 2 C 5 D 10

2 The diagram shows two pathogens, not drawn to the same scale.

pathogen L pathogen M

S
R cytoplasm

What identifies pathogens L and M and structures R and S?

L M R S

A prokaryote virus cell wall protein coat


B prokaryote virus protein coat cell wall
C virus prokaryote cell wall protein coat
D virus prokaryote protein coat cell wall

36
© UCLES 2014 0610/02/SP/16
3

3 The diagram shows a high-power drawing of a plant cell.

The distance between X and Y on the diagram below is 80 mm.

The actual length of the cell between X and Y was 160 µm.

Y
X

What is the magnification of the cell?

A ×50 B ×100 C ×500 D ×1000

4 The diagram shows part of a plant root in the soil. The root is absorbing water.

At which labelled point is the water potential highest?

soil
particle
D
A B C

xylem
root hair cell

37
© UCLES 2014 0610/02/SP/16 [Turn over
4

5 The diagram shows two solutions that are separated by a partially permeable membrane.

partially permeable
membrane

dilute concentrated
solution solution
of of
sucrose sucrose

X Y

In which direction will most water molecules move in relation to their concentration gradient?

A from X to Y against their concentration gradient


B from X to Y down their concentration gradient
C from Y to X against their concentration gradient
D from Y to X down their concentration gradient

6 Which statement describes active transport?

A the movement of ions through proteins in a cell membrane, releasing energy


B the movement of ions through proteins in a cell membrane, using energy
C the movement of water through lipids in a cell membrane, releasing energy
D the movement of water through lipids in a cell membrane, using energy

7 The diagram shows an experiment using an uncooked potato. The skin of the potato was
removed as shown.

concentrated
sugar solution

potato
skin has been
water removed

Which diagram shows the result of the experiment after 24 hours?

A B C D

38
© UCLES 2014 0610/02/SP/16
5

8 One strand of a DNA molecule contains the base sequence AGT.

What is the base sequence on the other strand of the DNA molecule?

A AGT
B GAT
C TAC
D TCA

9 The following can be used to write a word equation for photosynthesis.

1 carbon dioxide and water

2 light and chlorophyll

3 glucose and oxygen

Which shows a correct word equation for photosynthesis?

A 1 → 2 in the presence of 3

B 1 → 3 in the presence of 2

C 2 → 3 in the presence of 1

D 3 → 1 in the presence of 2

10 Which row shows the most likely number of chloroplasts in three types of cell in a leaf?

vascular
epidermis mesophyll
bundle cell

A 0 6 17
B 0 17 0
C 17 6 0
D 17 0 6

39
© UCLES 2014 0610/02/SP/16 [Turn over
6

11 The diagram shows a tooth.

bone

What are the parts labelled 1, 2 and 3?

1 2 3

A dentine enamel pulp


B enamel dentine pulp
C enamel pulp dentine
D pulp dentine enamel

12 What is the function of the lacteal in a villus?

A absorbs the fatty acids


B allows substances to pass from the lumen of the intestine into the villus
C takes oxygen to the villus
D absorbs the glucose and amino acids

13 On a dry, sunny day, how does water vapour move through the stomata of a leaf?

A into the leaf by diffusion


B into the leaf by respiration
C out of the leaf by diffusion
D out of the leaf by respiration

40
© UCLES 2014 0610/02/SP/16
7

14 The diagram shows a capillary and some body cells.

body cell

Q
P capillary wall

How do ions pass from P to Q?

A by diffusion in blood
B by diffusion in tissue fluid
C by osmosis in blood
D by osmosis in tissue fluid

15 These actions may be important in controlling the spread of disease.

1 washing hands after going to the toilet

2 disposing of waste frequently

3 using separate cutting boards for meat and salad

4 disposing of raw sewage into a river

Which would help control the spread of disease?

A 1, 2, 3 and 4
B 1, 2 and 3 only
C 2 and 3 only
D 4 only

16 New-born babies have passive immunity.

Why is this only temporary?

A No memory cells are produced in the baby.


B The antibodies are insufficient in number.
C The antibodies only act in the mother.
D The immunity is not inherited.

41
© UCLES 2014 0610/02/SP/16 [Turn over
8

17 The table shows some of the features of respiration.

Which row is correct for anaerobic respiration?

amount of energy
releases
released per chemical reaction
carbon dioxide
glucose molecule

A high always the same sometimes


B low different in different organisms sometimes
C high different in different organisms always
D low always the same always

18 The diagram shows some apparatus used to investigate respiration. Yeast, warm water and
substance Z were put into a test-tube. After a while, the limewater began to go cloudy.

yeast, warm water limewater


and substance Z

What is substance Z?

A alcohol
B glucose
C nitrogen
D oxygen

19 What happens during the process of breathing out?

external intercostal pressure in the lungs volume of thorax


diaphragm contracts
muscles contract increases increases

A yes yes no yes


B yes no yes yes
C no yes no no
D no no yes no

42
© UCLES 2014 0610/02/SP/16
9

20 The diagram shows a kidney tubule and some of its associated blood vessels.

Which substance is entirely reabsorbed from the fluid at R to the blood at P?

A glucose
B salts
C urea
D water

21 Which of the following can be an effector in a reflex arc?

A a gland
B a light receptor
C the brain
D the spinal cord

43
© UCLES 2014 0610/02/SP/16 [Turn over
10

22 The diagram shows a section through a human eye.

When focusing on a close object at night, what is the state of structures P and Q?

P Q

A contracted tight
B contracted slack
C relaxed tight
D relaxed slack

23 The diagram shows a shoot that has been placed on its side. The shoot begins to grow upwards.

up
P

What causes the shoot to grow upwards?

A increased cell division by meiosis at P


B increased cell division by mitosis at P
C more cell elongation at P than Q
D more cell elongation at Q than P

44
© UCLES 2014 0610/02/SP/16
11

24 Some roots are known to be gravitropic.

Which pair of diagrams show a controlled experiment to find out if these roots are also
phototropic?

light proof light proof light


box box

light

1 2 3 4

A 1 and 2
B 1 and 3
C 2 and 3
D 2 and 4

25 Which feature of sexual reproduction helps a species to evolve?

A Fewer offspring are produced than in asexual reproduction.


B Offspring are the result of the fusion of the nuclei of dissimilar gametes.
C Offspring always inherit advantageous characteristics.
D Offspring produced will always be in a suitable environment.

26 The diagram shows the female reproductive system.

What is the function of the part labelled X?

A gamete production and hormone secretion


B gamete production only
C hormone secretion only
D zygote production and hormone secretion

45
© UCLES 2014 0610/02/SP/16 [Turn over
12

27 The diagram shows the menstrual cycle of a woman during the month of September.

Sunday Monday Tuesday Wednesday Thursday Friday Saturday

1 2 3 4 5 6

7 8 9 10 11 12 13

14 15 16 17 18 19 20

21 22 23 24 25 26 27

28 29 30

key

= ovulation

= menstruation

Why can fertilisation not take place if sperms are released into the vagina on 8th September?

A Sperms are washed out of the female uterus by the menstrual flow.
B Sperms can survive in the female reproductive system for only 3 or 4 days.
C Sperms must be released after ovulation for fertilisation to take place.
D The uterus lining is washed out of the female body during menstruation.

28 Which states a function of the four hormones FSH, LH, progesterone and oestrogen in controlling
the menstrual cycle?

FSH LH progesterone oestrogen

A stimulates the maintains the lining causes an egg to causes the mature
pituitary gland to of the uterus during mature in an ovary egg to be released
release LH the middle part of from the ovary
the menstrual cycle

B causes an egg to causes the mature prepares the uterus stimulates the
mature in an ovary egg to be released to receive a fertilised pituitary gland to
from the ovary ovum release LH

C prepares the uterus stimulates the stops FSH being maintains the lining
to receive a fertilised ovaries to release produced of the uterus during
ovum oestrogen the middle part of
the menstrual cycle

D maintains the lining stops FSH being causes the mature prepares the uterus
of the uterus during produced egg to be released to receive a fertilised
the middle part of from the ovary ovum
the menstrual cycle

46
© UCLES 2014 0610/02/SP/16
13

29 The diagram shows a cell of an organism formed by reduction division. The nucleus contains
20 chromosomes.

What is the diploid number for this organism?

A 10 B 20 C 40 D 46

30 What are the possible blood groups of the offspring of parents who have blood group A and B?

A AB only
B A and B
C A, B and AB
D A, B, AB and O

47
© UCLES 2014 0610/02/SP/16 [Turn over
14

31 The diagram shows a family tree and the inheritance of the ability to taste a certain substance.

The allele for the ability to taste this substance is dominant.

1st generation David Mary Jeanne Mark

2nd generation

key
male ‘taster’ male ‘non-taster’

female ‘taster’ female ‘non-taster’

Which statement about the genotypes of the sisters Mary and Jeanne is correct?

A Mary is heterozygous and Jeanne is homozygous.


B Mary is homozygous and Jeanne is heterozygous.
C They are both heterozygous.
D They are both homozygous.

32 Haemophilia is a sex-linked recessive condition. A haemophiliac man has one son who has
inherited haemophilia and two more sons who have not.

The man’s wife is pregnant again. If this baby is a girl, what is the chance that she will have
haemophilia?

A 0% B 25 % C 50 % D 75 %

33 Antibiotics often do not work as effectively against bacterial infections as they used to.

What change has taken place in the bacteria, and what type of selection has brought this about?

change in the bacteria type of selection

A have become immune to the antibiotic artificial


B have become immune to the antibiotic natural
C have become resistant to the antibiotic artificial
D have become resistant to the antibiotic natural
48
© UCLES 2014 0610/02/SP/16
15

34 The diagram shows an Arctic ecosystem.

polar bear

glaucous gull ringed seal

little auk arctic cod

squid

amphipods shrimps

herbivorous
copepods

phytoplankton

sunlight

How many organisms can act as tertiary consumers in this food web?

A 3 B 4 C 5 D 6

49
© UCLES 2014 0610/02/SP/16 [Turn over
16

35 The diagram shows the circulation of nitrogen in nature.

nitrogen
in the air
3
2

lightning
ammonium organic
nitrate in nitrogen
1 fertiliser NO in plants
legumes

plants NH4+
5
2 in soil

nitrate animals
in soil
nitrite
in soil
4 4

What is correct?

denitrification nitrification nitrogen fixation

A 1 5 3
B 2 5 4
C 3 2 1
D 1 4 3

36 What is the correct match of example to ecological term?

community ecosystem population trophic level

A all lake organisms freshwater lake freshwater shrimps pond weed as


primary producer
B freshwater shrimps all lake organisms pond weed as freshwater lake
primary producer
C freshwater lake pond weed as freshwater shrimps all lake organisms
primary producer
D freshwater shrimps freshwater lake all lake organisms pond weed as
primary producer

50
© UCLES 2014 0610/02/SP/16
17

37 Some rabbits colonised an island for the first time. The graph shows how their population size
changed over the next few years.

population
size

time

What explains the way the size of the rabbit population changed during the exponential (log)
phase?

A birth rate and death rate in equilibrium


B increasing number of rabbits able to reproduce
C increase in the number of predators
D limiting factors begin to take effect

51
© UCLES 2014 0610/02/SP/16 [Turn over
18

38 The diagram shows a bacterium whose plasmid is being used during genetic engineering to
produce human insulin.

section of
DNA cut
and removed

plasmid

What is inserted at P so that the bacterium can produce human insulin, and which enzyme is
used to catalyse the insertion?

A a section of human DNA, using DNA ligase


B a section of human DNA, using restriction enzymes
C a section of human mRNA, using DNA ligase
D a section of human mRNA, using restriction enzymes

39 Modern technology has resulted in the production and use of insecticides.

Which statement is a false description of insecticides?

A Insecticides can affect food chains.


B Insecticides can cause pollution.
C Insecticides improve yields.
D Insecticides target weeds.

40 Which of these measures will help to prevent acid rain?

A avoiding the use of non-recyclable plastics


B filtering sulfur dioxide in power station chimneys
C reducing methane emissions in industry and agriculture
D using alkaline fertilisers on fields

Permission to reproduce items where third-party owned material protected by copyright is included has been sought and cleared where possible. Every
reasonable effort has been made by the publisher (UCLES) to trace copyright holders, but if any items requiring clearance have unwittingly been included, the
publisher will be pleased to make amends at the earliest possible opportunity.

52
Cambridge International Examinations is part of the Cambridge Assessment Group. Cambridge Assessment is the brand name of University of Cambridge Local
Examinations Syndicate (UCLES), which is itself a department of the University of Cambridge.

© UCLES 2014 0610/02/SP/16


Cambridge International Examinations
Cambridge International General Certificate of Secondary Education
*0123456789*

BIOLOGY 0610/01
Paper 1 Multiple Choice (Core) For Examination from 2016
SPECIMEN PAPER
45 minutes
Additional Materials: Multiple Choice Answer Sheet
Soft clean eraser
Soft pencil (type B or HB is recommended)

READ THESE INSTRUCTIONS FIRST

Write in soft pencil.


Do not use staples, paper clips, glue or correction fluid.
Write your name, Centre number and candidate number on the Answer Sheet in the spaces provided unless
this has been done for you.
DO NOT WRITE IN ANY BARCODES.

There are forty questions on this paper. Answer all questions. For each question there are four possible
answers A, B, C and D.
Choose the one you consider correct and record your choice in soft pencil on the separate Answer Sheet.

Read the instructions on the Answer Sheet very carefully.

Each correct answer will score one mark. A mark will not be deducted for a wrong answer.
Any rough working should be done in this booklet.
Electronic calculators may be used.

The syllabus is accredited for use in England, Wales and Northern Ireland as a Cambridge International Level 1/Level 2 Certificate.

This document consists of 16 printed pages.

© UCLES 2014 53
[Turn over
2

1 The diagram shows a structure found in the human body.

What is this structure an example of?

A an organ
B an organism
C an organ system
D a tissue

2 Which system is used for naming species?

A binomial
B conservation
C dichotomous
D natural selection

54
© UCLES 2014 0610/01/SP/16
3

3 The diagram shows a section through a flower.

petal

Using the key, identify this flower.

1 sepals present ............................................. go to 2


sepals not present ....................................... go to 3
2 stamens attached to petals ......................... A
stamens not attached to petals ................... B
3 stigma above anthers .................................. C
stigma below anthers .................................. D

4 The table shows some features of animals.

Which animal could be a bird?

feature
feathers gills hair wings

A     key
B      = present
C      = not present
D    

5 The diagram shows some liver cells as they appear under a microscope.

How many cell walls can be seen?

A 0 B 2 C 5 D 10

55
© UCLES 2014 0610/01/SP/16 [Turn over
4

6 Which row shows the most likely number of chloroplasts in three types of cell in a leaf?

vascular
epidermis mesophyll
bundle cell

A 0 6 17
B 0 17 0
C 17 6 0
D 17 0 6

7 The diagram shows an experiment using an uncooked potato. The skin of the potato was
removed as shown.

concentrated
sugar solution

potato
skin has been
water removed

Which diagram shows the result of the experiment after 24 hours?

A B C D

8 A student draws a diagram of a plant cell.

The diagram is 40 mm in width. The plant cell is 0.02 mm in width.

What is the magnification of the student’s drawing?

A ×0.005

B ×0.08

C ×200

D ×2000

9 On a dry, sunny day, how does water vapour move through the stomata of a leaf?

A into the leaf by diffusion


B into the leaf by respiration
C out of the leaf by diffusion
D out of the leaf by respiration
56
© UCLES 2014 0610/01/SP/16
5

10 The diagram shows two solutions that are separated by a partially permeable membrane.

partially permeable
membrane

dilute concentrated
solution solution
of of
sucrose sucrose

X Y

In which direction will most water molecules move in relation to their concentration gradient?

A from X to Y against their concentration gradient


B from X to Y down their concentration gradient
C from Y to X against their concentration gradient
D from Y to X down their concentration gradient

11 The graph shows how the rate of an enzyme-controlled reaction changes with temperature.

rate of
reaction

temperature X

What describes the shape of the graph within the temperature range marked X?

A The rate of reaction decreases.


B The rate of reaction increases then decreases.
C The rate of reaction reaches a maximum.
D The reaction is occurring at the optimum temperature.

57
© UCLES 2014 0610/01/SP/16 [Turn over
6

12 Six test-tubes were set up at different pH values. They each contained the same quantity of
starch and amylase in water. The table shows the time taken for the reaction to be completed in
each test-tube.

pH 1 3 5 7 9 11

time to be completed / s 95 64 42 5 35 66

At which pH does the amylase work best?

A pH 1 B pH 5 C pH 7 D pH 11

13 What are the chemical elements found in carbohydrates?

A carbon, hydrogen and oxygen only


B carbon, hydrogen, oxygen and nitrogen only
C carbon, hydrogen, oxygen and sulfur only
D carbon, hydrogen, oxygen, nitrogen and sulfur only

14 A protease is added to a cloudy suspension of protein in a test-tube and kept at 37 °C.

After eight minutes, the suspension changes from cloudy to transparent.

Which product, or products, will now be present in the test-tube?

A amino acids
B fatty acids
C glycerol
D simple sugars

15 The following can be used to write a word equation for photosynthesis.

1 carbon dioxide and water

2 light and chlorophyll

3 glucose and oxygen

Which shows a correct word equation for photosynthesis?

A 1 → 2 in the presence of 3

B 1 → 3 in the presence of 2

C 2 → 3 in the presence of 1

D 3 → 1 in the presence of 2

58
© UCLES 2014 0610/01/SP/16
7

16 The photograph shows a section through the leaf of a plant.

Which label shows the position of the cuticle?

B
D

17 What is a symptom of scurvy?

A bleeding gums
B breathlessness
C diarrhoea
D pain in joints

18 What is an appropriate treatment for severe diarrhoea?

A avoiding drinking
B avoiding high-fibre foods
C drinking a dilute solution of salt and glucose
D eating high-fat foods

59
© UCLES 2014 0610/01/SP/16 [Turn over
8

19 The diagram shows a tooth.

bone

What are the parts labelled 1, 2 and 3?

1 2 3

A dentine enamel pulp


B enamel dentine pulp
C enamel pulp dentine
D pulp dentine enamel

20 The diagram shows a human heart and some of its major blood vessels.

Blockage of which labelled part is a cause of coronary heart disease?

60
© UCLES 2014 0610/01/SP/16
9

21 These actions may be important in controlling the spread of disease.

1 washing hands after going to the toilet

2 disposing of waste frequently

3 using separate cutting boards for meat and salad

4 disposing of raw sewage into a river

Which would help control the spread of disease?

A 1, 2, 3 and 4
B 1, 2 and 3 only
C 2 and 3 only
D 4 only

22 The table shows some of the features of respiration.

Which row is correct for anaerobic respiration?

amount of energy
releases
released per chemical reaction
carbon dioxide
glucose molecule

A high always the same sometimes


B low different in different organisms sometimes
C high different in different organisms always
D low always the same always

61
© UCLES 2014 0610/01/SP/16 [Turn over
10

23 The diagram shows some apparatus used to investigate respiration. Yeast, warm water and
substance Z were put into a test-tube. After a while, the limewater began to go cloudy.

yeast, warm water limewater


and substance Z

What is substance Z?

A alcohol
B glucose
C nitrogen
D oxygen

24 The diagram shows someone blowing up a balloon.

How do the proportions of gases in the air inside the balloon compare with the air outside the
balloon?

carbon dioxide oxygen water vapour

A less more more


B less more less
C more less more
D more less less

62
© UCLES 2014 0610/01/SP/16
11

25 Which organ produces urea?

A bladder
B kidney
C liver
D pancreas

26 The diagram shows a neurone carrying an impulse.

direction of impulse

Which row describes the type of neurone and the direction of impulse?

type of neurone direction of impulse

A motor towards the spinal cord


B motor away from the spinal cord
C sensory towards the spinal cord
D sensory away from the spinal cord

27 Which of the following can be an effector in a reflex arc?

A a gland
B a light receptor
C the brain
D the spinal cord

63
© UCLES 2014 0610/01/SP/16 [Turn over
12

28 The diagram shows the female reproductive system.

What is the function of the part labelled X?

A gamete production and hormone secretion


B gamete production only
C hormone secretion only
D zygote production and hormone secretion

29 The diagram shows the menstrual cycle of a woman during the month of September.

Sunday Monday Tuesday Wednesday Thursday Friday Saturday

1 2 3 4 5 6

7 8 9 10 11 12 13

14 15 16 17 18 19 20

21 22 23 24 25 26 27

28 29 30

key

= ovulation

= menstruation

Why can fertilisation not take place if sperms are released into the vagina on 8th September?

A Sperms are washed out of the female uterus by the menstrual flow.
B Sperms can survive in the female reproductive system for only 3 or 4 days.
C Sperms must be released after ovulation for fertilisation to take place.
D The uterus lining is washed out of the female body during menstruation.

64
© UCLES 2014 0610/01/SP/16
13

30 Some roots are known to be gravitropic.

Which pair of diagrams show a controlled experiment to find out if these roots are also
phototropic?

light proof light proof light


box box

light

1 2 3 4

A 1 and 2
B 1 and 3
C 2 and 3
D 2 and 4

31 What term is used for the transfer of a gene from one organism to another?

A artificial selection
B genetic engineering
C mutation
D natural selection

32 In an experiment to investigate the effects of various environmental factors on germination, four


boiling tubes were set up as shown.

In which tube would the seeds germinate most quickly?

A B C D
cotton wool

cress seed
moist moist
moist dry cotton wool cotton wool
cotton wool cotton wool
substance to
water absorb O2 water

25 °C 25 °C 25 °C 5 °C

65
© UCLES 2014 0610/01/SP/16 [Turn over
14

33 The diagram shows some parts of the male reproductive system.

Which part is cut during a vasectomy?

34 The diagram shows a family tree and the inheritance of the ability to taste a certain substance.

The allele for the ability to taste this substance is dominant.

1st generation David Mary Jeanne Mark

2nd generation

key
male ‘taster’ male ‘non-taster’

female ‘taster’ female ‘non-taster’

Which statement about the genotypes of the sisters Mary and Jeanne is correct?

A Mary is heterozygous and Jeanne is homozygous.


B Mary is homozygous and Jeanne is heterozygous.
C They are both heterozygous.
D They are both homozygous.

66
© UCLES 2014 0610/01/SP/16
15

35 The diagram shows an eagle.

Eagles have hooked beaks that help them to ingest food.

What is this an example of?

A adaptation
B assimilation
C nutrition
D variation

36 In which order do organisms occur in the food chain?

A carnivore → herbivore → producer


B herbivore → carnivore → producer
C producer → carnivore → herbivore
D producer → herbivore → carnivore

37 How is energy transferred between organisms in a food chain?

A combustion
B ingestion
C photosynthesis
D respiration

67
© UCLES 2014 0610/01/SP/16 [Turn over
16

38 The diagram shows part of a section through a leaf in daylight.

What substance follows the path of the arrow out of the leaf?

A carbon dioxide
B energy
C oxygen
D water

39 Which factors, if present, may cause a population of animals to decrease?

predation disease

A  
B  
C  
D  

40 Modern technology has resulted in the production and use of insecticides.

Which statement is a false description of insecticides?

A Insecticides can affect food chains.


B Insecticides can cause pollution.
C Insecticides improve yields.
D Insecticides target weeds.

Permission to reproduce items where third-party owned material protected by copyright is included has been sought and cleared where possible. Every
reasonable effort has been made by the publisher (UCLES) to trace copyright holders, but if any items requiring clearance have unwittingly been included, the
publisher will be pleased to make amends at the earliest possible opportunity.

68
Cambridge International Examinations is part of the Cambridge Assessment Group. Cambridge Assessment is the brand name of University of Cambridge Local
Examinations Syndicate (UCLES), which is itself a department of the University of Cambridge.

© UCLES 2014 0610/01/SP/16


Cambridge International Examinations
Cambridge International General Certificate of Secondary Education

BIOLOGY 0610/21
Paper 2 Multiple Choice (Extended) October/November 2018
45 minutes
Additional Materials: Multiple Choice Answer Sheet
Soft clean eraser
*3938804197*

Soft pencil (type B or HB is recommended)

READ THESE INSTRUCTIONS FIRST

Write in soft pencil.


Do not use staples, paper clips, glue or correction fluid.
Write your name, Centre number and candidate number on the Answer Sheet in the spaces provided
unless this has been done for you.
DO NOT WRITE IN ANY BARCODES.

There are forty questions on this paper. Answer all questions. For each question there are four possible
answers A, B, C and D.
Choose the one you consider correct and record your choice in soft pencil on the separate Answer Sheet.

Read the instructions on the Answer Sheet very carefully.

Each correct answer will score one mark. A mark will not be deducted for a wrong answer.
Any rough working should be done in this booklet.
Electronic calculators may be used.

This syllabus is approved for use in England, Wales and Northern Ireland as a Cambridge International Level 1/Level 2 Certificate.

This document consists of 18 printed pages and 2 blank pages.

IB18 11_0610_21/3RP
© UCLES 2018 69
[Turn over
2

1 The sundew is a carnivorous plant that can trap small insects with sticky hairs and then digest
them. When an insect gets stuck, other nearby sticky hairs bend over to trap the insect.

Which characteristics of living organisms are demonstrated when the sundew traps insects?

A growth and excretion


B growth and sensitivity
C movement and excretion
D movement and sensitivity

2 Systems of classification show which organisms share more recent ancestors.

What is the most accurate system of classification?

A using anatomy
B using DNA base sequences
C using morphology
D using a pedigree diagram

3 Which part of a plant cell controls the movement of substances into and out of the cell?

A cell membrane
B cell wall
C cytoplasm
D vacuole

70
© UCLES 2018 0610/21/O/N/18
3

4 The diagram shows part of a leaf in cross-section.

Structures X and Y are both part of the same

A cell.
B organ.
C tissue.
D vessel.

5 How do carbon dioxide and oxygen move into and out of a mesophyll cell?

A active transport
B diffusion
C respiration
D transpiration

71
© UCLES 2018 0610/21/O/N/18 [Turn over
4

6 Four freshly cut potato cylinders were soaked for one hour in different salt solutions before being
pinned to cork blocks. Two of the blocks are shown.

pin rigid
flaccid
potato cylinder
potato cylinder

cork block

Which solution would cause the potato cylinder to be most flaccid?

A 0.1 mol per dm3 salt solution


B 0.3 mol per dm3 salt solution
C 0.7 mol per dm3 salt solution
D 1.0 mol per dm3 salt solution

7 The data show the concentrations of sugar and starch in an onion.

total sugar including


starch
reducing sugar
/ g per 100g
/ g per 100g

3.7 0.0

The onion is tested with Benedict’s solution and iodine solution.

Which set of results is correct?

Benedict’s iodine
solution solution

A blue blue-black
B blue brown
C brick red blue-black
D brick red brown

8 Which statement about the structure of DNA is correct?

A Base A always pairs with base C.


B Base A always pairs with base T.
C It consists of proteins.
D It forms a single helix.

72
© UCLES 2018 0610/21/O/N/18
5

9 The apparatus shown is used for an experiment on starch digestion.

Which test-tube contains the most sugar after 20 minutes?

A B C D

starch starch
solution starch solution
and salivary solution and salivary
amylase amylase

water-bath at 15 °C water-bath at 37 °C

10 A student wrote some notes about enzymes.

She wrote:

‘The ......1...... of the enzyme is ......2...... to an area on the substrate.

This area on the substrate can fit into it to form an ......3...... complex’.

Which words correctly complete gaps 1, 2 and 3?

1 2 3

A active site complementary enzyme-substrate


B active site similar enzyme-product
C shape complementary enzyme-product
D shape similar enzyme-substrate

73
© UCLES 2018 0610/21/O/N/18 [Turn over
6

11 An experiment was carried out using the apparatus shown.

The carbon dioxide content of the water in each test-tube was measured at the start and again
three hours later.

In which test-tube would there be a decrease in carbon dioxide content?

A B C D

black
water polythene
to keep
out light light light light
water
plant

water
snail

12 The diagram shows a plant cell.

cell membrane vacuole

nucleus
cell wall

cytoplasm
chloroplast

Which type of cell is shown?

A cuticle cell
B epidermal cell
C palisade mesophyll cell
D spongy mesophyll cell

13 What is the result of a diet lacking iron?

A bleeding gums
B poor wound healing
C reduced number of red blood cells
D weak bones and teeth

74
© UCLES 2018 0610/21/O/N/18
7

14 The diagram shows part of the alimentary canal.

Which structure produces lipase?

A
C

15 The photomicrograph shows a cross-section through a buttercup root.

What is the function of the tissue labelled Z?

A site of photosynthesis
B site of respiration
C transport of sugars
D transport of water

75
© UCLES 2018 0610/21/O/N/18 [Turn over
8

16 Roots and leaves both act as a source and a sink for sucrose and amino acids at different times
during the year.

At which point in the year are the roots most active as a source?

A
spring
new leaves
start to grow

D B
winter summer
no leaves leaves are
are present mature

C
autumn
leaves die
and fall

17 The diagram shows a circulatory system.

2 3

capillaries of capillaries of the


the lungs heart rest of the body

1 4

Which vessels carry oxygenated blood?

A 1 and 2 B 1 and 4 C 2 and 3 D 2 and 4

18 What happens to the heart valves when the ventricles contract?

atrioventricular semilunar
valves valves

A valves close valves close


B valves close valves open
C valves open valves close
D valves open valves open
76
© UCLES 2018 0610/21/O/N/18
9

19 The diagram with the structure labelled X shows a bacterium with proteins on its surface. The
diagram labelled Y shows proteins made by the human body.

Which row shows the correct combination for destroying the bacterium?

correct
name of X name of Y
shape of Y

A antigen antibody

B antibody antigen

C antigen antibody

D antibody antigen

77
© UCLES 2018 0610/21/O/N/18 [Turn over
10

20 The graph shows changes in the volume of air in the lungs of a person at rest, over a period of 30
seconds.

2
volume
of air in
lungs 1
/ dm3
0
0 30
time / s

Which graph shows changes in the volume of air in the lungs of the same person immediately
after they have done five minutes of vigorous exercise?

A B
2 2
volume volume
of air in of air in
lungs 1 lungs 1
/ dm3 / dm3
0 0
0 30 0 30
time / s time / s

C D
2 2
volume volume
of air in of air in
lungs 1 lungs 1
/ dm3 / dm3
0 0
0 30 0 30
time / s time / s

21 What is produced by anaerobic respiration in mammals?

A alcohol + carbon dioxide


B alcohol + oxygen
C lactic acid + carbon dioxide
D lactic acid

78
© UCLES 2018 0610/21/O/N/18
11

22 Which row describes the functions of the bladder, kidneys and liver?

production excretion storage


of urea of urea of urine

A liver bladder kidneys


B bladder kidneys liver
C liver kidneys bladder
D kidneys liver bladder

23 The diagram shows the structure of the eye.

Which structure refracts light?

A
C

79
© UCLES 2018 0610/21/O/N/18 [Turn over
12

24 A student used two seedlings X and Y to investigate phototropism.

The diagram shows their investigation.

light light
piece of glass piece of glass

X X
at the start of after some time
the experiment no curvature towards light

light light

piece of glass piece of glass

Y Y
at the start of after some time
the experiment gives curvature towards light

Which statement explains the difference in results between X and Y?

A The piece of glass destroyed the auxin on the shaded side of the seedling.
B The piece of glass destroyed the auxin on the side of the seedling facing the light.
C The piece of glass in X stopped the auxin travelling down the shaded side of the seedling.
D The piece of glass in X stopped the auxin travelling down the side of the seedling facing the
light.

80
© UCLES 2018 0610/21/O/N/18
13

25 The diagram shows part of the mechanism that controls blood sugar concentration.

liver

1 more glucose 3 more glycogen


2 less glucose 4 less glycogen blood vessels

blood vessels 5 more glucagon


6 less glucagon

pancreas

A person does one hour of exercise.

Starting with the pancreas, what is the sequence of events in which the hormone glucagon is
involved?

A 5→3→2 B 5→4→1 C 6→3→1 D 6→4→2

26 The graph shows the number of cases of MRSA in one country between 2001 and 2006.

8000

7250
number of
cases of 6500
MRSA
5750

5000
2001 2002 2003 2004 2005 2006
year

Between which years was the greatest change in the number of cases of MRSA seen?

A 2002 and 2003


B 2003 and 2004
C 2004 and 2005
D 2005 and 2006

81
© UCLES 2018 0610/21/O/N/18 [Turn over
14

27 Which row describes sexual reproduction?

gamete zygote genetically different


nucleus nucleus offspring produced

A diploid diploid 
B diploid haploid 
C haploid diploid 
D haploid haploid 

28 Which statement describes human male gametes?

A large, few and non-motile


B large, numerous and motile
C small, few and non-motile
D small, numerous and motile

29 Which organ secretes the most progesterone during pregnancy?

A adrenal gland
B ovary
C placenta
D uterus

30 What is the role of mRNA?

A assembles amino acids into protein molecules


B carries a copy of the gene out of the nucleus
C controls cell function by controlling the production of proteins
D duplicates chromosomes before mitosis

82
© UCLES 2018 0610/21/O/N/18
15

31 The diagram shows the formation of new diploid cells.

Y
X

What do arrows X and Y represent?

X Y

A duplication of chromosomes meiosis


B duplication of chromosomes mitosis
C meiosis duplication of chromosomes
D mitosis duplication of chromosomes

32 A man marries a woman who has a different blood group from him. They have two children. The
children have different blood groups from each other and different blood groups from their
parents.

What are the genotypes of the parent’s blood groups?

A IAIA and IAIB B IAIA and IoIo C IAIB and IBIB D IAIB and IoIo

33 Sickle-cell anaemia is a genetic disease that is caused by the allele HbS.

HbA is the normal allele.

A woman does not have the symptoms of the disease but her brother does show symptoms.

If her mother and father do not have symptoms, which statement is correct?

A The woman has the genotype HbSHbS.


B The woman’s brother has the genotype HbAHbA.
C The woman’s father and mother both have the genotype HbAHbS.
D The woman’s father and mother both have the genotype HbSHbS.

83
© UCLES 2018 0610/21/O/N/18 [Turn over
16

34 Over the last 30 years some antibiotics have become less effective in treating bacterial infection.

What is the reason for this?

A artificial selection
B asexual reproduction
C more effective new antibiotics
D natural selection

35 The diagram shows part of the nitrogen cycle.

nitrogen in
atmosphere
Z
W
animals plants
nitrate
ions
Y

X
ammonium ions

Which row correctly identifies the bacteria involved in processes W, X, Y and Z?

W X Y Z

A denitrifying decomposer nitrifying nitrogen-fixing


B denitrifying nitrifying decomposer nitrogen-fixing
C nitrifying decomposer nitrogen-fixing denitrifying
D nitrogen-fixing nitrifying decomposer denitrifying

84
© UCLES 2018 0610/21/O/N/18
18

40 Pieces of plastic between 1 µm and 1 mm in size are called microplastics. Microplastics are put
into some face creams and are produced during clothing manufacture. They can now be found in
increasing quantities in oceans all over the world.

As well as their small size, which other property of microplastics make them dangerous to living
organisms?

A They are lightweight.


B They are non-biodegradable.
C They are non-reactive.
D They are toxic.

85
© UCLES 2018 0610/21/O/N/18
Cambridge International Examinations
Cambridge International General Certificate of Secondary Education

BIOLOGY 0610/22
Paper 2 Multiple Choice (Extended) October/November 2018
45 minutes
Additional Materials: Multiple Choice Answer Sheet
Soft clean eraser
*3397490337*

Soft pencil (type B or HB is recommended)

READ THESE INSTRUCTIONS FIRST

Write in soft pencil.


Do not use staples, paper clips, glue or correction fluid.
Write your name, Centre number and candidate number on the Answer Sheet in the spaces provided
unless this has been done for you.
DO NOT WRITE IN ANY BARCODES.

There are forty questions on this paper. Answer all questions. For each question there are four possible
answers A, B, C and D.
Choose the one you consider correct and record your choice in soft pencil on the separate Answer Sheet.

Read the instructions on the Answer Sheet very carefully.

Each correct answer will score one mark. A mark will not be deducted for a wrong answer.
Any rough working should be done in this booklet.
Electronic calculators may be used.

This syllabus is approved for use in England, Wales and Northern Ireland as a Cambridge International Level 1/Level 2 Certificate.

This document consists of 18 printed pages and 2 blank pages.

IB18 11_0610_22/3RP
© UCLES 2018 86
[Turn over
2

1 A person drinks a glass of iced water and the volume of sweat they secrete decreases.

This is an example of which characteristic of living organisms?

A growth
B movement
C respiration
D sensitivity

2 Lichens are formed from two different organisms living together.

The table shows some of the characteristics of two organisms, X and Y, found in most lichens.

X Y
made of strands called hyphae single celled
hyphae have cell walls cell contains a nucleus
and many nuclei and chloroplasts

Which kingdoms are represented by X and Y?

X Y

A Fungus Plant
B Fungus Protoctist
C Protoctist Fungus
D Protoctist Plant

3 Two types of cell, one animal and one plant, were examined using a light microscope.

Which row shows the correct combination of cellular features that would be observed in the cells?

cell structure observed


animal cell plant cell

A chloroplast membrane vacuole cytoplasm


B cytoplasm nucleus chloroplast membrane
C membrane cell wall cytoplasm nucleus
D nucleus chloroplast cell wall membrane

87
© UCLES 2018 0610/22/O/N/18
3

4 The diagram shows part of a leaf in cross-section.

Structures X and Y are both part of the same

A cell.
B organ.
C tissue.
D vessel.

5 How do carbon dioxide and oxygen move into and out of a mesophyll cell?

A active transport
B diffusion
C respiration
D transpiration

88
© UCLES 2018 0610/22/O/N/18 [Turn over
4

6 The diagram shows a plant cell after it has been submerged in a solution, P, for 20 minutes.

Which row describes the water potential of solution P and the condition of the cell?

water potential of solution P condition of the cell

A higher than the cell sap in the vacuole plasmolysed and turgid
B higher than the cell sap in the vacuole under high turgor pressure
C lower than the cell sap in the vacuole plasmolysed and flaccid
D the same as the cell sap in the vacuole under low turgor pressure

7 The data show the concentrations of sugar and starch in an onion.

total sugar including


starch
reducing sugar
/ g per 100g
/ g per 100g

3.7 0.0

The onion is tested with Benedict’s solution and iodine solution.

Which set of results is correct?

Benedict’s iodine
solution solution

A blue blue-black
B blue brown
C brick red blue-black
D brick red brown

8 The base sequence of part of one strand of a DNA molecule is shown.

ATAGCC

What is the base sequence of the other strand?

A GCGATT B CGCTAA C TATCGG D ATAGCC

89
© UCLES 2018 0610/22/O/N/18
5

9 The apparatus shown is used for an experiment on starch digestion.

Which test-tube contains the most sugar after 20 minutes?

A B C D

starch starch
solution starch solution
and salivary solution and salivary
amylase amylase

water-bath at 15 °C water-bath at 37 °C

10 The graph shows the effect of temperature on the action of an enzyme.

enzyme
activity

0 20 40 60 80
temperature / °C

Why does the rate of reaction change when the temperature is increased from 20 °C to 30 °C?

more kinetic more frequent


energy of particles collisions of particles

A  
B  
C  
D  

90
© UCLES 2018 0610/22/O/N/18 [Turn over
6

11 An experiment was carried out using the apparatus shown.

The carbon dioxide content of the water in each test-tube was measured at the start and again
three hours later.

In which test-tube would there be a decrease in carbon dioxide content?

A B C D

black
water polythene
to keep
out light light light light
water
plant

water
snail

12 The diagram shows the structure of cells from the leaf of a plant.

What type of cells are they?

A epidermal cells
B guard cells
C palisade cells
D spongy cells

13 What is the result of a diet lacking iron?

A bleeding gums
B poor wound healing
C reduced number of red blood cells
D weak bones and teeth

91
© UCLES 2018 0610/22/O/N/18
7

14 Which row shows an enzyme with the correct site of production and products?

enzyme product(s)
enzyme
produced by of digestion

A amylase salivary glands amino acids


B amylase stomach sugar
C protease salivary glands sugar
D protease stomach amino acids

15 The diagram shows a plant cell.

What type of plant cell is this?

A guard cell
B mesophyll cell
C root cortex cell
D root hair cell

92
© UCLES 2018 0610/22/O/N/18 [Turn over
8

16 Roots and leaves both act as a source and a sink for sucrose and amino acids at different times
during the year.

At which point in the year are the roots most active as a source?

A
spring
new leaves
start to grow

D B
winter summer
no leaves leaves are
are present mature

C
autumn
leaves die
and fall

17 The diagram shows a circulatory system.

2 3

capillaries of capillaries of the


the lungs heart rest of the body

1 4

Which vessels carry oxygenated blood?

A 1 and 2 B 1 and 4 C 2 and 3 D 2 and 4

18 What happens to the heart valves when the ventricles contract?

atrioventricular semilunar
valves valves

A valves close valves close


B valves close valves open
C valves open valves close
D valves open valves open
93
© UCLES 2018 0610/22/O/N/18
9

19 The diagram with the structure labelled X shows a bacterium with proteins on its surface. The
diagram labelled Y shows proteins made by the human body.

Which row shows the correct combination for destroying the bacterium?

correct
name of X name of Y
shape of Y

A antigen antibody

B antibody antigen

C antigen antibody

D antibody antigen

94
© UCLES 2018 0610/22/O/N/18 [Turn over
10

20 The graph shows changes in the volume of air in the lungs of a person at rest, over a period of 30
seconds.

2
volume
of air in
lungs 1
/ dm3
0
0 30
time / s

Which graph shows changes in the volume of air in the lungs of the same person immediately
after they have done five minutes of vigorous exercise?

A B
2 2
volume volume
of air in of air in
lungs 1 lungs 1
/ dm3 / dm3
0 0
0 30 0 30
time / s time / s

C D
2 2
volume volume
of air in of air in
lungs 1 lungs 1
/ dm3 / dm3
0 0
0 30 0 30
time / s time / s

21 As we breathe out, ......1...... is ......2...... through the lungs.

Which words correctly complete gaps 1 and 2?

1 2

A carbon dioxide excreted


B carbon dioxide respired
C oxygen excreted
D oxygen respired

95
© UCLES 2018 0610/22/O/N/18
11

22 A person carries out vigorous exercise without drinking any water.

What would happen to the concentration and volume of the person’s urine immediately after
exercise?

urine urine
concentration volume

A decrease decrease
B decrease increase
C increase decrease
D increase increase

23 What does the central nervous system consist of?

A brain, spinal cord and peripheral nerves


B brain and spinal cord only
C brain only
D spinal cord and peripheral nerves only

96
© UCLES 2018 0610/22/O/N/18 [Turn over
12

24 A student used two seedlings X and Y to investigate phototropism.

The diagram shows their investigation.

light light
piece of glass piece of glass

X X
at the start of after some time
the experiment no curvature towards light

light light

piece of glass piece of glass

Y Y
at the start of after some time
the experiment gives curvature towards light

Which statement explains the difference in results between X and Y?

A The piece of glass destroyed the auxin on the shaded side of the seedling.
B The piece of glass destroyed the auxin on the side of the seedling facing the light.
C The piece of glass in X stopped the auxin travelling down the shaded side of the seedling.
D The piece of glass in X stopped the auxin travelling down the side of the seedling facing the
light.

25 What is a response to a low concentration of glucose in the blood?

A Glucagon will cause the body to convert glucose into glycogen.


B Glucagon will cause the body to convert glycogen into glucose.
C Insulin will cause the body to convert glucose into glycogen.
D Insulin will cause the body to convert glycogen into glucose.

97
© UCLES 2018 0610/22/O/N/18
13

26 The graph shows the number of cases of MRSA in one country between 2001 and 2006.

8000

7250
number of
cases of 6500
MRSA
5750

5000
2001 2002 2003 2004 2005 2006
year

Between which years was the greatest change in the number of cases of MRSA seen?

A 2002 and 2003


B 2003 and 2004
C 2004 and 2005
D 2005 and 2006

27 What are two adaptive features of a human sperm cell?

1 jelly coat present


2 relatively high number of mitochondria
3 acrosome present
4 relatively high energy stores

A 1 and 2 B 1 and 4 C 2 and 3 D 3 and 4

28 Which hormone maintains the thickness of the lining of the uterus during pregnancy?

A follicle stimulating hormone (FSH)


B luteinising hormone (LH)
C oestrogen
D progesterone

29 What is a possible disadvantage of in vitro fertilisation (IVF)?

A Donated eggs and sperm can be used.


B Embryos can be screened for genetic disorders.
C It requires more medical resources.
D Unused embryos can be stored.

98
© UCLES 2018 0610/22/O/N/18 [Turn over
14

30 A sperm cell from a domestic cat contains 19 chromosomes.

If this cell fertilises an egg, which zygote is produced?

A diploid, with 19 chromosomes


B diploid, with 38 chromosomes
C haploid, with 19 chromosomes
D haploid, with 38 chromosomes

31 The diagram shows a cell of an organism. The nucleus contains 12 chromosomes.

After it divides by mitosis, how many chromosomes would be present in one of the daughter
cells?

A 6 B 12 C 18 D 24

32 Pure-breeding black-feathered chickens are mated with pure-breeding white-feathered chickens.


All of the individuals in the offspring in the F1 generation have both black and white feathers.

What will be the ratio of offspring phenotypes when two of the F1 generation chickens are
crossed?

A 1 black : 1 white
B 1 black : 2 black and white : 1 white
C 3 white : 1 black
D 3 black : 1 white

99
© UCLES 2018 0610/22/O/N/18
15

33 The diagram shows the bases on part of a chromosome, P, responsible for the production of
normal haemoglobin. The same part of another chromosome, Q, is responsible for the production
of sickle-cell haemoglobin.

G A G G T G
C T C C A C

chromosome P chromosome Q

What has caused the difference between the two chromosomes?

A discontinuous variation
B gene mutation
C phenotypic variation
D selective breeding

34 Farmers have bred Holstein-Friesian cattle to produce more milk than older breeds of cattle.

Which process was used to produce these cattle?

A adaptation
B genetic engineering
C natural selection
D selective breeding

100
© UCLES 2018 0610/22/O/N/18 [Turn over
16

35 The diagram shows a food web.

hedgehog
robin

caterpillar
greenfly beetle

rose plant cabbage plant

Which row shows a food chain in this food web?

primary secondary
producer
consumer consumer

A hedgehog caterpillar robin


B cabbage plant greenfly beetle
C cabbage plant beetle robin
D rose plant hedgehog greenfly

36 The diagram shows part of the nitrogen cycle.

Which change is caused by the action of denitrifying bacteria?

nitrogen gas
in the air

D A plants
death
nitrate in B
feeding
the soil
death
animals
C

faeces and
urine

101
© UCLES 2018 0610/22/O/N/18
17

37 The diagram shows an industrial fermenter used to produce penicillin.

What is a function of the part labelled X?

A add oxygen to the solution


B maintain an even temperature throughout the solution
C record the pH of the solution
D sterilise the solution

38 Genetic engineering involves various stages.

1 human DNA is inserted into bacterial plasmid DNA


2 recombinant plasmid inserted into bacteria
3 restriction enzyme cuts bacterial plasmid DNA
4 restriction enzyme cuts human DNA

What is the correct sequence for genetic engineering?

A 1→2→4→3

B 2→3→4→1

C 3→2→4→1

D 4→3→1→2

102
© UCLES 2018 0610/22/O/N/18 [Turn over
18

39 The statements describe some of the events that occur during eutrophication.

What is directly responsible for the increase in bacteria?

A a decrease in dissolved oxygen concentration


B an increase in nitrate concentration
C an increase in the population of algae
D an increase in the death of producers

40 In 1991 there were only fourteen Daviesia cunderdin plants.

What happens to a plant population that becomes very small?

A Each plant will produce fewer offspring.


B Each plant will produce more offspring.
C Variation in the population is increased.
D Variation in the population is reduced.

103
© UCLES 2018 0610/22/O/N/18
Cambridge International Examinations
Cambridge International General Certificate of Secondary Education

BIOLOGY 0610/23
Paper 2 Multiple Choice (Extended) October/November 2018
45 minutes
Additional Materials: Multiple Choice Answer Sheet
Soft clean eraser
*6778270274*

Soft pencil (type B or HB is recommended)

READ THESE INSTRUCTIONS FIRST

Write in soft pencil.


Do not use staples, paper clips, glue or correction fluid.
Write your name, Centre number and candidate number on the Answer Sheet in the spaces provided
unless this has been done for you.
DO NOT WRITE IN ANY BARCODES.

There are forty questions on this paper. Answer all questions. For each question there are four possible
answers A, B, C and D.
Choose the one you consider correct and record your choice in soft pencil on the separate Answer Sheet.

Read the instructions on the Answer Sheet very carefully.

Each correct answer will score one mark. A mark will not be deducted for a wrong answer.
Any rough working should be done in this booklet.
Electronic calculators may be used.

This syllabus is approved for use in England, Wales and Northern Ireland as a Cambridge International Level 1/Level 2 Certificate.

This document consists of 18 printed pages and 2 blank pages.

IB18 11_0610_23/4RP
© UCLES 2018 [Turn104
over
2

1 The Venus flytrap is a plant that feeds on insects. When a fly lands on the leaf, the leaf folds very
quickly and traps the fly. The leaves produce enzymes which digest the fly.

Which characteristics of living organisms are involved?

A excretion, growth, nutrition


B movement, excretion, nutrition
C movement, sensitivity, growth
D movement, sensitivity, nutrition

2 The diagrams show four organisms not drawn to the same scale.

Which organism is a prokaryote?

A B

not to
scale
C D

3 Sunflowers have yellow flowers.

Which cell structure is found in sunflower leaves but not in the petals of the flowers?

A cell membrane
B cell wall
C chloroplast
D vacuole

105
© UCLES 2018 0610/23/O/N/18
3

4 The diagram shows part of a leaf in cross-section.

Structures X and Y are both part of the same

A cell.
B organ.
C tissue.
D vessel.

5 How do carbon dioxide and oxygen move into and out of a mesophyll cell?

A active transport
B diffusion
C respiration
D transpiration

6 Which process is involved in the uptake of glucose by the epithelial cells of kidney tubules?

A active transport
B diffusion
C osmosis
D transpiration

106
© UCLES 2018 0610/23/O/N/18 [Turn over
4

7 The data show the concentrations of sugar and starch in an onion.

total sugar including


starch
reducing sugar
/ g per 100g
/ g per 100g

3.7 0.0

The onion is tested with Benedict’s solution and iodine solution.

Which set of results is correct?

Benedict’s iodine
solution solution

A blue blue-black
B blue brown
C brick red blue-black
D brick red brown

8 Which processes depend on the fact that water is a solvent?

evaporation glucose
movement of water loss of sweat from
from the spongy transported
by osmosis the skin surface
mesophyll cells in blood plasma

A    
B    
C    
D    

107
© UCLES 2018 0610/23/O/N/18
5

9 The apparatus shown is used for an experiment on starch digestion.

Which test-tube contains the most sugar after 20 minutes?

A B C D

starch starch
solution starch solution
and salivary solution and salivary
amylase amylase

water-bath at 15 °C water-bath at 37 °C

10 The graph shows how an enzyme-controlled reaction is affected by temperature.

X
6
5
4
rate
/ arbitrary 3
units
2 Y
1
0
0 10 20 30 40 50
temperature / °C

Which statement explains the change in activity between X and Y?

A There are more effective collisions.


B There is a change in the enzyme shape.
C There is more substrate present.
D The kinetic energy of the molecules has increased.

108
© UCLES 2018 0610/23/O/N/18 [Turn over
6

11 An experiment was carried out using the apparatus shown.

The carbon dioxide content of the water in each test-tube was measured at the start and again
three hours later.

In which test-tube would there be a decrease in carbon dioxide content?

A B C D

black
water polythene
to keep
out light light light light
water
plant

water
snail

12 Which region in the leaf of a green plant contains phloem?

A palisade mesophyll
B spongy mesophyll
C upper epidermis
D vascular bundle

13 What is the result of a diet lacking iron?

A bleeding gums
B poor wound healing
C reduced number of red blood cells
D weak bones and teeth

109
© UCLES 2018 0610/23/O/N/18
7

14 The diagram shows the activity of salivary amylase, pancreatic lipase and stomach protease at
different pH levels.

enzyme
activity

1 2 3 4 5 6 7 8 9 10 11 12
pH

From the graph, what is the optimum pH for the protease enzyme?

A 2.0 B 3.5 C 7.0 D 8.0

15 The diagram shows a section through the stem of a dicotyledonous plant.

Which part transports water and mineral ions?

110
© UCLES 2018 0610/23/O/N/18 [Turn over
8

16 Roots and leaves both act as a source and a sink for sucrose and amino acids at different times
during the year.

At which point in the year are the roots most active as a source?

A
spring
new leaves
start to grow

D B
winter summer
no leaves leaves are
are present mature

C
autumn
leaves die
and fall

17 The diagram shows a circulatory system.

2 3

capillaries of capillaries of the


the lungs heart rest of the body

1 4

Which vessels carry oxygenated blood?

A 1 and 2 B 1 and 4 C 2 and 3 D 2 and 4

18 What happens to the heart valves when the ventricles contract?

atrioventricular semilunar
valves valves

A valves close valves close


B valves close valves open
C valves open valves close
D valves open valves open
111
© UCLES 2018 0610/23/O/N/18
9

19 The diagram with the structure labelled X shows a bacterium with proteins on its surface. The
diagram labelled Y shows proteins made by the human body.

Which row shows the correct combination for destroying the bacterium?

correct
name of X name of Y
shape of Y

A antigen antibody

B antibody antigen

C antigen antibody

D antibody antigen

112
© UCLES 2018 0610/23/O/N/18 [Turn over
10

20 The graph shows changes in the volume of air in the lungs of a person at rest, over a period of 30
seconds.

2
volume
of air in
lungs 1
/ dm3
0
0 30
time / s

Which graph shows changes in the volume of air in the lungs of the same person immediately
after they have done five minutes of vigorous exercise?

A B
2 2
volume volume
of air in of air in
lungs 1 lungs 1
/ dm3 / dm3
0 0
0 30 0 30
time / s time / s

C D
2 2
volume volume
of air in of air in
lungs 1 lungs 1
/ dm3 / dm3
0 0
0 30 0 30
time / s time / s

21 Glucose is required for respiration.

Which other molecule is required for aerobic respiration?

A carbon dioxide
B nitrogen
C oxygen
D water

113
© UCLES 2018 0610/23/O/N/18
11

22 Which row correctly shows the organ where each substance is excreted?

carbon dioxide excess water salts urea

A kidneys liver lungs lungs


B liver liver liver skin
C lungs kidneys kidneys kidneys
D lungs kidneys liver skin

23 The diagram shows the appearance of the iris and pupil in three different light conditions.

1 2 3
iris

pupil

Which row shows the size of the pupil of the eye in each light condition?

no moderate bright
light light light

A 1 2 3
B 1 3 2
C 2 1 3
D 3 2 1

114
© UCLES 2018 0610/23/O/N/18 [Turn over
12

24 A student used two seedlings X and Y to investigate phototropism.

The diagram shows their investigation.

light light
piece of glass piece of glass

X X
at the start of after some time
the experiment no curvature towards light

light light

piece of glass piece of glass

Y Y
at the start of after some time
the experiment gives curvature towards light

Which statement explains the difference in results between X and Y?

A The piece of glass destroyed the auxin on the shaded side of the seedling.
B The piece of glass destroyed the auxin on the side of the seedling facing the light.
C The piece of glass in X stopped the auxin travelling down the shaded side of the seedling.
D The piece of glass in X stopped the auxin travelling down the side of the seedling facing the
light.

25 In a mammal, body temperature is regulated by negative feedback.

As a result of negative feedback, what happens in the mammal’s body as the temperature of the
external environment decreases?

A relaxation of hair erector muscles


B sweating
C vasoconstriction
D vasodilation

115
© UCLES 2018 0610/23/O/N/18
13

26 The graph shows the number of cases of MRSA in one country between 2001 and 2006.

8000

7250
number of
cases of 6500
MRSA
5750

5000
2001 2002 2003 2004 2005 2006
year

Between which years was the greatest change in the number of cases of MRSA seen?

A 2002 and 2003


B 2003 and 2004
C 2004 and 2005
D 2005 and 2006

27 Pollen grains are transferred from the anthers to the stigma. The pollen grains adhere to the
sticky stigma. The statements describe what happens next.

1 The pollen grain grows a pollen tube.


2 The pollen tube enters the ovule.
3 The pollen tube grows down the style.
4 The male nucleus fuses with an egg cell nucleus.

In which order do these stages occur?

A 1→2→4→3

B 1→3→2→4

C 2→3→1→4

D 3→1→4→2

116
© UCLES 2018 0610/23/O/N/18 [Turn over
14

28 The diagram shows a woman’s menstrual cycle.

bleeding bleeding
starts stops

day day
0 28

P QR S

On which day does ovulation occur, and on which day could fertilisation occur?

ovulation fertilisation

A P R
B P S
C Q R
D Q S

29 Which method of contraception could also help prevent the spread of HIV?

A condom
B contraceptive pill
C monitoring cervical mucus
D vasectomy

30 Which name is given to different versions of a gene?

A allele
B chromosome
C length of DNA
D protein

31 Which feature of meiosis ensures that the zygote is diploid?

A Chromosomes are duplicated.


B Four gametes are formed.
C Genetically different gametes are formed.
D Haploid gametes are formed.

117
© UCLES 2018 0610/23/O/N/18
15

32 Which genotypes result in a person having blood group A?

A IAIA and IAIB B IAIA and IAIo C IAIo and IoIo D IAIo and IAIB

33 The diagram shows a plant reproducing asexually by growing a plantlet from a runner. The
leaves of the plantlet appear different to the leaves of the parent plant.

parent plant

runner

plantlet

Which statement explains the difference in the leaf shape of the plantlet?

A A mutation has occurred in the genes of the plantlet.


B The plantlet inherited the genes from the parent plant.
C The plant was produced by meiosis.
D The plantlet was produced by the fusion of gametes.

34 What results from the process of natural selection and adaptation?

A artificial selection
B evolution
C reproduction
D selective breeding

118
© UCLES 2018 0610/23/O/N/18 [Turn over
16

35 The diagram shows a food web in the ocean.

shark

penguin seal

krill herring

algae

Which two animals are secondary and tertiary consumers?

A penguin and herring


B penguin and shark
C seal and penguin
D seal and shark

119
© UCLES 2018 0610/23/O/N/18
17

36 A student was given data on the change in the number of insects in a population over a period of
twelve weeks. The student was asked to name and label each phase.

phase
one phase two phase three
13
12
11
10
9
8
number
7
of insects
/ thousand 6
5
4
3
2
1
0
0 1 2 3 4 5 6 7 8 9 10 11 12
time / weeks

What is the correct label for phase three?

A death phase
B exponential (log) phase
C lag phase
D stationary phase

37 Insulin is now produced using genetically modified bacteria. Previously, diabetics were given
insulin extracted from the pancreas of animals.

Why is the insulin from bacteria regarded as a better ethical choice?

A insulin is produced from bacterial DNA


B it is accepted by vegetarians
C plasmids are involved
D the genetic code is shared

120
© UCLES 2018 0610/23/O/N/18 [Turn over
18

38 Plasmids are often used in genetic engineering.

What is a plasmid?

A bacterial cell
B gene
C loop of DNA
D protein

39 The diagram shows a food web in a woodland.

hawks

hedgehogs blackbirds sparrows hornets

snails caterpillars

tree type X tree type Y

What is most likely to happen if all of tree type X is removed by humans?

A The carbon dioxide concentration in the woodland will decrease.


B The population of hedgehogs will increase.
C The population of hornets will increase.
D The population of sparrows will decrease.

40 The increased availability of nitrates in rivers and lakes can result in the death of fish.

Which statement explains why fish die in rivers and lakes containing a high concentration of
nitrates?

A The increase in water plants stops the fish swimming.


B There is an increase in anaerobic respiration by plants.
C There are fewer water plants for fish to eat.
D There is an increase in aerobic respiration by decomposers.

121
© UCLES 2018 0610/23/O/N/18
Cambridge International Examinations
Cambridge International General Certificate of Secondary Education

BIOLOGY 0610/11
Paper 1 Multiple Choice (Core) October/November 2018
45 minutes
Additional Materials: Multiple Choice Answer Sheet
Soft clean eraser
*1629595112*

Soft pencil (type B or HB is recommended)

READ THESE INSTRUCTIONS FIRST

Write in soft pencil.


Do not use staples, paper clips, glue or correction fluid.
Write your name, Centre number and candidate number on the Answer Sheet in the spaces provided
unless this has been done for you.
DO NOT WRITE IN ANY BARCODES.

There are forty questions on this paper. Answer all questions. For each question there are four possible
answers A, B, C and D.
Choose the one you consider correct and record your choice in soft pencil on the separate Answer Sheet.

Read the instructions on the Answer Sheet very carefully.

Each correct answer will score one mark. A mark will not be deducted for a wrong answer.
Any rough working should be done in this booklet.
Electronic calculators may be used.

This syllabus is approved for use in England, Wales and Northern Ireland as a Cambridge International Level 1/Level 2 Certificate.

This document consists of 18 printed pages and 2 blank pages.

IB18 11_0610_11/3RP
© UCLES 2018 [Turn122
over
2

1 Which characteristic of living organisms is represented by gas exchange in the alveoli?

A excretion
B growth
C nutrition
D reproduction

2 The binomial name for humans is Homo sapiens.

Which row is correct?

Homo sapiens

A genus kingdom
B genus species
C species genus
D species kingdom

3 The diagram shows a stonefly larva.

antenna

thorax gills

abdomen

cerci

Use the key to identify the stonefly larva.

1 has two cerci at the end of the abdomen ............. go to 2


has three cerci at the end of the abdomen ........... go to 3
2 abdomen longer than thorax ................................ A
thorax longer than abdomen ................................ B
3 gills visible on the thorax ...................................... C
gills not visible ...................................................... D

123
© UCLES 2018 0610/11/O/N/18
3

4 Which part of a plant cell controls the movement of substances into and out of the cell?

A cell membrane
B cell wall
C cytoplasm
D vacuole

5 The diagram shows part of a leaf in cross-section.

Structures X and Y are both part of the same

A cell.
B organ.
C tissue.
D vessel.

124
© UCLES 2018 0610/11/O/N/18 [Turn over
4

6 The photograph shows a chloroplast magnified ×7000.

70 mm

What is the actual size of the chloroplast?

A 0.0001 mm B 0.001 mm C 0.01 mm D 100 mm

7 How do carbon dioxide and oxygen move into and out of a mesophyll cell?

A active transport
B diffusion
C respiration
D transpiration

8 What are the features of active transport?

particles move
occurs through uses energy
from a higher to a
a cell membrane from respiration
lower concentration

A   
B   
C   
D   

125
© UCLES 2018 0610/11/O/N/18
5

9 The data show the concentrations of sugar and starch in an onion.

total sugar including


starch
reducing sugar
/ g per 100g
/ g per 100g

3.7 0.0

The onion is tested with Benedict’s solution and iodine solution.

Which set of results is correct?

Benedict’s iodine
solution solution

A blue blue-black
B blue brown
C brick red blue-black
D brick red brown

10 The apparatus shown is used for an experiment on starch digestion.

Which test-tube contains the most sugar after 20 minutes?

A B C D

starch starch
solution starch solution
and salivary solution and salivary
amylase amylase

water-bath at 15 °C water-bath at 37 °C

126
© UCLES 2018 0610/11/O/N/18 [Turn over
6

11 An experiment was carried out using the apparatus shown.

The carbon dioxide content of the water in each test-tube was measured at the start and again
three hours later.

In which test-tube would there be a decrease in carbon dioxide content?

A B C D

black
water polythene
to keep
out light light light light
water
plant

water
snail

12 The diagram shows a cross-section of a leaf.

Which tissue is immediately below the upper epidermis?

A cuticle
B guard cells
C palisade mesophyll
D spongy mesophyll

127
© UCLES 2018 0610/11/O/N/18
7

13 What is the result of a diet lacking iron?

A bleeding gums
B poor wound healing
C reduced number of red blood cells
D weak bones and teeth

14 The diagram shows part of the alimentary canal.

Which structure produces lipase?

A
C

15 The photomicrograph shows a cross-section through a buttercup root.

What is the function of the tissue labelled Z?

A site of photosynthesis
B site of respiration
C transport of sugars
D transport of water

128
© UCLES 2018 0610/11/O/N/18 [Turn over
8

16 The diagram shows two shoots at the start of an experiment on transpiration.

shoot X shoot Y

spring balance

30 g 30 g

oil

water

What are the likely readings on the spring balances after three days?

shoot X / g shoot Y / g

A 25 25
B 25 30
C 30 25
D 30 30

17 The diagram shows a circulatory system.

2 3

capillaries of capillaries of the


the lungs heart rest of the body

1 4

Which vessels carry oxygenated blood?

A 1 and 2 B 1 and 4 C 2 and 3 D 2 and 4

129
© UCLES 2018 0610/11/O/N/18
9

18 Some features that help to defend the body against pathogens are listed.

1 mucus
2 skin
3 stomach acid
4 phagocytosis

Which features can prevent pathogens entering body tissues?

A 1, 2, 3 and 4
B 1, 2 and 3 only
C 2 and 3 only
D 4 only

130
© UCLES 2018 0610/11/O/N/18 [Turn over
10

19 The graph shows changes in the volume of air in the lungs of a person at rest, over a period of 30
seconds.

2
volume
of air in
lungs 1
/ dm3
0
0 30
time / s

Which graph shows changes in the volume of air in the lungs of the same person immediately
after they have done five minutes of vigorous exercise?

A B
2 2
volume volume
of air in of air in
lungs 1 lungs 1
/ dm3 / dm3
0 0
0 30 0 30
time / s time / s

C D
2 2
volume volume
of air in of air in
lungs 1 lungs 1
/ dm3 / dm3
0 0
0 30 0 30
time / s time / s

20 What is produced by anaerobic respiration in mammals?

A alcohol + carbon dioxide


B alcohol + oxygen
C lactic acid + carbon dioxide
D lactic acid

131
© UCLES 2018 0610/11/O/N/18
11

21 Which row describes the functions of the bladder, kidneys and liver?

production excretion storage


of urea of urea of urine

A liver bladder kidneys


B bladder kidneys liver
C liver kidneys bladder
D kidneys liver bladder

22 The diagram shows a reflex arc.

Which label points to the sensory neurone?

A B

hot pan

132
© UCLES 2018 0610/11/O/N/18 [Turn over
12

23 The diagram shows the structure of the eye.

Which structure refracts light?

A
C

24 Which disease can be caused by tobacco smoking?

A cholera
B COPD
C HIV
D scurvy

25 What is formed when the nucleus of a sperm fuses with the nucleus of an egg?

A gamete
B ovule
C stamen
D zygote

133
© UCLES 2018 0610/11/O/N/18
13

26 The diagram shows the female reproductive system.

Where does implantation normally occur?

C
B

27 Which precautions could help to prevent the spread of AIDS?

1 avoiding the mixing of blood


2 using a femidom
3 using the contraceptive pill
4 using a condom

A 1 and 3 B 1, 2 and 4 C 2, 3 and 4 D 2 and 4 only

134
© UCLES 2018 0610/11/O/N/18 [Turn over
14

28 The diagram shows a timeline of a woman’s menstrual cycle, which lasts for 28 days.

13 14 15 16
12 17
11 18
10 19
9 20

8 21

7 22

6 23

5 menstruation 24
(period)
4 25
3 26
2 28 27
1

On which days of the menstrual cycle is a woman most likely to become pregnant?

A days 1– 4
B days 7–10
C days 13–16
D days 20–23

29 The diagram shows what happens during fertilisation.

egg + sperm → fertilised egg → male embryo

Which sex chromosomes are present in the egg, sperm and fertilised egg shown?

fertilised
egg sperm
egg

A X X XX
B X Y XY
C Y X XY
D Y Y YY

135
© UCLES 2018 0610/11/O/N/18
15

30 Many of the varieties of apple in the world evolved from a single wild species, Malus sieversii.

Which processes were involved in creating thousands of genetically different varieties of apple?

fertilisation meiosis mitosis

A   
B   
C   
D   

31 The genetic diagram shows a monohybrid cross. B is the dominant allele and b is the recessive
allele.

parent 1 × 2
genotypes

gametes B b b b

offspring 3 4 5 6
genotypes

Which of the parents and offspring are heterozygous?

A 1, 3 and 4 B 1, 5 and 6 C 2, 3 and 4 D 2, 5 and 6

32 The curve shows the distribution of a human characteristic.

number
of people

human characteristic

Which characteristic is shown by the curve?

A blood groups
B height
C sex
D tongue rolling
136
© UCLES 2018 0610/11/O/N/18 [Turn over
16

33 A farmer grows different varieties of dates, which are a type of fruit.

The table shows some features of the dates he grows.

variety colour texture yield size

Barhee amber soft high small to medium


Dayri dark reddish brown semi dry variable medium to large
Hayany purplish black soft medium large
Maktoom amber soft medium medium to large
Thoory straw coloured dry medium medium to large

The farmer would like to produce a new variety of date using selective breeding.

He wants a medium sized date that is straw coloured and soft, with a high yield.

Which two varieties could the farmer breed together to get the variety he wants?

A Barhee and Dayri


B Barhee and Thoory
C Dayri and Maktoom
D Maktoom and Thoory

34 The diagram shows a woodland food web.

hawk
fox
frog

rabbit snail

grass dandelion

Which statement is correct?

A Dandelions and grass are both primary consumers.


B The fox and the hawk are both secondary consumers.
C The frog is a tertiary consumer.
D The rabbit and the snail are both primary consumers.

137
© UCLES 2018 0610/11/O/N/18
17

35 Which process releases carbon dioxide into the air and which process removes carbon dioxide
from the air?

releases carbon removes carbon


dioxide into the air dioxide from the air

A decay photosynthesis
B decay respiration
C photosynthesis combustion
D photosynthesis decay

36 The graph shows a population growth curve for a species of insect which has been reintroduced
to an island from where it had previously become extinct.

100

90

80

70

60
population size
/ insects per m2 50

40

30

20

10

0
0 50 100 150 200 250 300
time since reintroduction / days

How long after reintroduction does the insect population size start to become limited by resources
such as food?

A 50 days B 100 days C 150 days D 200 days

138
© UCLES 2018 0610/11/O/N/18 [Turn over
18

37 Biotechnology is used to produce ethanol for biofuels.

Which type of organism can be used to produce the ethanol?

A fish
B myriapods
C viruses
D yeast

38 What is an example of genetic engineering?

A using enzymes to make washing powders


B using pectinase to make fruit juice
C producing plants that have been given genes for resistance to insect pests
D using yeast to make bread

39 What is least likely to result from deforestation?

A increase in flooding
B increase in species
C loss of habitats
D loss of soil

40 What could prevent a species from becoming endangered?

A captive breeding programme


B climate change
C introduction of other species
D pollution

139
© UCLES 2018 0610/11/O/N/18
Cambridge International Examinations
Cambridge International General Certificate of Secondary Education

BIOLOGY 0610/12
Paper 1 Multiple Choice (Core) October/November 2018
45 minutes
Additional Materials: Multiple Choice Answer Sheet
Soft clean eraser
*2954426877*

Soft pencil (type B or HB is recommended)

READ THESE INSTRUCTIONS FIRST

Write in soft pencil.


Do not use staples, paper clips, glue or correction fluid.
Write your name, Centre number and candidate number on the Answer Sheet in the spaces provided
unless this has been done for you.
DO NOT WRITE IN ANY BARCODES.

There are forty questions on this paper. Answer all questions. For each question there are four possible
answers A, B, C and D.
Choose the one you consider correct and record your choice in soft pencil on the separate Answer Sheet.

Read the instructions on the Answer Sheet very carefully.

Each correct answer will score one mark. A mark will not be deducted for a wrong answer.
Any rough working should be done in this booklet.
Electronic calculators may be used.

This syllabus is approved for use in England, Wales and Northern Ireland as a Cambridge International Level 1/Level 2 Certificate.

This document consists of 17 printed pages and 3 blank pages.

IB18 11_0610_12/4RP
© UCLES 2018 [Turn140
over
2

1 All living organisms release energy from nutrient molecules within their cells.

What is the name of this characteristic?

A growth
B nutrition
C respiration
D sensitivity

2 The photographs show two different rats.

Rattus norvegicus Rattus rattus

Which statement about the rats is correct?

A The rats are the same genus.


B The rats are the same species.
C The rats can breed together to produce fertile offspring.
D The rats do not share any of the same features.

3 The diagram shows an arthropod.

Which group of arthropods does it belong to?

A arachnids
B crustaceans
C insects
D myriapods
141
© UCLES 2018 0610/12/O/N/18
3

4 Two types of cell, one animal and one plant, were examined using a light microscope.

Which row shows the correct combination of cellular features that would be observed in the cells?

cell structure observed


animal cell plant cell

A chloroplast membrane vacuole cytoplasm


B cytoplasm nucleus chloroplast membrane
C membrane cell wall cytoplasm nucleus
D nucleus chloroplast cell wall membrane

5 The diagram shows part of a leaf in cross-section.

Structures X and Y are both part of the same

A cell.
B organ.
C tissue.
D vessel.

6 A photograph shows a plant cell nucleus measuring 2 mm across.

If the magnification of the cell is ×500, what is the actual size of the nucleus?

A 0.00002 mm B 0.004 mm C 0.04 mm D 250 mm

142
© UCLES 2018 0610/12/O/N/18 [Turn over
4

7 How do carbon dioxide and oxygen move into and out of a mesophyll cell?

A active transport
B diffusion
C respiration
D transpiration

8 The diagram shows apparatus used to investigate osmosis.

The volumes of solutions A, B, C and D were the same at the start of the investigation.

After one hour, the solutions had moved up the glass tubes.

Which solution was the most concentrated at the start of the investigation?

glass glass glass glass


tube tube tube tube

partially partially
permeable permeable
membrane membrane

water

solution solution solution solution


A B C D

143
© UCLES 2018 0610/12/O/N/18
5

9 The data show the concentrations of sugar and starch in an onion.

total sugar including


starch
reducing sugar
/ g per 100g
/ g per 100g

3.7 0.0

The onion is tested with Benedict’s solution and iodine solution.

Which set of results is correct?

Benedict’s iodine
solution solution

A blue blue-black
B blue brown
C brick red blue-black
D brick red brown

10 The apparatus shown is used for an experiment on starch digestion.

Which test-tube contains the most sugar after 20 minutes?

A B C D

starch starch
solution starch solution
and salivary solution and salivary
amylase amylase

water-bath at 15 °C water-bath at 37 °C

144
© UCLES 2018 0610/12/O/N/18 [Turn over
6

11 An experiment was carried out using the apparatus shown.

The carbon dioxide content of the water in each test-tube was measured at the start and again
three hours later.

In which test-tube would there be a decrease in carbon dioxide content?

A B C D

black
water polythene
to keep
out light light light light
water
plant

water
snail

12 The diagram shows a cross-section of a leaf.

Which is the xylem?

13 What is the result of a diet lacking iron?

A bleeding gums
B poor wound healing
C reduced number of red blood cells
D weak bones and teeth

145
© UCLES 2018 0610/12/O/N/18
7

14 Which row shows an enzyme with the correct site of production and products?

enzyme product(s)
enzyme
produced by of digestion

A amylase salivary glands amino acids


B amylase stomach sugar
C protease salivary glands sugar
D protease stomach amino acids

15 The diagram shows a plant cell.

What type of plant cell is this?

A guard cell
B mesophyll cell
C root cortex cell
D root hair cell

146
© UCLES 2018 0610/12/O/N/18 [Turn over
8

16 The diagram shows two shoots at the start of an experiment on transpiration.

shoot X shoot Y

spring balance

30 g 30 g

oil

water

What are the likely readings on the spring balances after three days?

shoot X / g shoot Y / g

A 25 25
B 25 30
C 30 25
D 30 30

17 The diagram shows a circulatory system.

2 3

capillaries of capillaries of the


the lungs heart rest of the body

1 4

Which vessels carry oxygenated blood?

A 1 and 2 B 1 and 4 C 2 and 3 D 2 and 4

147
© UCLES 2018 0610/12/O/N/18
9

18 Some organisms cause transmissible diseases.

What is defined as a disease-causing organism?

A bacterium
B pathogen
C phagocyte
D virus

19 The graph shows changes in the volume of air in the lungs of a person at rest, over a period of 30
seconds.

2
volume
of air in
lungs 1
/ dm3
0
0 30
time / s

Which graph shows changes in the volume of air in the lungs of the same person immediately
after they have done five minutes of vigorous exercise?

A B
2 2
volume volume
of air in of air in
lungs 1 lungs 1
/ dm3 / dm3
0 0
0 30 0 30
time / s time / s

C D
2 2
volume volume
of air in of air in
lungs 1 lungs 1
/ dm3 / dm3
0 0
0 30 0 30
time / s time / s

148
© UCLES 2018 0610/12/O/N/18 [Turn over
10

20 As we breathe out, ......1...... is ......2...... through the lungs.

Which words correctly complete gaps 1 and 2?

1 2

A carbon dioxide excreted


B carbon dioxide respired
C oxygen excreted
D oxygen respired

21 A person carries out vigorous exercise without drinking any water.

What would happen to the concentration and volume of the person’s urine immediately after
exercise?

urine urine
concentration volume

A decrease decrease
B decrease increase
C increase decrease
D increase increase

22 The diagram shows a reflex arc.

Which label points to the sensory neurone?

A B

hot pan

149
© UCLES 2018 0610/12/O/N/18
11

23 What does the central nervous system consist of?

A brain, spinal cord and peripheral nerves


B brain and spinal cord only
C brain only
D spinal cord and peripheral nerves only

24 Where is alcohol broken down in the body?

A bladder
B kidneys
C liver
D stomach

25 The diagram shows parts of a flower.

Which structure represents the site of fertilisation?

150
© UCLES 2018 0610/12/O/N/18 [Turn over
12

26 The diagram shows the female reproductive system.

Where does implantation normally occur?

C
B

27 Which precautions could help to prevent the spread of AIDS?

1 avoiding the mixing of blood


2 using a femidom
3 using the contraceptive pill
4 using a condom

A 1 and 3 B 1, 2 and 4 C 2, 3 and 4 D 2 and 4 only

151
© UCLES 2018 0610/12/O/N/18
13

28 The diagram shows a timeline of a woman’s menstrual cycle, which lasts for 28 days.

13 14 15 16
12 17
11 18
10 19
9 20

8 21

7 22

6 23

5 menstruation 24
(period)
4 25
3 26
2 28 27
1

On which days of the menstrual cycle is a woman most likely to become pregnant?

A days 1– 4
B days 7–10
C days 13–16
D days 20–23

29 What is the transmission of genetic information from generation to generation known as?

A cell division
B inheritance
C meiosis
D mitosis

152
© UCLES 2018 0610/12/O/N/18 [Turn over
14

30 The diagram shows the chromosomes in one human cell.

1 2 3 4 5 6

7 8 9 10 11 12

13 14 15 16 17 18

19 20 21 22 XX

What can be concluded from the chromosomes in this cell?

A The cell is from a man.


B The cell is from a woman.
C There are only 23 chromosomes per cell.
D There are only 46 pairs of chromosomes per cell.

31 The allele for detached earlobes is dominant to the allele for attached earlobes.

Two parents are heterozygous for detached earlobes.

What is the probability of their first child having attached earlobes?

A 0% B 25% C 50% D 75%

32 Which is an example of discontinuous variation?

A height
B skin colour
C tongue rolling
D weight

153
© UCLES 2018 0610/12/O/N/18
15

33 Farmers have bred Holstein-Friesian cattle to produce more milk than older breeds of cattle.

Which process was used to produce these cattle?

A adaptation
B genetic engineering
C natural selection
D selective breeding

34 The diagram shows a food web.

hedgehog
robin

caterpillar
greenfly beetle

rose plant cabbage plant

Which row shows a food chain in this food web?

primary secondary
producer
consumer consumer

A hedgehog caterpillar robin


B cabbage plant greenfly beetle
C cabbage plant beetle robin
D rose plant hedgehog greenfly

154
© UCLES 2018 0610/12/O/N/18 [Turn over
16

35 The diagram shows part of the carbon cycle.

Which arrow represents photosynthesis?

carbon dioxide
in air D

A B C
animals
burning plants

decomposers

36 The population of rabbits in a woodland halves over a ten year period. Rabbits are herbivores.

What could have caused this change?

A an increased food supply


B an increased light intensity
C an increased number of predators
D an increased water supply

37 The production of ethanol for biofuels involves two stages.

Firstly, starch is converted to glucose by ......1...... . Secondly, glucose is converted to ethanol


by ......2...... during ......3...... .

Which row correctly completes gaps 1, 2 and 3?

1 2 3

A enzymes bacteria aerobic respiration


B enzymes yeast anaerobic respiration
C pectinase bacteria aerobic respiration
D pectinase yeast anaerobic respiration

155
© UCLES 2018 0610/12/O/N/18
17

38 Scientists have produced yellow rice called Golden Rice. Golden Rice has been produced by the
insertion of genes into rice plants.

What is involved in the production of Golden Rice?

A genetic engineering
B mutation
C natural selection
D selective breeding

39 What is a negative impact on the environment caused by deforestation?

A decrease in land for livestock production


B decreased levels of carbon dioxide in the air
C increase in flooding
D increased levels of soil

40 Which is a greenhouse gas?

A herbicides
B insecticides
C methane
D oxygen

156
© UCLES 2018 0610/12/O/N/18
Cambridge International Examinations
Cambridge International General Certificate of Secondary Education

BIOLOGY 0610/13
Paper 1 Multiple Choice (Core) October/November 2018
45 minutes
Additional Materials: Multiple Choice Answer Sheet
Soft clean eraser
*9885623081*

Soft pencil (type B or HB is recommended)

READ THESE INSTRUCTIONS FIRST

Write in soft pencil.


Do not use staples, paper clips, glue or correction fluid.
Write your name, Centre number and candidate number on the Answer Sheet in the spaces provided
unless this has been done for you.
DO NOT WRITE IN ANY BARCODES.

There are forty questions on this paper. Answer all questions. For each question there are four possible
answers A, B, C and D.
Choose the one you consider correct and record your choice in soft pencil on the separate Answer Sheet.

Read the instructions on the Answer Sheet very carefully.

Each correct answer will score one mark. A mark will not be deducted for a wrong answer.
Any rough working should be done in this booklet.
Electronic calculators may be used.

This syllabus is approved for use in England, Wales and Northern Ireland as a Cambridge International Level 1/Level 2 Certificate.

This document consists of 17 printed pages and 3 blank pages.

IB18 11_0610_13/3RP
© UCLES 2018 [Turn157
over
2

1 Which process occurs both in plants and in animals?

A excretion
B phagocytosis
C photosynthesis
D transpiration

2 What does the term species mean?

A a group of animals that reproduce asexually


B a group of organisms that can reproduce to produce fertile offspring
C a group of plants that reproduce to produce fertile offspring
D a group of vertebrates that reproduce sexually

3 Woodlice are arthropods with 14 jointed legs.

To which arthropod class do they belong?

A arachnids
B crustaceans
C insects
D myriapods

4 Sunflowers have yellow flowers.

Which cell structure is found in sunflower leaves but not in the petals of the flowers?

A cell membrane
B cell wall
C chloroplast
D vacuole

158
© UCLES 2018 0610/13/O/N/18
3

5 The diagram shows part of a leaf in cross-section.

Structures X and Y are both part of the same

A cell.
B organ.
C tissue.
D vessel.

6 A bacterial cell has a length of 40 mm when it is magnified by ×20 000.

What is the actual length of the bacterial cell?

A 0.0002 mm B 0.002 mm C 0.02 mm D 0.2 mm

7 How do carbon dioxide and oxygen move into and out of a mesophyll cell?

A active transport
B diffusion
C respiration
D transpiration

159
© UCLES 2018 0610/13/O/N/18 [Turn over
4

8 The diagram shows four identical pieces of potato in test-tubes. The potato pieces were left as
shown for six hours.

Which piece of potato would have the greatest increase in mass?

A B C D

distilled water 5% sugar 20% sugar air


solution solution

9 The data show the concentrations of sugar and starch in an onion.

total sugar including


starch
reducing sugar
/ g per 100g
/ g per 100g

3.7 0.0

The onion is tested with Benedict’s solution and iodine solution.

Which set of results is correct?

Benedict’s iodine
solution solution

A blue blue-black
B blue brown
C brick red blue-black
D brick red brown

160
© UCLES 2018 0610/13/O/N/18
5

10 The apparatus shown is used for an experiment on starch digestion.

Which test-tube contains the most sugar after 20 minutes?

A B C D

starch starch
solution starch solution
and salivary solution and salivary
amylase amylase

water-bath at 15 °C water-bath at 37 °C

11 An experiment was carried out using the apparatus shown.

The carbon dioxide content of the water in each test-tube was measured at the start and again
three hours later.

In which test-tube would there be a decrease in carbon dioxide content?

A B C D

black
water polythene
to keep
out light light light light
water
plant

water
snail

161
© UCLES 2018 0610/13/O/N/18 [Turn over
6

12 The diagram shows a cross-section of a leaf.

Which row shows the correct labels?

palisade spongy
stoma
mesophyll cell mesophyll cell

A X Y Z
B X Z Y
C Y X Z
D Y Z X

13 What is the result of a diet lacking iron?

A bleeding gums
B poor wound healing
C reduced number of red blood cells
D weak bones and teeth

162
© UCLES 2018 0610/13/O/N/18
7

14 The diagram shows the activity of salivary amylase, pancreatic lipase and stomach protease at
different pH levels.

enzyme
activity

1 2 3 4 5 6 7 8 9 10 11 12
pH

From the graph, what is the optimum pH for the protease enzyme?

A 2.0 B 3.5 C 7.0 D 8.0

15 The diagram shows a section through the stem of a dicotyledonous plant.

Which part transports water and mineral ions?

163
© UCLES 2018 0610/13/O/N/18 [Turn over
8

16 The diagram shows two shoots at the start of an experiment on transpiration.

shoot X shoot Y

spring balance

30 g 30 g

oil

water

What are the likely readings on the spring balances after three days?

shoot X / g shoot Y / g

A 25 25
B 25 30
C 30 25
D 30 30

17 The diagram shows a circulatory system.

2 3

capillaries of capillaries of the


the lungs heart rest of the body

1 4

Which vessels carry oxygenated blood?

A 1 and 2 B 1 and 4 C 2 and 3 D 2 and 4

164
© UCLES 2018 0610/13/O/N/18
9

18 The body’s first line of defence can prevent some pathogens from entering the body.

Which is a first line of defence?

A antibody production
B mucus
C phagocytosis
D vaccination

19 The graph shows changes in the volume of air in the lungs of a person at rest, over a period of 30
seconds.

2
volume
of air in
lungs 1
/ dm3
0
0 30
time / s

Which graph shows changes in the volume of air in the lungs of the same person immediately
after they have done five minutes of vigorous exercise?

A B
2 2
volume volume
of air in of air in
lungs 1 lungs 1
/ dm3 / dm3
0 0
0 30 0 30
time / s time / s

C D
2 2
volume volume
of air in of air in
lungs 1 lungs 1
/ dm3 / dm3
0 0
0 30 0 30
time / s time / s

165
© UCLES 2018 0610/13/O/N/18 [Turn over
10

20 Glucose is required for respiration.

Which other molecule is required for aerobic respiration?

A carbon dioxide
B nitrogen
C oxygen
D water

21 Which row correctly shows the organ where each substance is excreted?

carbon dioxide excess water salts urea

A kidneys liver lungs lungs


B liver liver liver skin
C lungs kidneys kidneys kidneys
D lungs kidneys liver skin

22 The diagram shows a reflex arc.

Which label points to the sensory neurone?

A B

hot pan

166
© UCLES 2018 0610/13/O/N/18
11

23 The diagram shows the appearance of the iris and pupil in three different light conditions.

1 2 3
iris

pupil

Which row shows the size of the pupil of the eye in each light condition?

no moderate bright
light light light

A 1 2 3
B 1 3 2
C 2 1 3
D 3 2 1

24 Some possible effects of injecting substances are listed.

1 It can cause addiction.


2 It can provide immunity to a disease.
3 It can cause HIV infection if needles are shared.
4 It can improve reaction times.

Which effects are associated with injecting heroin?

A 1 and 3 B 1 and 4 C 2 and 3 D 2 and 4

25 When asexual reproduction takes place, there will always be

A fertilisation.
B offspring genetically different from parent.
C only one parent required.
D production of a pollen tube.

167
© UCLES 2018 0610/13/O/N/18 [Turn over
12

26 The diagram shows the female reproductive system.

Where does implantation normally occur?

C
B

27 Which precautions could help to prevent the spread of AIDS?

1 avoiding the mixing of blood


2 using a femidom
3 using the contraceptive pill
4 using a condom

A 1 and 3 B 1, 2 and 4 C 2, 3 and 4 D 2 and 4 only

168
© UCLES 2018 0610/13/O/N/18
13

28 The diagram shows a timeline of a woman’s menstrual cycle, which lasts for 28 days.

13 14 15 16
12 17
11 18
10 19
9 20

8 21

7 22

6 23

5 menstruation 24
(period)
4 25
3 26
2 28 27
1

On which days of the menstrual cycle is a woman most likely to become pregnant?

A days 1– 4
B days 7–10
C days 13–16
D days 20–23

29 Which name is given to different versions of a gene?

A allele
B chromosome
C length of DNA
D protein

30 Which cells in the human body are produced by meiosis?

A egg cells
B muscle cells
C nerve cells
D white blood cells

169
© UCLES 2018 0610/13/O/N/18 [Turn over
14

31 Two pea plants with white flowers are crossed. Some of the offspring pea plants have red flowers
and some have white flowers.

Which statement about the parent pea plants is correct?

A Both parents are heterozygous.


B Both parents are homozygous.
C One parent is heterozygous and one parent is homozygous.
D The white allele for flower colour is recessive.

32 The diagram shows a plant reproducing asexually by growing a plantlet from a runner. The
leaves of the plantlet appear different to the leaves of the parent plant.

parent plant

runner

plantlet

Which statement explains the difference in the leaf shape of the plantlet?

A A mutation has occurred in the genes of the plantlet.


B The plantlet inherited the genes from the parent plant.
C The plant was produced by meiosis.
D The plantlet was produced by the fusion of gametes.

33 Part of the process of natural selection is described.

Organisms produce many ......1......, which results in competition for ......2...... . This means that
organisms struggle for ......3...... .

Which row correctly completes gaps 1, 2 and 3?

1 2 3

A offspring survival resources


B offspring resources survival
C resources survival offspring
D resources offspring survival

170
© UCLES 2018 0610/13/O/N/18
15

34 Which organism in the food chain shown uses light energy?

A B C D
green plant chicken rat cobra

35 What is the definition of a food chain?

A a line of different living things


B taking in the Sun’s energy
C the process of one living thing ingesting another living thing
D the transfer of energy from one organism to the next, beginning with a producer

36 Elk are mammals and they are herbivores.

The graph shows the total number of elk in a national park between 1978 and 2008.

700

600

500
total 400
number
of elk 300

200

100

0
19 8
19 0
19 2
19 4
19 6
19 8
19 0
19 2
19 4
19 6
20 8
20 0
20 2
20 4
20 6
08
7
8
8
8
8
8
9
9
9
9
9
0
0
0
0
19

year

What is a possible explanation for the change in the elk population between 1998 and 2000?

A decrease in disease
B decrease in hunting
C increase in food source
D increase in predation

171
© UCLES 2018 0610/13/O/N/18 [Turn over
16

37 Bacteria have the following features.

1 the ability to make complex molecules


2 the ability to reproduce quickly
3 have a cell membrane
4 have cytoplasm

Which features make bacteria useful in biotechnology?

A 1, 2, 3 and 4
B 1, 2 and 3 only
C 1 and 2 only
D 1 only

38 Some washing powders contain enzymes as well as detergent.

In an experiment, three pieces of cloth were stained with the same substance. They were left in
different beakers of detergent, some with added enzyme, at 40 °C for 30 minutes.

detergent cloth
at the start of
the experiment beaker black stain

after 30
minutes
at 40 °C

detergent detergent detergent


only and lipase and protease

Which statement is correct for the results shown by this experiment?

A The black stain contained carbohydrate.


B The black stain contained fat.
C The black stain contained protein.
D The black stain contained water.

172
© UCLES 2018 0610/13/O/N/18
17

39 Which activity is the least likely source of pollution?

direction of
water flow D field sprayed with
fertiliser just before
heavy rainfall
am
C farmer spraying fields re
st
with insecticide
A farmer spraying fields
with herbicide

B sewage treatment works

direction of
flow of stream

40 What is used in the conservation of species?

A deforestation
B moving species to different habitats
C seed banks
D selective breeding

173
© UCLES 2018 0610/13/O/N/18
Cambridge International Examinations
Cambridge International General Certificate of Secondary Education

BIOLOGY 0610/21
Paper 2 Multiple Choice (Extended) May/June 2018
45 minutes
Additional Materials: Multiple Choice Answer Sheet
Soft clean eraser
*0654737364*

Soft pencil (type B or HB is recommended)

READ THESE INSTRUCTIONS FIRST

Write in soft pencil.


Do not use staples, paper clips, glue or correction fluid.
Write your name, Centre number and candidate number on the Answer Sheet in the spaces provided
unless this has been done for you.
DO NOT WRITE IN ANY BARCODES.

There are forty questions on this paper. Answer all questions. For each question there are four possible
answers A, B, C and D.
Choose the one you consider correct and record your choice in soft pencil on the separate Answer Sheet.

Read the instructions on the Answer Sheet very carefully.

Each correct answer will score one mark. A mark will not be deducted for a wrong answer.
Any rough working should be done in this booklet.
Electronic calculators may be used.

This syllabus is approved for use in England, Wales and Northern Ireland as a Cambridge International Level 1/Level 2 Certificate.

This document consists of 15 printed pages and 1 blank page.

IB18 06_0610_21/2RP
© UCLES 2018 [Turn174
over
2

1 Which organisms carry out respiration, growth, movement and excretion?

A all animals and all plants


B animals only
C arthropods and flowering plants only
D plants only

2 The diagram shows an animal whose scientific name is Falco peregrinus.

To which species does it belong?

A bird
B F. peregrinus
C Falco
D vertebrate

3 What kind of skin do amphibians have?

A dry without scales


B dry with scales
C moist without scales
D moist with scales

175
© UCLES 2018 0610/21/M/J/18
3

4 The diagram shows a flowering plant.

Use the key to identify the plant.

plant

flower has four petals flower has five petals

leaves with leaves with leaves with leaves with


smooth edges jagged edges smooth edges jagged edges

A B C D

5 In which part of the cell does aerobic respiration occur?

A cytoplasm
B mitochondrion
C ribosome
D vesicle

6 Why do some root cells have root hairs?

A for the maintenance of the temperature of the cell sap


B to increase the surface area of the cells
C to increase the volume of the cell sap
D to provide a place for cell nuclei

176
© UCLES 2018 0610/21/M/J/18 [Turn over
4

7 Which words correctly complete the paragraph?

Diffusion may be defined as the net movement of particles from a region of their ««1««
concentration, to a region of their ««2«« concentration, where movement is ««3«« a
concentration gradient.

1 2 3

A higher lower down


B higher lower up
C lower higher down
D lower higher up

8 Which part of a plant root hair is partially permeable?

A the cell sap


B the cell surface membrane
C the cell vacuole
D the cell wall

9 The table shows the results of food tests carried out on a fruit.

test Benedict’s biuret ethanol iodine


result positive positive negative negative

What did the fruit contain?

A fat and reducing sugar


B fat and starch
C protein and reducing sugar
D protein and starch

177
© UCLES 2018 0610/21/M/J/18
5

10 An experiment was carried out to investigate the effect of pH on enzyme action. The graph shows
the results.

y-axis

x-axis

What are the labels for the x-axis and the y-axis?

x-axis y-axis

A pH rate of reaction
B pH time
C rate of reaction pH
D time pH

11 The equation for photosynthesis is shown.

......1...... + ......2...... → glucose + ......3......

Which words correctly complete gaps 1, 2 and 3?

1 2 3

A carbon dioxide light oxygen


B carbon dioxide water oxygen
C oxygen light carbon dioxide
D oxygen water carbon dioxide

12 What is the best source of vitamin C in a balanced diet?

A fish
B fruit
C meat
D rice

178
© UCLES 2018 0610/21/M/J/18 [Turn over
6

13 A young, active woman requires more of which constituent in her diet than a young, active man?

A fat
B iron
C protein
D vitamin C

14 Which condition could be caused by a lack of iron?

A anaemia
B cholera
C scurvy
D diabetes

15 The diagram shows part of a section through a plant stem.

Which tissue transports water from the roots to the leaves?

16 Petroleum jelly is waterproof and transparent.

Covering the underside of the leaves of a plant with a thin layer of petroleum jelly will slow down
the rate of water loss from the plant.

Which statement explains this?

A Plants absorb nutrients from the petroleum jelly.


B Plants absorb water from the petroleum jelly.
C Stomata are blocked by the petroleum jelly.
D The petroleum jelly stops photosynthesis.

179
© UCLES 2018 0610/21/M/J/18
7

17 A student places two samples of crushed apple into two beakers, P and Q. The samples are of
equal size. She adds 5 cm3 of pectinase solution to beaker P and 5 cm3 of water to beaker Q.

After five minutes, she places the samples of crushed apple into two different filter funnels, and
measures the volume of juice filtering through from each sample over a period of 10 minutes.

Which graph shows her results?

A B

Q P

total volume total volume


of juice filtered P of juice filtered
through through Q

0 0
0 time 0 time

C D

P
Q

total volume total volume


of juice filtered of juice filtered Q
through P through

0 0
0 time 0 time

18 During the process of blood clotting, damage to blood vessels stimulates L, and M is converted
to N.

What are L, M and N?

L M N

A fibrin platelets fibrinogen


B fibrinogen platelets fibrin
C platelets fibrin fibrinogen
D platelets fibrinogen fibrin

180
© UCLES 2018 0610/21/M/J/18 [Turn over
8

19 A child is vaccinated against measles. After a period of time the child is infected with the measles
virus.

The graph shows the concentration of measles antibodies in the child’s bloodstream during this
time.

antibody
concentration
in the blood

time

Which statement is consistent with the information in the graph?

A After the vaccination, the child produced memory cells.


B The child had passive immunity against measles.
C The measles virus contains antibodies.
D The vaccination failed to protect the child against measles.

20 Muscles are responsible for the ventilation of the lungs during breathing.

Which row describes their action during the inspiration of air?

diaphragm external internal


muscles intercostal muscles intercostal muscles

A contract contract relax


B contract relax contract
C relax contract relax
D relax relax contract

21 Yeast is able to respire both aerobically and anaerobically.

Which statement describes the waste products of yeast respiration?

A Aerobic respiration produces alcohol as one of its waste products.


B Aerobic respiration produces three times as much carbon dioxide as anaerobic respiration
from one molecule of glucose.
C Anaerobic respiration and aerobic respiration both produce the same amount of carbon
dioxide from one molecule of glucose.
D Anaerobic respiration produces three times as much carbon dioxide as aerobic respiration
from one molecule of glucose.
181
© UCLES 2018 0610/21/M/J/18
9

22 The table shows the presence or absence of chemicals in solution in different parts of a healthy
kidney.

Which row is correct?

blood plasma fluid entering


chemical fluid in ureter
in glomerulus kidney tubule

A glucose    key
B protein    = present
C salts    = absent
D urea   

23 Four processes occur when impulses cross a synapse.

P neurotransmitter diffuses across the gap


Q neurotransmitter binds with receptors
R impulse stimulates vesicles
S release of neurotransmitter

What is the correct sequence for these processes?

A P→R→Q→S

B R→P→Q→S

C R→S→P→Q

D S→Q→R→P

24 Which hormone is involved in the conversion of glucose to glycogen?

A adrenaline
B insulin
C oestrogen
D testosterone

182
© UCLES 2018 0610/21/M/J/18 [Turn over
10

25 The diagram shows the structure of human skin.

X
Z

What are X, Y and Z?

X Y Z

A receptor sensory neurone sweat gland


B sensory neurone hair erector muscle receptor
C sweat gland hair erector muscle sensory neurone
D sweat gland receptor blood vessel

26 A seedling was placed in a horizontal position.

Which diagram shows the result of the gravitropic responses in the seedling?

A B

root
root shoot
shoot

C D

shoot

root root shoot

183
© UCLES 2018 0610/21/M/J/18
11

27 Which hormone may be used to improve sporting performance?

A FSH
B LH
C oestrogen
D testosterone

28 Specific grape varieties are maintained using stem cuttings from mature plants that are then
planted and cultivated to produce grapes. This is an example of artificial asexual reproduction.

What is a disadvantage of using asexual reproduction to produce fruit?

A An outbreak of disease will affect the whole crop in the same way.
B Genetically identical fruit is produced relatively quickly.
C No pollination or pollinators are required.
D The characteristics of the grapes will vary between plants.

29 Which two statements are correct for the process of cross-pollination in plants?

1 increases potential for variation in offspring


2 pollen is transferred to a different flower on the same plant
3 reduces potential to respond to environmental change
4 pollen is transferred to a flower on a different plant of the same species

A 1 and 2 B 1 and 4 C 2 and 3 D 3 and 4

30 What is a function of the mitochondria in a sperm cell?

A penetrating the surface of the egg cell


B propelling the sperm towards the egg
C storing food energy
D supplying the energy for movement

184
© UCLES 2018 0610/21/M/J/18 [Turn over
12

31 The diagram shows the chromosomes in the nucleus of a body cell in an adult fruit fly.

What are the diploid and haploid number of chromosomes in the fruit fly?

diploid haploid

A 4 8
B 8 4
C 8 16
D 16 8

32 The diagram shows the phenotypes for blood groups in a family.

parents A B key
male

female
children A AB O

Which statement about the genotypes of the parents is correct?

A Both parents have alleles for blood group A and B.


B Both parents have the allele for blood group O.
C Only the father has the allele for blood group O.
D Only the mother has the allele for blood group O.

33 Sickle-cell anaemia is a genetic disorder which results in severe illness in homozygous


individuals. In some human populations being heterozygous can be beneficial.

What could be the reason for this?

A Heterozygous individuals are not affected by the disorder.


B Heterozygous individuals are more resistant to malaria.
C It is caused by a dominant allele.
D The disorder is sex-linked.

185
© UCLES 2018 0610/21/M/J/18
13

34 Which statement describes the relationship between evolution and natural selection?

A A change in the adaptive features of a population over time causes evolution, resulting in
natural selection.
B Evolution causes a change in the adaptive features of a population over time, resulting in
natural selection.
C Evolution causes natural selection, resulting in a change in the adaptive features of a
population over time.
D Evolution is the change in the adaptive features of a population over time as a result of
natural selection.

35 The diagram shows a food chain.

maize → locusts → lizards → snakes


10 000 kJ 1000 kJ 100 kJ 10 kJ

What is the efficiency of energy transfer between the maize and the lizards in this food chain?

A 0.01% B 0.1% C 1% D 10%

36 The diagram shows part of a food web from a rainforest.

jaguars

anacondas harpy eagles

iguanas howler monkeys

fruits
and leaves

In this food web, at which trophic level are the anacondas?

A primary consumers
B secondary consumers
C tertiary consumers
D quaternary consumers

186
© UCLES 2018 0610/21/M/J/18 [Turn over
14

37 The diagram shows the structure of a bacterial cell.

DNA
X

The presence of structure X in the bacterial cell is one reason why bacteria are used in genetic
engineering.

What is structure X?

A endoplasmic reticulum
B mitochondria
C plasmid
D ribosome

38 What does Penicillium need to grow in a fermenter?

amino acids carbohydrates oxygen

A   
B   
C   
D   

39 Deforestation can have a negative impact on the environment.

Which statement about the negative impact of deforestation is correct?

A decreases the levels of carbon dioxide which can lead to reduced rates of photosynthesis
B decreases the amount of water flowing in local rivers so there is less flooding
C leads to soil loss so there is less fertile soil for the growth of crops in the area
D provides less land for the extraction of natural resources

187
© UCLES 2018 0610/21/M/J/18
15

40 When nitrates enter a lake they cause rapid growth of algae on the surface of the water. This
causes the following changes in the lake:

1 a decrease in the concentration of dissolved oxygen in the water


2 fish and other aquatic animals die
3 an increase in aerobic respiration by decomposers
4 producers die and decomposition increases

In which order do these changes occur?

A 1→2→4→3

B 3→1→2→4

C 4→2→3→1

D 4→3→1→2

188
© UCLES 2018 0610/21/M/J/18
Cambridge International Examinations
Cambridge International General Certificate of Secondary Education

BIOLOGY 0610/22
Paper 2 Multiple Choice (Extended) May/June 2018
45 minutes
Additional Materials: Multiple Choice Answer Sheet
Soft clean eraser
*1751718756*

Soft pencil (type B or HB is recommended)

READ THESE INSTRUCTIONS FIRST

Write in soft pencil.


Do not use staples, paper clips, glue or correction fluid.
Write your name, Centre number and candidate number on the Answer Sheet in the spaces provided
unless this has been done for you.
DO NOT WRITE IN ANY BARCODES.

There are forty questions on this paper. Answer all questions. For each question there are four possible
answers A, B, C and D.
Choose the one you consider correct and record your choice in soft pencil on the separate Answer Sheet.

Read the instructions on the Answer Sheet very carefully.

Each correct answer will score one mark. A mark will not be deducted for a wrong answer.
Any rough working should be done in this booklet.
Electronic calculators may be used.

This syllabus is approved for use in England, Wales and Northern Ireland as a Cambridge International Level 1/Level 2 Certificate.

This document consists of 16 printed pages.

IB18 06_0610_22/2RP
© UCLES 2018 [Turn189
over
2

1 Which organisms carry out respiration, growth, movement and excretion?

A all animals and all plants


B animals only
C arthropods and flowering plants only
D plants only

2 The diagram shows an animal whose scientific name is Falco peregrinus.

To which species does it belong?

A bird
B F. peregrinus
C Falco
D vertebrate

3 What kind of skin do amphibians have?

A dry without scales


B dry with scales
C moist without scales
D moist with scales

190
© UCLES 2018 0610/22/M/J/18
3

4 The diagram shows a flowering plant.

Use the key to identify the plant.

plant

flower has four petals flower has five petals

leaves with leaves with leaves with leaves with


smooth edges jagged edges smooth edges jagged edges

A B C D

5 In which part of the cell does aerobic respiration occur?

A cytoplasm
B mitochondrion
C ribosome
D vesicle

6 Why do some root cells have root hairs?

A for the maintenance of the temperature of the cell sap


B to increase the surface area of the cells
C to increase the volume of the cell sap
D to provide a place for cell nuclei

191
© UCLES 2018 0610/22/M/J/18 [Turn over
4

7 The table shows the concentration of gases in a blood vessel and in an alveolus.

Which row shows the conditions that cause a gas produced in respiration to diffuse from the
blood vessel into the alveolus?

concentration in concentration in
gas produced
the blood vessel the alveolus

A carbon dioxide low high


B carbon dioxide high low
C oxygen low high
D oxygen high low

8 Which part of a plant root hair is partially permeable?

A the cell sap


B the cell surface membrane
C the cell vacuole
D the cell wall

9 The table shows the results of food tests carried out on a fruit.

test Benedict’s biuret ethanol iodine


result positive positive negative negative

What did the fruit contain?

A fat and reducing sugar


B fat and starch
C protein and reducing sugar
D protein and starch

192
© UCLES 2018 0610/22/M/J/18
5

10 An experiment was carried out to investigate the effect of pH on enzyme action. The graph shows
the results.

y-axis

x-axis

What are the labels for the x-axis and the y-axis?

x-axis y-axis

A pH rate of reaction
B pH time
C rate of reaction pH
D time pH

193
© UCLES 2018 0610/22/M/J/18 [Turn over
6

11 An experiment is set up as shown.

small plant

gauze platform

green
indicator

test-tube X test-tube Y

The green indicator turns yellow when the concentration of carbon dioxide increases. The green
indicator turns blue when the concentration of carbon dioxide decreases.

After several hours, the indicator in test-tube X turned blue. The indicator in test-tube Y remained
green.

Which process caused the colour change?

A germination
B photosynthesis
C respiration
D transpiration

12 Why do plants need nitrate ions?

A for making amino acids


B for making fatty acids
C for making glucose
D for making starch

13 In which part of the alimentary canal do chemical digestion and mechanical digestion take place?

A colon
B duodenum
C mouth
D oesophagus

194
© UCLES 2018 0610/22/M/J/18
7

14 The diagram shows part of the alimentary canal.

Where is most water absorbed?

B
A

15 A celery stalk was placed in a beaker which contained a red stain. After 24 hours, the red stain
appeared at the top of the celery stalk.

Which structures stained red?

A cortex cells
B mesophyll cells
C phloem
D xylem

16 A student is investigating the effect of temperature on the rate of transpiration.

Which environmental conditions should be kept constant during this investigation?

humidity light intensity temperature wind speed

A    
B    
C    
D    
195
© UCLES 2018 0610/22/M/J/18 [Turn over
8

17 A student places two samples of crushed apple into two beakers, P and Q. The samples are of
equal size. She adds 5 cm3 of pectinase solution to beaker P and 5 cm3 of water to beaker Q.

After five minutes, she places the samples of crushed apple into two different filter funnels, and
measures the volume of juice filtering through from each sample over a period of 10 minutes.

Which graph shows her results?

A B

Q P

total volume total volume


of juice filtered P of juice filtered
through through Q

0 0
0 time 0 time

C D

P
Q

total volume total volume


of juice filtered of juice filtered Q
through P through

0 0
0 time 0 time

18 During the process of blood clotting, damage to blood vessels stimulates L, and M is converted
to N.

What are L, M and N?

L M N

A fibrin platelets fibrinogen


B fibrinogen platelets fibrin
C platelets fibrin fibrinogen
D platelets fibrinogen fibrin

196
© UCLES 2018 0610/22/M/J/18
9

19 A child is vaccinated against measles. After a period of time the child is infected with the measles
virus.

The graph shows the concentration of measles antibodies in the child’s bloodstream during this
time.

antibody
concentration
in the blood

time

Which statement is consistent with the information in the graph?

A After the vaccination, the child produced memory cells.


B The child had passive immunity against measles.
C The measles virus contains antibodies.
D The vaccination failed to protect the child against measles.

20 Muscles are responsible for the ventilation of the lungs during breathing.

Which row describes their action during the inspiration of air?

diaphragm external internal


muscles intercostal muscles intercostal muscles

A contract contract relax


B contract relax contract
C relax contract relax
D relax relax contract

197
© UCLES 2018 0610/22/M/J/18 [Turn over
10

21 Aerobic respiration involves the break down of glucose.

C6H12O6 + ...x...O2 → ...y...H2O + ...z...CO2 + energy

Which values for x, y and z balance the equation?

x y z

A 6 4 6
B 6 6 6
C 6 12 6
D 12 12 12

22 The composition of the blood in the renal vein is different to the composition of the blood in the
renal artery.

What substance has a higher concentration in the renal vein than in the renal artery?

A carbon dioxide
B glucose
C oxygen
D urea

23 The diagram shows the structures in a reflex arc.

receptor

muscle

What is X?

A effector
B relay neurone
C sensory neurone
D synapse

198
© UCLES 2018 0610/22/M/J/18
11

24 Which row describes accommodation when viewing a near object?

suspensory
ciliary muscles lens shape
ligaments

A contracted slackened more spherical


B contracted tight more spherical
C relaxed slackened less spherical
D relaxed tight more spherical

25 Hormones and the nervous system both control our bodies.

Which statement about the control provided by our hormones is correct?

A fast response and long lasting


B fast response and short lived
C slow response and long lasting
D slow response and short lived

26 Which row describes heroin?

type of drug location of effect

A depressant blood
B depressant synapses
C stimulant blood
D stimulant synapses

27 What is an advantage, for an organism, of using asexual reproduction?

A Disease spreads rapidly.


B It does not cause variation.
C It easily adapts to environmental change.
D Population increases rapidly.

28 The diploid number in Ovis aries (sheep) is 54.

How many chromosomes will there be in a zygote of this species?

A 0 B 27 C 54 D 108

199
© UCLES 2018 0610/22/M/J/18 [Turn over
12

29 Which adaptive feature of sperm provides the energy that enables it to swim?

A the acrosome
B the cell membrane
C the flagellum
D the mitochondria

30 Two hormones, X and Y, are secreted by the ovaries. The graph shows the concentrations of
these hormones in the blood during one complete menstrual cycle.

X Y

concentration
of hormone
in the blood

0 7 14 21 28
time / days

What are hormones X and Y?

hormone X hormone Y

A FSH LH
B LH FSH
C oestrogen progesterone
D progesterone oestrogen

200
© UCLES 2018 0610/22/M/J/18
13

31 The mitotic index of a tissue is the percentage of cells in a tissue that are undergoing division.

The table shows the number of cells in two tissue samples, X and Y.

number of cells number of cells total number of


tissue
undergoing not undergoing cells in the
sample
mitosis mitosis tissue sample

X 25 55 80
Y 15 95 110

Which statement is correct?

A Tissue X is growing faster with a mitotic index of 31.3%.


B Tissue X is growing faster with a mitotic index of 45.4%.
C Tissue Y is growing faster with a mitotic index of 13.6%.
D Tissue Y is growing faster with a mitotic index of 15.8%.

32 Some features of cell division are listed.

1 haploid cells are produced


2 new cells are genetically identical
3 reduction division
4 results in variation

Which features would be associated with meiosis?

A 1, 2 and 3 B 1, 2 and 4 C 1, 3 and 4 D 2, 3 and 4

33 Sickle-cell anaemia is an inherited disease.

parental 1 × 2
genotypes

gametes HbS HbA HbA HbA

offspring
genotypes 3 4 5 6

Which genotypes represented in the diagram would increase an individual’s resistance to


malaria?

A 1, 3 and 4 B 1, 4 and 5 C 2, 3 and 4 D 2, 5 and 6

201
© UCLES 2018 0610/22/M/J/18 [Turn over
14

34 Fitness is defined as the probability of an organism ««1«« and ««2«« in the environment
in which it is found.

Which words correctly complete gaps 1 and 2?

1 2

A photosynthesising respiring
B reproducing respiring
C surviving reproducing
D surviving photosynthesising

35 A food chain has the pyramid of numbers shown.

carnivorous insects

herbivorous insects

producer

What is the pyramid of biomass for the same food chain?

A B

C D

202
© UCLES 2018 0610/22/M/J/18
15

36 The graph shows the changes in a population of brine shrimp over a period of time.

1 2 3

number of
brine shrimp

0
0 time

Over which section or sections is the birth rate greater than the death rate?

A 1, 2 and 3 B 1 and 2 only C 1 only D 2 only

37 What is a disadvantage of using bacteria to produce human insulin?

A few ethical concerns


B genetic code shared with other organisms
C they contain plasmids
D they mutate frequently

38 The diagram shows a bacterial cell containing a plasmid.

plasmid

What is inserted into the plasmid if this cell is to be used for the production of insulin?

A a length of DNA from a human


B a length of DNA from another bacterium
C a molecule of insulin
D an enzyme

39 What would be unlikely to increase the risk of human famine?

A a rapidly increasing human population


B flooding of land
C lack of rain causing a drought
D the use of pesticides on crops

203
© UCLES 2018 0610/22/M/J/18 [Turn over
16

40 Since 1970 sulfur dioxide emissions have dropped by 95% in some parts of the world.

What is the main reason for the reduction in emissions?

A fewer coal-powered power stations


B increase in wind turbines
C less cars on the road
D less industry

Permission to reproduce items where third-party owned material protected by copyright is included has been sought and cleared where possible. Every
reasonable effort has been made by the publisher (UCLES) to trace copyright holders, but if any items requiring clearance have unwittingly been included, the
publisher will be pleased to make amends at the earliest possible opportunity.

To avoid the issue of disclosure of answer-related information to candidates, all copyright acknowledgements are reproduced online in the Cambridge
International Examinations Copyright Acknowledgements Booklet. This is produced for each series of examinations and is freely available to download at
www.cie.org.uk after the live examination series.

204
Cambridge International Examinations is part of the Cambridge Assessment Group. Cambridge Assessment is the brand name of University of Cambridge Local
Examinations Syndicate (UCLES), which is itself a department of the University of Cambridge.

© UCLES 2018 0610/22/M/J/18


Cambridge International Examinations
Cambridge International General Certificate of Secondary Education

BIOLOGY 0610/23
Paper 2 Multiple Choice (Extended) May/June 2018
45 minutes
Additional Materials: Multiple Choice Answer Sheet
Soft clean eraser
*7539978956*

Soft pencil (type B or HB is recommended)

READ THESE INSTRUCTIONS FIRST

Write in soft pencil.


Do not use staples, paper clips, glue or correction fluid.
Write your name, Centre number and candidate number on the Answer Sheet in the spaces provided
unless this has been done for you.
DO NOT WRITE IN ANY BARCODES.

There are forty questions on this paper. Answer all questions. For each question there are four possible
answers A, B, C and D.
Choose the one you consider correct and record your choice in soft pencil on the separate Answer Sheet.

Read the instructions on the Answer Sheet very carefully.

Each correct answer will score one mark. A mark will not be deducted for a wrong answer.
Any rough working should be done in this booklet.
Electronic calculators may be used.

This syllabus is approved for use in England, Wales and Northern Ireland as a Cambridge International Level 1/Level 2 Certificate.

This document consists of 15 printed pages and 1 blank page.

IB18 06_0610_23/3RP
© UCLES 2018 [Turn205
over
2

1 Which organisms carry out respiration, growth, movement and excretion?

A all animals and all plants


B animals only
C arthropods and flowering plants only
D plants only

2 The diagram shows an animal whose scientific name is Falco peregrinus.

To which species does it belong?

A bird
B F. peregrinus
C Falco
D vertebrate

3 What kind of skin do amphibians have?

A dry without scales


B dry with scales
C moist without scales
D moist with scales

206
© UCLES 2018 0610/23/M/J/18
3

4 The diagram shows a flowering plant.

Use the key to identify the plant.

plant

flower has four petals flower has five petals

leaves with leaves with leaves with leaves with


smooth edges jagged edges smooth edges jagged edges

A B C D

5 In which part of the cell does aerobic respiration occur?

A cytoplasm
B mitochondrion
C ribosome
D vesicle

6 Why do some root cells have root hairs?

A for the maintenance of the temperature of the cell sap


B to increase the surface area of the cells
C to increase the volume of the cell sap
D to provide a place for cell nuclei

207
© UCLES 2018 0610/23/M/J/18 [Turn over
4

7 The diagram shows a section through an alveolus and through a capillary.

wall of alveolus
air

capillary red
wall blood cells

capillary

low carbon dioxide Y


concentration

high carbon dioxide


concentration

How does carbon dioxide move from Y to X?

A by diffusion
B by osmosis
C by translocation
D by transpiration

8 Which part of a plant root hair is partially permeable?

A the cell sap


B the cell surface membrane
C the cell vacuole
D the cell wall

208
© UCLES 2018 0610/23/M/J/18
5

9 The table shows the results of food tests carried out on a fruit.

test Benedict’s biuret ethanol iodine


result positive positive negative negative

What did the fruit contain?

A fat and reducing sugar


B fat and starch
C protein and reducing sugar
D protein and starch

10 An experiment was carried out to investigate the effect of pH on enzyme action. The graph shows
the results.

y-axis

x-axis

What are the labels for the x-axis and the y-axis?

x-axis y-axis

A pH rate of reaction
B pH time
C rate of reaction pH
D time pH

209
© UCLES 2018 0610/23/M/J/18 [Turn over
6

11 The diagram shows a section through a leaf.

S R

What are structures P, Q, R and S?

P Q R S

A chloroplast phloem palisade tissue xylem


B cuticle xylem chloroplast stoma
C phloem palisade tissue cuticle stoma
D xylem chloroplast phloem cuticle

12 A lack of which dietary component can result in constipation?

A fat
B fibre (roughage)
C carbohydrate
D protein

13 A woman requires more calcium in her diet when she is pregnant.

Which statement explains why?

A bone growth in the fetus


B muscle growth in the fetus
C production of haemoglobin in the fetus
D to provide energy for the fetus

210
© UCLES 2018 0610/23/M/J/18
7

14 The diagram shows part of the alimentary canal.

M
K

Which row correctly identifies the structures labelled J to M?

J K L M

A oesophagus pancreas small intestine stomach


B pancreas small intestine stomach oesophagus
C small intestine stomach oesophagus pancreas
D stomach oesophagus pancreas small intestine

15 The diagram shows the pathway of water from the soil through a plant.

soil → X → Y → Z → mesophyll cells

Which cells are represented by X, Y and Z?

X Y Z

A root hair cells root cortex cells xylem


B root hair cells xylem root cortex cells
C xylem root cortex cells root hair cells
D xylem root hair cells root cortex cells

16 What will not affect the rate of transpiration?

A humidity of the atmosphere


B number of open stomata
C rate of respiration
D temperature

211
© UCLES 2018 0610/23/M/J/18 [Turn over
8

17 A student places two samples of crushed apple into two beakers, P and Q. The samples are of
equal size. She adds 5 cm3 of pectinase solution to beaker P and 5 cm3 of water to beaker Q.

After five minutes, she places the samples of crushed apple into two different filter funnels, and
measures the volume of juice filtering through from each sample over a period of 10 minutes.

Which graph shows her results?

A B

Q P

total volume total volume


of juice filtered P of juice filtered
through through Q

0 0
0 time 0 time

C D

P
Q

total volume total volume


of juice filtered of juice filtered Q
through P through

0 0
0 time 0 time

18 During the process of blood clotting, damage to blood vessels stimulates L, and M is converted
to N.

What are L, M and N?

L M N

A fibrin platelets fibrinogen


B fibrinogen platelets fibrin
C platelets fibrin fibrinogen
D platelets fibrinogen fibrin

212
© UCLES 2018 0610/23/M/J/18
9

19 A child is vaccinated against measles. After a period of time the child is infected with the measles
virus.

The graph shows the concentration of measles antibodies in the child’s bloodstream during this
time.

antibody
concentration
in the blood

time

Which statement is consistent with the information in the graph?

A After the vaccination, the child produced memory cells.


B The child had passive immunity against measles.
C The measles virus contains antibodies.
D The vaccination failed to protect the child against measles.

20 Muscles are responsible for the ventilation of the lungs during breathing.

Which row describes their action during the inspiration of air?

diaphragm external internal


muscles intercostal muscles intercostal muscles

A contract contract relax


B contract relax contract
C relax contract relax
D relax relax contract

213
© UCLES 2018 0610/23/M/J/18 [Turn over
10

21 A student investigated aerobic respiration.

gauze drop of liquid

soda lime

woodlice capillary tube

Soda lime absorbs carbon dioxide.

If the woodlice are respiring aerobically, what will happen to the drop of liquid?

A It will move towards the woodlice as oxygen is used up and carbon dioxide is released.
B It will move away from the woodlice as oxygen is used up and carbon dioxide is released.
C It will not move as carbon dioxide is used up and oxygen is released.
D It will not move as oxygen is used up and carbon dioxide is released.

22 The diagram shows a kidney and associated structures.

blood in

blood out

What is the name of structure X?

A pulmonary artery
B pulmonary vein
C renal artery
D renal vein

23 What is the correct sequence in a reflex action?

A receptor → stimulus → motor neurone → relay neurone → sensory neurone → effector

B receptor → stimulus → sensory neurone → relay neurone → motor neurone → effector

C stimulus → receptor → motor neurone → relay neurone → sensory neurone → effector

D stimulus → receptor → sensory neurone → relay neurone → motor neurone → effector

214
© UCLES 2018 0610/23/M/J/18
11

24 The diagram shows the cross-section of an eye.

Which statement is true when a person views a near object?

A E contracts; G is pulled less; F bulges less; more refraction of light takes place
B E contracts; G is pulled less; F bulges more; more refraction of light takes place
C E relaxes; G is pulled less; F bulges less; less refraction of light takes place
D E relaxes; G is pulled more; F bulges more; less refraction of light takes place

25 Compared to the hormonal system, the responses of the nervous system are

A faster and of longer-duration.


B faster and of shorter-duration.
C slower and of longer-duration.
D slower and of shorter-duration.

26 In an experiment to investigate phototropism, a plant shoot is grown with light coming from one
side only.

After two days, in which region has the greatest rate of growth occurred?

D
A
C
light B light

start of experiment after two days

215
© UCLES 2018 0610/23/M/J/18 [Turn over
12

27 Crop plants produced by asexual reproduction are identical. This can be a disadvantage.

Which statements describe disadvantages?

1 Crop plants are produced rapidly.


2 A disease could kill all of the plants.
3 Only one parent is needed.
4 The plants cannot adapt to environmental changes.

A 1 and 2 B 1 and 3 C 2 and 4 D 3 and 4

28 The table shows some features of artificial insemination and in-vitro fertilisation.

Which row shows the features that are correct for both artificial insemination and in-vitro
fertilisation?

fertilisation happens implantation happens involves pregnancy


outside the female outside the female sexual develops naturally
reproductive system reproductive system intercourse after implantation

A    
B    
C    
D    

29 The graph shows the concentration of four hormones during the menstrual cycle.

Which line represents progesterone?

hormone B
level C
D

0 14 28
ovulation

day of cycle

216
© UCLES 2018 0610/23/M/J/18
13

30 Cystic fibrosis is an inherited disease. The allele for cystic fibrosis is recessive.

A woman is heterozygous and her partner is homozygous dominant.

What is true of any children they produce?

A All of their children will have cystic fibrosis.


B There is a one in two chance of being heterozygous.
C There is a one in four chance of being heterozygous.
D There is no chance of having the cystic fibrosis allele.

31 Which graph shows the correct change in mass of DNA in one cell during cell division by mitosis?

A B

mass of DNA mass of DNA

0 0
0 time 0 time

C D

mass of DNA mass of DNA

0 0
0 time 0 time

32 Cell X contains 16 chromosomes. It divides by meiosis.

How many chromosomes do each of the new cells contain?

A 4 B 8 C 16 D 32

33 Which statement about blood groups in humans is correct?

A They are determined by genes and the environment.


B They are determined only by the environment.
C They are determined only by genes.
D They show continuous variation.

217
© UCLES 2018 0610/23/M/J/18 [Turn over
14

34 Plants are adapted to survive in different environments.

Which are features of xerophytes?

number
cuticle
of stomata

A thick few
B thick many
C thin few
D thin many

35 In which form is nitrogen taken up by the roots of plants?

A as amino acids
B as nitrate ions
C as protein
D as urea

36 What is a community?

A a unit containing all the organisms in a given area, and their environment, interacting
together
B a unit in which energy is transferred from one organism to the next
C all of the populations of different species in an ecosystem
D all the organisms of one species in a given area

37 Which feature of bacteria makes it possible for them to be used to produce human insulin?

A Bacteria possess plasmids.


B Bacteria possess exactly the same DNA base sequences as humans.
C Bacteria possess the same genes for insulin.
D Bacteria possess the same number of chromosomes as humans.

218
© UCLES 2018 0610/23/M/J/18
15

38 In some areas, farmers who grow genetically modified (GM) corn have to make sure there is a
gap between GM and non-GM crops.

What is the reason for leaving a gap between the crops?

A so they can get large machinery into the fields


B to prevent cross-pollination between GM and non-GM crops
C to prevent disease spreading between crops
D to prevent pests attacking crops

39 How does eutrophication lead to the death of aquatic organisms?

A algae not releasing enough oxygen


B algae respiring instead of photosynthesising
C decomposer bacteria lowering oxygen concentration in the water
D poisoning due to carbon dioxide accumulation in the water

40 What is an example of sustainable development?

A burning more fossil fuels to provide more energy


B catching more fish to feed a growing population
C clearing tropical forest to plant large areas of oil palm plants
D selective felling of forest trees for timber and replanting cleared areas

219
© UCLES 2018 0610/23/M/J/18
Cambridge International Examinations
Cambridge International General Certificate of Secondary Education

BIOLOGY 0610/11
Paper 1 Multiple Choice (Core) May/June 2018
45 minutes
Additional Materials: Multiple Choice Answer Sheet
Soft clean eraser
*9449437177*

Soft pencil (type B or HB is recommended)

READ THESE INSTRUCTIONS FIRST

Write in soft pencil.


Do not use staples, paper clips, glue or correction fluid.
Write your name, Centre number and candidate number on the Answer Sheet in the spaces provided
unless this has been done for you.
DO NOT WRITE IN ANY BARCODES.

There are forty questions on this paper. Answer all questions. For each question there are four possible
answers A, B, C and D.
Choose the one you consider correct and record your choice in soft pencil on the separate Answer Sheet.

Read the instructions on the Answer Sheet very carefully.

Each correct answer will score one mark. A mark will not be deducted for a wrong answer.
Any rough working should be done in this booklet.
Electronic calculators may be used.

This syllabus is approved for use in England, Wales and Northern Ireland as a Cambridge International Level 1/Level 2 Certificate.

This document consists of 14 printed pages and 2 blank pages.

IB18 06_0610_11/2RP
© UCLES 2018 [Turn220
over
2

1 Which organisms carry out respiration, growth, movement and excretion?

A all animals and all plants


B animals only
C arthropods and flowering plants only
D plants only

2 The diagram shows an animal whose scientific name is Falco peregrinus.

To which species does it belong?

A bird
B F. peregrinus
C Falco
D vertebrate

3 What kind of skin do amphibians have?

A dry without scales


B dry with scales
C moist without scales
D moist with scales

221
© UCLES 2018 0610/11/M/J/18
3

4 The diagram shows a flowering plant.

Use the key to identify the plant.

plant

flower has four petals flower has five petals

leaves with leaves with leaves with leaves with


smooth edges jagged edges smooth edges jagged edges

A B C D

5 The diagram shows a plant cell.

In which labelled part of the cell is sugar made?

D A
cell wall vacuole

B
chloroplast

C
nucleus

222
© UCLES 2018 0610/11/M/J/18 [Turn over
4

6 Why do some root cells have root hairs?

A for the maintenance of the temperature of the cell sap


B to increase the surface area of the cells
C to increase the volume of the cell sap
D to provide a place for cell nuclei

7 Which words correctly complete the paragraph?

Diffusion may be defined as the net movement of particles from a region of their ««1««
concentration, to a region of their ««2«« concentration, where movement is ««3«« a
concentration gradient.

1 2 3

A higher lower down


B higher lower up
C lower higher down
D lower higher up

8 Which part of a plant root hair is partially permeable?

A the cell sap


B the cell surface membrane
C the cell vacuole
D the cell wall

9 The table shows the results of food tests carried out on a fruit.

test Benedict’s biuret ethanol iodine


result positive positive negative negative

What did the fruit contain?

A fat and reducing sugar


B fat and starch
C protein and reducing sugar
D protein and starch

223
© UCLES 2018 0610/11/M/J/18
5

10 An experiment was carried out to investigate the effect of pH on enzyme action. The graph shows
the results.

y-axis

x-axis

What are the labels for the x-axis and the y-axis?

x-axis y-axis

A pH rate of reaction
B pH time
C rate of reaction pH
D time pH

11 The equation for photosynthesis is shown.

......1...... + ......2...... → glucose + ......3......

Which words correctly complete gaps 1, 2 and 3?

1 2 3

A carbon dioxide light oxygen


B carbon dioxide water oxygen
C oxygen light carbon dioxide
D oxygen water carbon dioxide

12 What is the best source of vitamin C in a balanced diet?

A fish
B fruit
C meat
D rice

224
© UCLES 2018 0610/11/M/J/18 [Turn over
6

13 A young, active woman requires more of which constituent in her diet than a young, active man?

A fat
B iron
C protein
D vitamin C

14 Which condition could be caused by a lack of iron?

A anaemia
B cholera
C scurvy
D diabetes

15 The diagram shows part of a section through a plant stem.

Which tissue transports water from the roots to the leaves?

16 Petroleum jelly is waterproof and transparent.

Covering the underside of the leaves of a plant with a thin layer of petroleum jelly will slow down
the rate of water loss from the plant.

Which statement explains this?

A Plants absorb nutrients from the petroleum jelly.


B Plants absorb water from the petroleum jelly.
C Stomata are blocked by the petroleum jelly.
D The petroleum jelly stops photosynthesis.

225
© UCLES 2018 0610/11/M/J/18
7

17 The diagram shows the human heart.

P Q

S R

In which order does blood pass through the chambers during a complete circuit of the body after
it returns from the lungs?

A Q→R→S→P

B Q→R→P→S

C P→S→Q→R

D P→S→R→Q

18 The diagrams show some components of the blood of a mammal.

Which component causes the blood to start clotting?

A B C D

19 Which statement describes a transmissible disease?

A a disease caused when a pathogen passes from one host to another


B a disease caused when a pathogen passes through a host’s body
C a disease caused when a pathogen passes to the host’s body only by direct contact
D a disease caused when a pathogen passes to the host’s body only by indirect contact

226
© UCLES 2018 0610/11/M/J/18 [Turn over
8

20 A sample of expired air is collected in a gas jar. Another gas jar contains normal atmospheric air.

A lighted candle is placed inside each gas jar as shown. The time taken for each flame to go out
is measured. As the candles burn they use up the oxygen available in the jar.

lid

air
sample

candle

gas jar X gas jar Y

The table shows the results of this experiment.

time for candle flame


gas jar
to go out / s

X 15
Y 9

What is an explanation of the difference between the results in jars X and Y?

A Jar X contains atmospheric air which has more carbon dioxide.


B Jar X contains expired air which has more carbon dioxide.
C Jar Y contains atmospheric air which has less oxygen.
D Jar Y contains expired air which has less oxygen.

21 Which row describes anaerobic respiration?

energy oxygen
waste products
released required

A a little no lactic acid


B a little yes carbon dioxide and water
C a lot no lactic acid
D a lot yes carbon dioxide and water

227
© UCLES 2018 0610/11/M/J/18
9

22 The diagram shows the excretory system.

What is structure X?

A bladder
B kidney
C ureter
D urethra

23 Which are effectors?

A brain and spinal cord


B muscles and glands
C receptors
D sensory neurones and motor neurones

24 Which hormone is involved in the conversion of glucose to glycogen?

A adrenaline
B insulin
C oestrogen
D testosterone

25 Which mechanism for maintaining body temperature involves the action of muscles?

A detection of temperature
B insulation with fatty tissue
C shivering
D sweating

228
© UCLES 2018 0610/11/M/J/18 [Turn over
10

26 Antibiotics are used to treat some diseases.

Which diseases could they be effective against?

bacterial
HIV scurvy
infection

A   
B   
C   
D   

27 The graph shows the number of people infected with HIV, in one part of the world, between 1985
and 2010.

30

25

number of 20
people
infected 15
with HIV
/ million 10

0
1985 1990 1995 2000 2005 2010

Using data from the graph, which statement is correct?

A Between 1995 and 2000 the number of people infected with HIV increased by 67%.
B Between 1995 and 2000 the number of people infected with HIV increased by 20%.
C Between 1990 and 1995 the number of people infected with HIV doubled.
D Between 1995 and 2000 the number of people infected with HIV doubled.

229
© UCLES 2018 0610/11/M/J/18
11

28 The diagram shows reproduction in a potato plant. Potato X was planted into the ground and a
plant grew from it. The plant then produced potato Y.

potato Y
potato X

Which statement is correct?

A X and Y are genetically different.


B Y was produced by asexual reproduction.
C Y was produced by sexual reproduction.
D Y was produced by the fusion of gametes.

29 Which row correctly links the named process with its description?

process description

A menstruation release of an egg


B menstruation shedding of the lining of the uterus
C ovulation fusion of nuclei
D ovulation start of puberty

230
© UCLES 2018 0610/11/M/J/18 [Turn over
12

30 The human immunodeficiency virus (HIV) can be transmitted in body fluid.

Some examples of body fluids are listed.

1 blood
2 saliva
3 semen
4 tear fluid

Which body fluids could transmit HIV?

A 1, 2, 3 and 4
B 1, 2 and 3 only
C 1 and 3 only
D 3 only

31 Which method of birth control involves a surgical procedure?

A condom
B contraceptive pill
C monitoring body temperature and cervical mucus
D vasectomy

32 The diagram shows the inheritance of sex in humans.

male gametes
X Y

female X XX XY
gametes
X XX XY

What is the chance of the couple’s next child being male?

A 25% B 50% C 75% D 100%

33 Which type of cell is produced by meiosis?

A fertilised egg cell


B leaf cell
C red blood cell
D sperm cell

231
© UCLES 2018 0610/11/M/J/18
13

34 What will cause the rate of mutation to increase?

A increase in exposure to ionising radiation


B increase in genetic variation in a population
C increase in phenotypic variation in a population
D increase in the rate of reproduction

35 A feature of some plants that live in dry places is that the leaves are reduced to spines.

What is an advantage of the leaves being reduced to spines?

A decrease in photosynthesis
B decrease in transpiration
C increase in photosynthesis
D increase in transpiration

36 Black truffle is a fungus that obtains nutrients from the roots of hazel trees. Female pigs find and
eat the fungi.

What is the principal source of the energy for this food chain?

A female pigs
B hazel trees
C soil
D the Sun

37 The diagram shows a food chain.

apple tree → herbivorous insects → carnivorous insects → bird

Which pyramid of numbers represents the food chain?

A B C D

232
© UCLES 2018 0610/11/M/J/18 [Turn over
14

38 Some washing powders are called biological washing powders.

What is the reason for this?

A They contain bacteria.


B They contain enzymes.
C They contain myriapods.
D They contain viruses.

39 What is an example of genetic engineering?

A inserting a gene for human insulin production into bacteria


B selective breeding to produce cattle with high milk yields
C using herbicides to kill unwanted plants
D using insecticides to kill insect pests

40 What is a negative impact to an ecosystem of large-scale monoculture?

A Crops need harvesting at the same time.


B Monocultures produce less food.
C Monocultures produce more food.
D The genetic variation of organisms is reduced.

233
© UCLES 2018 0610/11/M/J/18
Cambridge International Examinations
Cambridge International General Certificate of Secondary Education

BIOLOGY 0610/12
Paper 1 Multiple Choice (Core) May/June 2018
45 minutes
Additional Materials: Multiple Choice Answer Sheet
Soft clean eraser
*1276425606*

Soft pencil (type B or HB is recommended)

READ THESE INSTRUCTIONS FIRST

Write in soft pencil.


Do not use staples, paper clips, glue or correction fluid.
Write your name, Centre number and candidate number on the Answer Sheet in the spaces provided
unless this has been done for you.
DO NOT WRITE IN ANY BARCODES.

There are forty questions on this paper. Answer all questions. For each question there are four possible
answers A, B, C and D.
Choose the one you consider correct and record your choice in soft pencil on the separate Answer Sheet.

Read the instructions on the Answer Sheet very carefully.

Each correct answer will score one mark. A mark will not be deducted for a wrong answer.
Any rough working should be done in this booklet.
Electronic calculators may be used.

This syllabus is approved for use in England, Wales and Northern Ireland as a Cambridge International Level 1/Level 2 Certificate.

This document consists of 16 printed pages.

IB18 06_0610_12/2RP
© UCLES 2018 [Turn234
over
2

1 Which organisms carry out respiration, growth, movement and excretion?

A all animals and all plants


B animals only
C arthropods and flowering plants only
D plants only

2 The diagram shows an animal whose scientific name is Falco peregrinus.

To which species does it belong?

A bird
B F. peregrinus
C Falco
D vertebrate

3 What kind of skin do amphibians have?

A dry without scales


B dry with scales
C moist without scales
D moist with scales

235
© UCLES 2018 0610/12/M/J/18
3

4 The diagram shows a flowering plant.

Use the key to identify the plant.

plant

flower has four petals flower has five petals

leaves with leaves with leaves with leaves with


smooth edges jagged edges smooth edges jagged edges

A B C D

5 The diagram shows a plant cell.

In which labelled part of the cell is sugar made?

D A
cell wall vacuole

B
chloroplast

C
nucleus

236
© UCLES 2018 0610/12/M/J/18 [Turn over
4

6 Why do some root cells have root hairs?

A for the maintenance of the temperature of the cell sap


B to increase the surface area of the cells
C to increase the volume of the cell sap
D to provide a place for cell nuclei

7 The table shows the concentration of gases in a blood vessel and in an alveolus.

Which row shows the conditions that cause a gas produced in respiration to diffuse from the
blood vessel into the alveolus?

concentration in concentration in
gas produced
the blood vessel the alveolus

A carbon dioxide low high


B carbon dioxide high low
C oxygen low high
D oxygen high low

8 Which part of a plant root hair is partially permeable?

A the cell sap


B the cell surface membrane
C the cell vacuole
D the cell wall

9 The table shows the results of food tests carried out on a fruit.

test Benedict’s biuret ethanol iodine


result positive positive negative negative

What did the fruit contain?

A fat and reducing sugar


B fat and starch
C protein and reducing sugar
D protein and starch

237
© UCLES 2018 0610/12/M/J/18
5

10 An experiment was carried out to investigate the effect of pH on enzyme action. The graph shows
the results.

y-axis

x-axis

What are the labels for the x-axis and the y-axis?

x-axis y-axis

A pH rate of reaction
B pH time
C rate of reaction pH
D time pH

238
© UCLES 2018 0610/12/M/J/18 [Turn over
6

11 An experiment is set up as shown.

small plant

gauze platform

green
indicator

test-tube X test-tube Y

The green indicator turns yellow when the concentration of carbon dioxide increases. The green
indicator turns blue when the concentration of carbon dioxide decreases.

After several hours, the indicator in test-tube X turned blue. The indicator in test-tube Y remained
green.

Which process caused the colour change?

A germination
B photosynthesis
C respiration
D transpiration

12 Why do plants need nitrate ions?

A for making amino acids


B for making fatty acids
C for making glucose
D for making starch

13 In which part of the alimentary canal do chemical digestion and mechanical digestion take place?

A colon
B duodenum
C mouth
D oesophagus

239
© UCLES 2018 0610/12/M/J/18
7

14 The diagram shows part of the alimentary canal.

Where is most water absorbed?

B
A

15 A celery stalk was placed in a beaker which contained a red stain. After 24 hours, the red stain
appeared at the top of the celery stalk.

Which structures stained red?

A cortex cells
B mesophyll cells
C phloem
D xylem

16 A student is investigating the effect of temperature on the rate of transpiration.

Which environmental conditions should be kept constant during this investigation?

humidity light intensity temperature wind speed

A    
B    
C    
D    
240
© UCLES 2018 0610/12/M/J/18 [Turn over
8

17 The diagram shows the human heart.

P Q

S R

In which order does blood pass through the chambers during a complete circuit of the body after
it returns from the lungs?

A Q→R→S→P

B Q→R→P→S

C P→S→Q→R

D P→S→R→Q

18 The diagrams show some components of the blood of a mammal.

Which component causes the blood to start clotting?

A B C D

19 The body produces mucus as a defence against diseases.

What type of defence is stomach acid?

chemical mechanical
cells
barrier barrier

A   
B   
C   
D   
241
© UCLES 2018 0610/12/M/J/18
9

20 A sample of expired air is collected in a gas jar. Another gas jar contains normal atmospheric air.

A lighted candle is placed inside each gas jar as shown. The time taken for each flame to go out
is measured. As the candles burn they use up the oxygen available in the jar.

lid

air
sample

candle

gas jar X gas jar Y

The table shows the results of this experiment.

time for candle flame


gas jar
to go out / s

X 15
Y 9

What is an explanation of the difference between the results in jars X and Y?

A Jar X contains atmospheric air which has more carbon dioxide.


B Jar X contains expired air which has more carbon dioxide.
C Jar Y contains atmospheric air which has less oxygen.
D Jar Y contains expired air which has less oxygen.

21 Which row describes anaerobic respiration?

energy oxygen
waste products
released required

A a little no lactic acid


B a little yes carbon dioxide and water
C a lot no lactic acid
D a lot yes carbon dioxide and water

242
© UCLES 2018 0610/12/M/J/18 [Turn over
10

22 The diagram shows the excretory system.

What is structure X?

A bladder
B kidney
C ureter
D urethra

23 The diagram shows structures in a reflex arc.

receptor

muscle

What is X?

A effector
B relay neurone
C sensory neurone
D synapse

243
© UCLES 2018 0610/12/M/J/18
11

24 Which row describes the effect of the hormone adrenaline on breathing rate, pulse rate and pupil
size?

breathing rate pulse rate pupil size

A decrease decrease larger


B decrease increase smaller
C increase decrease smaller
D increase increase larger

25 The diagram shows a seedling growing inside a dark box.

dark box

shoot

root

What type of responses affect the direction of growth of the root and the shoot in this experiment?

response by the root response by the shoot

A gravitropism gravitropism
B gravitropism phototropism
C phototropism gravitropism
D phototropism phototropism

26 What is the definition of a drug?

A any illegal substance taken into the body


B any substance taken into the body
C any substance taken into the body that modifies or affects chemical reactions in the body
D any substance taken into the body which is not a medicine

244
© UCLES 2018 0610/12/M/J/18 [Turn over
12

27 The graph shows the number of alcohol-related deaths in men and women between the years
2001 and 2005 in one country.

400
key
300 men
number of
alcohol- women
200
related
deaths
100

0
0–19 20–34 35–49 50–64 65+
age group / years

Which conclusion is supported by the data in the graph?

A All alcohol-related deaths increase with age.


B Men in the 35– 49 year old age group are most likely to die due to alcohol-related reasons.
C Men in the 65+ age group are least likely to die due to alcohol-related reasons.
D Women are more likely to die due to alcohol-related reasons than men.

28 The diagram shows a potato tuber that developed from the stem of a parent potato plant. Three
shoots are starting to grow from the tuber.

shoot

tuber

How do the genotypes of the shoots compare with the genotypes of the tuber and of the parent?

genotype genotype
of tuber of parent

A is different to the shoots is different to the shoots


B is different to the shoots is identical to the shoots
C is identical to the shoots is different to the shoots
D is identical to the shoots is identical to the shoots

245
© UCLES 2018 0610/12/M/J/18
13

29 The list shows various terms used in sexual reproduction in organisms.

1 male gamete
2 female gamete
3 ovule
4 ovary
5 filament
6 fertilisation
7 haploid

Which terms are only used in sexual reproduction in plants?

A 1 and 7 B 2 and 4 C 2 and 6 D 3 and 5

30 The diagram shows stages of a menstrual cycle.

During which stage does menstruation occur?

B
ion
ovulat

246
© UCLES 2018 0610/12/M/J/18 [Turn over
14

31 The diagram shows part of the female reproductive system with a birth control device in place.

uterus

Which birth control device is being used?

A chemical implant
B diaphragm
C femidom
D IUD

32 Which chromosomes can be found in a single sperm?

A X and X B X and Y C X or X D X or Y

33 Which term is used for an organism that has two different alleles of a particular gene?

A dominant
B heterozygous
C homozygous
D recessive

34 What is an adaptive feature of an organism?

A any feature that is changed by the environment


B any feature that helps an organism to survive and reproduce
C any feature that shows continuous variation
D any inherited feature than an organism has

35 What is required for natural selection to occur?

A genetic variation between individuals


B humans selecting desirable characteristics
C no competition between individuals or resources
D offspring produced by asexual reproduction

247
© UCLES 2018 0610/12/M/J/18
15

36 What is the principal source of energy for most food chains?

A carbon dioxide gas


B glucose
C oxygen
D sunlight

37 The diagram shows part of the carbon cycle.

carbon dioxide in
atmosphere

gradual production of
fossil fuels

coal, oil
and gas

Which process is missing from the diagram?

A combustion
B photosynthesis
C plant respiration
D decomposition

38 Genetic engineering has been used to produce human insulin.

Into which type of cell were the human genes for insulin inserted?

A animal
B bacterial
C fungal
D human

248
© UCLES 2018 0610/12/M/J/18 [Turn over
16

39 What are reasons for using chemical fertilisers in food production?

reduce competition
increase pollution increase
between crops
in rivers and lakes crop yields
and weeds

A   
B   
C   
D   

40 As well as carbon dioxide, which other gas is mainly responsible for the enhanced greenhouse
effect?

A methane
B oxygen
C ozone
D sulfur dioxide

Permission to reproduce items where third-party owned material protected by copyright is included has been sought and cleared where possible. Every
reasonable effort has been made by the publisher (UCLES) to trace copyright holders, but if any items requiring clearance have unwittingly been included, the
publisher will be pleased to make amends at the earliest possible opportunity.

To avoid the issue of disclosure of answer-related information to candidates, all copyright acknowledgements are reproduced online in the Cambridge
International Examinations Copyright Acknowledgements Booklet. This is produced for each series of examinations and is freely available to download at
www.cie.org.uk after the live examination series.

249
Cambridge International Examinations is part of the Cambridge Assessment Group. Cambridge Assessment is the brand name of University of Cambridge Local
Examinations Syndicate (UCLES), which is itself a department of the University of Cambridge.

© UCLES 2018 0610/12/M/J/18


Cambridge International Examinations
Cambridge International General Certificate of Secondary Education

BIOLOGY 0610/13
Paper 1 Multiple Choice (Core) May/June 2018
45 minutes
Additional Materials: Multiple Choice Answer Sheet
Soft clean eraser
*0433363336*

Soft pencil (type B or HB is recommended)

READ THESE INSTRUCTIONS FIRST

Write in soft pencil.


Do not use staples, paper clips, glue or correction fluid.
Write your name, Centre number and candidate number on the Answer Sheet in the spaces provided
unless this has been done for you.
DO NOT WRITE IN ANY BARCODES.

There are forty questions on this paper. Answer all questions. For each question there are four possible
answers A, B, C and D.
Choose the one you consider correct and record your choice in soft pencil on the separate Answer Sheet.

Read the instructions on the Answer Sheet very carefully.

Each correct answer will score one mark. A mark will not be deducted for a wrong answer.
Any rough working should be done in this booklet.
Electronic calculators may be used.

This syllabus is approved for use in England, Wales and Northern Ireland as a Cambridge International Level 1/Level 2 Certificate.

This document consists of 15 printed pages and 1 blank page.

IB18 06_0610_13/3RP
© UCLES 2018 [Turn250
over
2

1 Which organisms carry out respiration, growth, movement and excretion?

A all animals and all plants


B animals only
C arthropods and flowering plants only
D plants only

2 The diagram shows an animal whose scientific name is Falco peregrinus.

To which species does it belong?

A bird
B F. peregrinus
C Falco
D vertebrate

3 What kind of skin do amphibians have?

A dry without scales


B dry with scales
C moist without scales
D moist with scales

251
© UCLES 2018 0610/13/M/J/18
3

4 The diagram shows a flowering plant.

Use the key to identify the plant.

plant

flower has four petals flower has five petals

leaves with leaves with leaves with leaves with


smooth edges jagged edges smooth edges jagged edges

A B C D

5 The diagram shows a plant cell.

In which labelled part of the cell is sugar made?

D A
cell wall vacuole

B
chloroplast

C
nucleus

252
© UCLES 2018 0610/13/M/J/18 [Turn over
4

6 Why do some root cells have root hairs?

A for the maintenance of the temperature of the cell sap


B to increase the surface area of the cells
C to increase the volume of the cell sap
D to provide a place for cell nuclei

7 The diagram shows a section through an alveolus and through a capillary.

wall of alveolus
air

capillary red
wall blood cells

capillary

low carbon dioxide Y


concentration

high carbon dioxide


concentration

How does carbon dioxide move from Y to X?

A by diffusion
B by osmosis
C by translocation
D by transpiration

8 Which part of a plant root hair is partially permeable?

A the cell sap


B the cell surface membrane
C the cell vacuole
D the cell wall

253
© UCLES 2018 0610/13/M/J/18
5

9 The table shows the results of food tests carried out on a fruit.

test Benedict’s biuret ethanol iodine


result positive positive negative negative

What did the fruit contain?

A fat and reducing sugar


B fat and starch
C protein and reducing sugar
D protein and starch

10 An experiment was carried out to investigate the effect of pH on enzyme action. The graph shows
the results.

y-axis

x-axis

What are the labels for the x-axis and the y-axis?

x-axis y-axis

A pH rate of reaction
B pH time
C rate of reaction pH
D time pH

254
© UCLES 2018 0610/13/M/J/18 [Turn over
6

11 The diagram shows a section through a leaf.

S R

What are structures P, Q, R and S?

P Q R S

A chloroplast phloem palisade tissue xylem


B cuticle xylem chloroplast stoma
C phloem palisade tissue cuticle stoma
D xylem chloroplast phloem cuticle

12 A lack of which dietary component can result in constipation?

A fat
B fibre (roughage)
C carbohydrate
D protein

13 A woman requires more calcium in her diet when she is pregnant.

Which statement explains why?

A bone growth in the fetus


B muscle growth in the fetus
C production of haemoglobin in the fetus
D to provide energy for the fetus

255
© UCLES 2018 0610/13/M/J/18
7

14 The diagram shows part of the alimentary canal.

M
K

Which row correctly identifies the structures labelled J to M?

J K L M

A oesophagus pancreas small intestine stomach


B pancreas small intestine stomach oesophagus
C small intestine stomach oesophagus pancreas
D stomach oesophagus pancreas small intestine

15 The diagram shows the pathway of water from the soil through a plant.

soil → X → Y → Z → mesophyll cells

Which cells are represented by X, Y and Z?

X Y Z

A root hair cells root cortex cells xylem


B root hair cells xylem root cortex cells
C xylem root cortex cells root hair cells
D xylem root hair cells root cortex cells

16 What will not affect the rate of transpiration?

A humidity of the atmosphere


B number of open stomata
C rate of respiration
D temperature

256
© UCLES 2018 0610/13/M/J/18 [Turn over
8

17 The diagram shows the human heart.

P Q

S R

In which order does blood pass through the chambers during a complete circuit of the body after
it returns from the lungs?

A Q→R→S→P

B Q→R→P→S

C P→S→Q→R

D P→S→R→Q

18 The diagrams show some components of the blood of a mammal.

Which component causes the blood to start clotting?

A B C D

19 What is an indirect method of transmitting disease?

A body fluid
B blood
C food
D skin contact between people

257
© UCLES 2018 0610/13/M/J/18
9

20 A sample of expired air is collected in a gas jar. Another gas jar contains normal atmospheric air.

A lighted candle is placed inside each gas jar as shown. The time taken for each flame to go out
is measured. As the candles burn they use up the oxygen available in the jar.

lid

air
sample

candle

gas jar X gas jar Y

The table shows the results of this experiment.

time for candle flame


gas jar
to go out / s

X 15
Y 9

What is an explanation of the difference between the results in jars X and Y?

A Jar X contains atmospheric air which has more carbon dioxide.


B Jar X contains expired air which has more carbon dioxide.
C Jar Y contains atmospheric air which has less oxygen.
D Jar Y contains expired air which has less oxygen.

21 Which row describes anaerobic respiration?

energy oxygen
waste products
released required

A a little no lactic acid


B a little yes carbon dioxide and water
C a lot no lactic acid
D a lot yes carbon dioxide and water

258
© UCLES 2018 0610/13/M/J/18 [Turn over
10

22 The diagram shows the excretory system.

What is structure X?

A bladder
B kidney
C ureter
D urethra

23 What is the correct sequence in a reflex action?

A receptor → stimulus → motor neurone → relay neurone → sensory neurone → effector

B receptor → stimulus → sensory neurone → relay neurone → motor neurone → effector

C stimulus → receptor → motor neurone → relay neurone → sensory neurone → effector

D stimulus → receptor → sensory neurone → relay neurone → motor neurone → effector

24 In 1921, Banting and Best extracted a substance from the pancreas of an animal. The substance
was injected into children whose blood sugar concentrations were too high. The children’s blood
sugar concentrations were returned to normal.

Which substance in the pancreatic extract would return the children’s blood sugar concentrations
to normal?

A adrenaline
B insulin
C glucose
D oestrogen

259
© UCLES 2018 0610/13/M/J/18
11

25 A seed is placed and grown on a rotating disc, as shown.

seed pinned to disc

Which diagram shows the appearance of the seedling shoot after seven days?

A B C D

26 Which organ is the site of break down of alcohol in the human body?

A bladder
B kidney
C liver
D stomach

27 Alcohol is a drug.

Which statement is correct?

A It can cause COPD.


B It is a depressant.
C It is not addictive.
D It speeds up reaction times.

28 The following statements give information about the reproduction of different organisms.

1 Daffodil plants produce genetically identical bulbs that grow into a new plant the
following year.
2 In summer, female aphids produce genetically identical offspring without fertilisation
by a male.
3 Most banana plants are genetically identical because they were produced from one
parent plant.

Which statements are examples of asexual reproduction?

A 1, 2 and 3 B 1 and 2 only C 1 and 3 only D 2 and 3 only


260
© UCLES 2018 0610/13/M/J/18 [Turn over
12

29 Which stages in sexual reproduction occur in both flowering plants and human reproduction?

1 Gametes are made by meiosis.


2 Gametes fuse to form a zygote.
3 The male gamete moves to the female gamete.

A 1, 2 and 3 B 1 and 2 only C 1 only D 2 and 3 only

30 Which part of the female reproductive system produces oestrogen?

A cervix
B ovary
C oviduct
D uterus

31 Which hormone is responsible for the development of secondary sexual characteristics?

A FSH
B LH
C oestrogen
D progesterone

32 A length of DNA codes for a protein.

What is this a definition of?

A a gene
B an amino acid
C an X chromosome
D a Y chromosome

261
© UCLES 2018 0610/13/M/J/18
13

33 The diagrams show human male and female reproductive organs.

X
P

Q Z

Where does meiosis take place?

A P and X B Q and X C Q and Y D R and Z

34 The bar chart shows the heights of pea plants grown from 500 pea seeds.

number of
pea plants

height of plants

What variation do the plants show?

continuous discontinuous

A  
B  
C  
D  

262
© UCLES 2018 0610/13/M/J/18 [Turn over
14

35 The picture shows a fox that lives in the desert.

Which adaptation does the fox have for living in the desert?

A eyes at front of head to help judge distance


B large ears to increase heat loss
C long bushy tail for balance
D sharp teeth to catch prey

36 The diagram shows a simple food chain.

1 2 3

green plants chickens wild dogs

Which process releases the energy that is lost at 1, 2 and 3?

A ingestion
B photosynthesis
C respiration
D transpiration

263
© UCLES 2018 0610/13/M/J/18
15

37 Which labelled stage in the water cycle shows precipitation?

B
C

plant
roots
A
land
sea river

38 What is the name of the structure that separates the left and right ventricles of the heart?

A atrium
B septum
C vena cava
D valve

39 What is a source of air pollution?

A herbicides
B methane
C untreated sewage
D water vapour

40 Which process does not change the carbon dioxide level in the air?

A decomposition
B feeding
C photosynthesis
D respiration

264
© UCLES 2018 0610/13/M/J/18
Cambridge International Examinations
Cambridge International General Certificate of Secondary Education

BIOLOGY 0610/22
Paper 2 Multiple Choice (Extended) February/March 2018
45 minutes
Additional Materials: Multiple Choice Answer Sheet
Soft clean eraser
*3265335475*

Soft pencil (type B or HB is recommended)

READ THESE INSTRUCTIONS FIRST

Write in soft pencil.


Do not use staples, paper clips, glue or correction fluid.
Write your name, Centre number and candidate number on the Answer Sheet in the spaces provided
unless this has been done for you.
DO NOT WRITE IN ANY BARCODES.

There are forty questions on this paper. Answer all questions. For each question there are four possible
answers A, B, C and D.
Choose the one you consider correct and record your choice in soft pencil on the separate Answer Sheet.

Read the instructions on the Answer Sheet very carefully.

Each correct answer will score one mark. A mark will not be deducted for a wrong answer.
Any rough working should be done in this booklet.
Electronic calculators may be used.

This syllabus is approved for use in England, Wales and Northern Ireland as a Cambridge International Level 1/Level 2 Certificate.

This document consists of 16 printed pages.

IB18 03_0610_22/4RP
© UCLES 2018 [Turn265
over
2

1 Biology is the study of living things.

Which characteristic applies to all forms of life?

A able to move from place to place


B able to reproduce
C carry out photosynthesis
D possess a nervous system

2 Two animals have an identical sequence of amino acids in one of the proteins found in their cells.

What does this indicate about these animals?

A They have been eating the same types of food.


B They have not been exposed to substances that cause mutation.
C They must be members of the same genus.
D They share a recent ancestor.

3 The diagram shows part of a flowering plant.

Using the key, identify this plant.

1 three petals ................................................. go to 2


more than three petals ............................... go to 3
2 leaves longer than they are wide ................ A
leaves wider than they are long .................. B
3 leaves parallel-veined ................................. C
leaves not parallel-veined ........................... D

266
© UCLES 2018 0610/22/F/M/18
3

4 The diagram shows a single cell from an organism called Spirogyra.

cytoplasm chloroplast cell wall

vacuole

cytoplasmic nucleus
strand

Which features does Spirogyra share with plant cells?

cell wall chloroplast cytoplasm nucleus vacuole

A     
B     
C     
D     

5 A student was told that a drawing of a bacterial cell had been magnified 30 000 times. The length
of the drawing was 45 mm.

What was the actual length of the bacterium?

A 0.15 mm B 1.5 µm C 0.66 mm D 0.66 µm

267
© UCLES 2018 0610/22/F/M/18 [Turn over
4

6 Which graph represents the effect of increasing temperature on the rate of diffusion?

A B

rate of rate of
diffusion diffusion

0 0
0 temperature 0 temperature

C D

rate of rate of
diffusion diffusion

0 0
0 temperature 0 temperature

7 Protoplasts are plant cells that have had their cell walls removed.

What happens if plant protoplasts are placed in distilled water?

A They get larger and become turgid.


B They get larger and burst.
C They get smaller and become plasmolysed.
D They get smaller and shrivel up.

8 When a food substance is tested with iodine solution, which colour shows the presence of
starch?

A blue-black
B brown
C orange
D purple

268
© UCLES 2018 0610/22/F/M/18
5

9 The graph shows the activity of three digestive enzymes at differing pH levels.

X Y Z

enzyme
activity

1 2 3 4 5 6 7 8 9 10 11 12
pH

Which statement is correct?

A Enzymes X and Y are both active at pH 7.


B Enzymes X and Z are both active at pH 4.
C Enzymes Y and Z are both active at pH 4.
D Enzymes Y and Z are both active at pH 8.

10 The diagram shows a cross-section of part of a leaf.

upper epidermis
layer X

lower epidermis

midrib

Which type of cell is found in layer X?

A B C D

269
© UCLES 2018 0610/22/F/M/18 [Turn over
6

11 Which molecule contains magnesium?

A chlorophyll
B fat
C glucose
D starch

12 In which part of the body of a mammal does mechanical digestion occur?

A gall bladder
B liver
C mouth
D pancreas

13 During growth, potato plants produce flowers and underground storage organs called tubers.

During this time, which parts of the plant act as sources and sinks for translocation?

flowers leaves potato tubers

A sink sink source


B sink source sink
C source sink source
D source source sink

14 What is an advantage of a double circulatory system in mammals?

A Blood can flow down the body on the left and up the body on the right.
B Blood can flow more slowly along the circulatory system.
C Blood pressure stays the same throughout the circulatory system.
D Oxygenated and deoxygenated blood are kept separate.

15 The body has defences to protect itself from diseases.

What is a mechanical barrier to diseases?

A hairs in the nose


B plasma
C stomach acid
D white blood cells

270
© UCLES 2018 0610/22/F/M/18
7

16 During exercise, receptors detect a change in the blood and cause the breathing rate to increase.

What change do the receptors detect and where are they found in the body?

change in blood site of detection

A carbon dioxide increases brain


B carbon dioxide increases intercostal muscles
C oxygen decreases brain
D oxygen decreases intercostal muscles

17 An experiment is set up to investigate the uptake of oxygen by germinating seeds.

water bath soda lime absorbs


at 25 °C carbon dioxide
dead seeds rinsed
germinating in antiseptic
seeds
X Y

coloured coloured
water water

What happens to the levels of the coloured water at X and Y?

X Y

A falls rises
B falls unchanged
C rises falls
D rises unchanged

271
© UCLES 2018 0610/22/F/M/18 [Turn over
8

18 The diagram shows a section of a kidney.

What are the correct labels?

X Y Z

A cortex medulla ureter


B cortex ureter medulla
C medulla cortex ureter
D medulla ureter cortex

19 A boy accidentally touches a very hot object and immediately takes his hand away.

In this reflex action, what is the effector?

A a heat receptor in his hand


B a motor neurone
C a muscle in his arm
D the spinal cord

20 Which description of how the pupil of the eye gets smaller is correct?

circular muscles radial muscles

A contract contract
B contract relax
C relax contract
D relax relax

272
© UCLES 2018 0610/22/F/M/18
9

21 There are two types of diabetes, type 1 and type 2.

The graph shows the number of children with each type of diabetes per 100 000 children, in one
country.

350

300 type 1
type 2
250
number of
children with 200
diabetes
per 100 000 150
children
100

50

0
2 3 4 5 6 7 8 9 10 11 12 13 14 15 16 17

age / years

Which conclusion can be made from the graph?

A 1.5% of 9-year-olds have diabetes.


B 31.3% of 17-year-olds with diabetes have type 2 diabetes.
C There are 10 more 12-year-olds in the country with diabetes than 13-year-olds.
D Type 2 diabetes will cause more health problems than type 1.

273
© UCLES 2018 0610/22/F/M/18 [Turn over
10

22 The diagram shows an experiment using wheat shoot tips to investigate plant growth.

after two days


in even light

shoot tip
removed

X Y X Y

Which statement is supported by the evidence provided by this experiment?

A Auxin moves through the plant by osmosis.


B Auxin is made in the shoot tip.
C Auxin is unequally distributed in response to light.
D Auxin inhibits cell elongation.

23 A wind-pollinated plant has which features?

A large anthers, coloured petals and produces nectar


B large petals, small anthers and a sticky stigma
C small petals, large anthers and a feathery stigma
D small petals, produces nectar and has a strong scent

24 Which describes a human male gamete?

relative size compared flagellum


motile
to female gamete present

A yes larger yes


B yes smaller yes
C no smaller no
D no larger no

274
© UCLES 2018 0610/22/F/M/18
11

25 What is a consequence of HIV infection on the human body?

A decreased lymphocyte numbers


B increased protection against bacterial infections
C increased haemoglobin production
D sickle-cell anaemia

26 Which sex chromosomes in the egg and the sperm will produce a male child?

sex chromosome sex chromosome


in egg in sperm

A X X
B X Y
C Y X
D Y Y

27 Cell division has the following functions.

1 asexual reproduction
2 growth
3 production of gametes
4 repair of damaged tissue
5 replacement of cells

Which functions are specific to mitosis?

A 1, 2, 3 and 4
B 1, 2, 4 and 5
C 2, 3 and 4 only
D 3, 4 and 5 only

275
© UCLES 2018 0610/22/F/M/18 [Turn over
12

28 The chart shows the inheritance of fur colour in a small mammal.

If the allele for white fur is dominant, which animal must be heterozygous for the gene controlling
fur colour?

A B

29 In areas of the world where malaria is present, the sickle-cell allele is more common.

What is the reason for this?

A Both diseases are caused by the same allele.


B Heterozygous individuals with the sickle-cell allele are less likely to have malaria.
C Heterozygous individuals with the sickle-cell allele are more likely to have malaria.
D These are parts of the world with many diseases.

30 Which feature helps a xerophyte survive in its environment?

A flat leaves with a large surface area


B no cuticle
C short roots
D sunken stomata

276
© UCLES 2018 0610/22/F/M/18
13

31 What is a feature of natural selection?

A It does not require a struggle for survival.


B It does not require variation in a population.
C It involves the selection of advantageous phenotypes.
D It involves the selection of disadvantageous genotypes.

32 The diagram shows energy flow through an ecosystem.

green
Sun plant herbivore carnivore

decomposer

In what form is energy transferred at X?

A chemical
B heat
C kinetic
D light

33 The diagram shows part of the carbon cycle.

Which process reduces the carbon dioxide content of the atmosphere?

A B
carbon dioxide
in atmosphere
C

industry
transport D animals plants
and power

fossil fuels (coal and oil) decomposers

277
© UCLES 2018 0610/22/F/M/18 [Turn over
14

34 The graph shows the human population of the world for the last 2000 years.

6000

4000
population
/ million

2000 P

0
0 1000 2000
time / years

What is the phase identified by P?

A death
B exponential (log)
C lag
D stationary

35 Some of the characteristics of bacteria are listed.

1 can make complex molecules


2 can reproduce quickly
3 have cytoplasm
4 may cause diseases

Which characteristics make bacteria useful in biotechnology and genetic engineering?

A 1, 2, 3 and 4 B 1 and 2 only C 2 and 3 only D 3 and 4 only

36 What is a requirement for the production of penicillin in a fermenter?

A adding bubbles of nitrogen gas to mix the nutrients and Penicillium


B adding the required amount of sugar as a nutrient
C ensuring no oxygen enters the fermenter so only anaerobic respiration occurs

D maintaining a constant temperature of 95 °C to prevent other microorganisms growing

278
© UCLES 2018 0610/22/F/M/18
15

37 Human insulin can be produced in large quantities by modified E. coli bacteria.

Four of the steps in this production process are listed.

1 Insulin is removed from the bacterial culture.


2 An enzyme is used to cut out the insulin gene from a human chromosome.
3 The insulin gene is placed into the plasmid of the bacterium.
4 Bacteria with the insulin gene reproduce very rapidly.

What is the order of these steps?

A 1→2→3→4

B 1→3→4→2

C 2→3→4→1

D 4→1→2→3

38 What is a major contributor to the problem of worldwide famine?

A equal distribution of food


B increasing population and life expectancy
C large-scale monoculture of crop plants
D use of herbicides and insecticides in farming

39 The flow diagram shows a process that can occur as a result of pollution in a river.

low
high
algae algae concentration
concentrations
reproduce die and of dissolved
of nitrate ions
rapidly decompose oxygen kills
enter a river
organisms

Which term describes the process shown in the flow diagram?

A aerobic respiration
B eutrophication
C nitrogen cycle
D photosynthesis

279
© UCLES 2018 0610/22/F/M/18 [Turn over
16

40 Which stage in the treatment of sewage removes large floating objects, such as plastic bags?

A aeration
B discharge
C screening
D sedimentation

Permission to reproduce items where third-party owned material protected by copyright is included has been sought and cleared where possible. Every
reasonable effort has been made by the publisher (UCLES) to trace copyright holders, but if any items requiring clearance have unwittingly been included, the
publisher will be pleased to make amends at the earliest possible opportunity.

To avoid the issue of disclosure of answer-related information to candidates, all copyright acknowledgements are reproduced online in the Cambridge
International Examinations Copyright Acknowledgements Booklet. This is produced for each series of examinations and is freely available to download at
www.cie.org.uk after the live examination series.

280
Cambridge International Examinations is part of the Cambridge Assessment Group. Cambridge Assessment is the brand name of University of Cambridge Local
Examinations Syndicate (UCLES), which is itself a department of the University of Cambridge.

© UCLES 2018 0610/22/F/M/18


Cambridge International Examinations
Cambridge International General Certificate of Secondary Education

BIOLOGY 0610/12
Paper 1 Multiple Choice (Core) February/March 2018
45 minutes
Additional Materials: Multiple Choice Answer Sheet
Soft clean eraser
*0935368408*

Soft pencil (type B or HB is recommended)

READ THESE INSTRUCTIONS FIRST

Write in soft pencil.


Do not use staples, paper clips, glue or correction fluid.
Write your name, Centre number and candidate number on the Answer Sheet in the spaces provided
unless this has been done for you.
DO NOT WRITE IN ANY BARCODES.

There are forty questions on this paper. Answer all questions. For each question there are four possible
answers A, B, C and D.
Choose the one you consider correct and record your choice in soft pencil on the separate Answer Sheet.

Read the instructions on the Answer Sheet very carefully.

Each correct answer will score one mark. A mark will not be deducted for a wrong answer.
Any rough working should be done in this booklet.
Electronic calculators may be used.

This syllabus is approved for use in England, Wales and Northern Ireland as a Cambridge International Level 1/Level 2 Certificate.

This document consists of 15 printed pages and 1 blank page.

IB18 03_0610_12/4RP
© UCLES 2018 [Turn281
over
2

1 What is a characteristic of all living organisms?

A breathing
B egestion
C excretion
D ingestion

2 What are the characteristics of fish?

maintain
external ears jelly-covered
constant body scales
present eggs
temperature

A    
B    
C    
D    

3 The diagram shows part of a flowering plant.

Using the key, identify this plant.

1 three petals ................................................. go to 2


more than three petals ............................... go to 3
2 leaves longer than they are wide ................ A
leaves wider than they are long .................. B
3 leaves parallel-veined ................................. C
leaves not parallel-veined ........................... D

282
© UCLES 2018 0610/12/F/M/18
3

4 The diagram shows a single cell from an organism called Spirogyra.

cytoplasm chloroplast cell wall

vacuole

cytoplasmic nucleus
strand

Which features does Spirogyra share with plant cells?

cell wall chloroplast cytoplasm nucleus vacuole

A     
B     
C     
D     

283
© UCLES 2018 0610/12/F/M/18 [Turn over
4

5 Which level of organisation is shown in the diagram?

lung capillaries

heart

body capillaries

A organ
B organism
C organ system
D tissue

6 An egg measured 6.5 cm in diameter. A student made a drawing of this egg and the diameter was
measured as 19.5 cm.

What was the magnification of the drawing?

A ×0.3 B ×3.0 C ×6.5 D ×300

284
© UCLES 2018 0610/12/F/M/18
5

7 The diagram shows a plant cell.

What is the appearance of this cell after it has been placed in pure water for 30 minutes?

A B

C D

8 When a food substance is tested with iodine solution, which colour shows the presence of
starch?

A blue-black
B brown
C orange
D purple

285
© UCLES 2018 0610/12/F/M/18 [Turn over
6

9 The graph shows the activity of three digestive enzymes at differing pH levels.

X Y Z

enzyme
activity

1 2 3 4 5 6 7 8 9 10 11 12
pH

Which statement is correct?

A Enzymes X and Y are both active at pH 7.


B Enzymes X and Z are both active at pH 4.
C Enzymes Y and Z are both active at pH 4.
D Enzymes Y and Z are both active at pH 8.

10 The diagram shows a cross-section of part of a leaf.

upper epidermis
layer X

lower epidermis

midrib

Which type of cell is found in layer X?

A B C D

286
© UCLES 2018 0610/12/F/M/18
7

11 Which molecule contains magnesium?

A chlorophyll
B fat
C glucose
D starch

12 In which part of the body of a mammal does mechanical digestion occur?

A gall bladder
B liver
C mouth
D pancreas

13 In which order does water pass through these tissues in a plant?

A mesophyll → xylem → root cortex

B root cortex → mesophyll → xylem

C root cortex → xylem → mesophyll

D xylem → mesophyll → root cortex

14 Which part of the blood contains haemoglobin?

A plasma
B platelets
C red blood cells
D white blood cells

15 The body has defences to protect itself from diseases.

What is a mechanical barrier to diseases?

A hairs in the nose


B plasma
C stomach acid
D white blood cells

287
© UCLES 2018 0610/12/F/M/18 [Turn over
8

16 Compared with inspired air, which description of expired air is correct?

A It has less oxygen and less carbon dioxide.


B It has less oxygen and more carbon dioxide.
C It has more oxygen and less carbon dioxide.
D It has more oxygen and more carbon dioxide.

17 An experiment is set up to investigate the uptake of oxygen by germinating seeds.

water bath soda lime absorbs


at 25 °C carbon dioxide
dead seeds rinsed
germinating in antiseptic
seeds
X Y

coloured coloured
water water

What happens to the levels of the coloured water at X and Y?

X Y

A falls rises
B falls unchanged
C rises falls
D rises unchanged

18 Which organ regulates the amount of water in a mammal’s body?

A kidney
B liver
C lungs
D skin

288
© UCLES 2018 0610/12/F/M/18
9

19 A boy accidentally touches a very hot object and immediately takes his hand away.

In this reflex action, what is the effector?

A a heat receptor in his hand


B a motor neurone
C a muscle in his arm
D the spinal cord

20 The graph shows the effect of an antibiotic treatment on bacterial populations in the blood.

start of
antibiotic
treatment

bacterial
population

0 10
time / days

Which conclusion can be drawn from the graph?

A Antibiotics are effective against viral and bacterial infections.


B Antibiotics cause meiosis in bacteria.
C Antibiotics take ten days to kill all bacteria.
D Before the start of antibiotic treatment the bacterial population was rising.

21 Drinking alcohol before driving a car is dangerous.

Which statement correctly explains why drinking alcohol and driving is dangerous?

A Alcohol increases reaction time.


B Alcohol can cause liver damage.
C Alcohol is a drug.
D Alcohol is addictive.

289
© UCLES 2018 0610/12/F/M/18 [Turn over
10

22 Which statement about sexual reproduction is correct?

A It is a process that involves the fusion of two nuclei.


B It always produces genetically identical offspring.
C No zygote is formed.
D There is no fertilisation.

23 The diagram shows a cross-section of a flower.

Where does fertilisation take place?

24 The graph shows changes in the thickness of the lining of the uterus wall during a menstrual
cycle.

thickness
of uterus
wall lining

1 7 14 21 28
days of the menstrual cycle

Which day is the last day of menstruation?

A 4 B 8 C 16 D 20

290
© UCLES 2018 0610/12/F/M/18
11

25 The table shows the percentage of pregnancies in four groups of women. Each group used a
different method of contraception.

percentage of
method of contraception
pregnancies

the contraceptive pill 1


monitoring body temperature 14
diaphragm 7
male sterilisation (vasectomy) 0

Which contraceptive method was the least effective?

A barrier
B chemical
C natural
D surgical

26 Which sex chromosomes in the egg and the sperm will produce a male child?

sex chromosome sex chromosome


in egg in sperm

A X X
B X Y
C Y X
D Y Y

27 Cell division has the following functions.

1 asexual reproduction
2 growth
3 production of gametes
4 repair of damaged tissue
5 replacement of cells

Which functions are specific to mitosis?

A 1, 2, 3 and 4
B 1, 2, 4 and 5
C 2, 3 and 4 only
D 3, 4 and 5 only
291
© UCLES 2018 0610/12/F/M/18 [Turn over
12

28 The chart shows the inheritance of fur colour in a small mammal.

If the allele for white fur is dominant, which animal must be heterozygous for the gene controlling
fur colour?

A B

29 Four phenotypes of a group of people were studied.

Which phenotype will show discontinuous variation?

A blood group
B foot size
C height
D weight

30 Which statement about selective breeding is correct?

A It does not involve humans.


B It involves a struggle for survival.
C It always involves one parent.
D It involves parents that possess desirable features.

292
© UCLES 2018 0610/12/F/M/18
13

31 Which diagram shows how energy flows through an ecosystem?

A B

Sun Sun

C D

Sun Sun

32 The diagram shows part of a food web.

spiders sandpipers mink

mites insects

plants geese

Which members of the food web are examples of producers, herbivores and carnivores?

producers herbivores carnivores

A insects sandpipers mink


B geese mites sandpipers
C plants geese spiders
D plants sandpipers insects

293
© UCLES 2018 0610/12/F/M/18 [Turn over
14

33 The diagram shows part of the carbon cycle.

Which process reduces the carbon dioxide content of the atmosphere?

A B
carbon dioxide
in atmosphere
C

industry
transport D animals plants
and power

fossil fuels (coal and oil) decomposers

34 Which factor will cause a decrease in population size?

A decreased food supply


B decreased predation
C decreased disease
D decreased death rate

35 Some of the characteristics of bacteria are listed.

1 can make complex molecules


2 can reproduce quickly
3 have cytoplasm
4 may cause diseases

Which characteristics make bacteria useful in biotechnology and genetic engineering?

A 1, 2, 3 and 4 B 1 and 2 only C 2 and 3 only D 3 and 4 only

36 What is a product of anaerobic respiration in yeast?

A biological washing powders


B ethanol
C fruit juice
D human insulin

294
© UCLES 2018 0610/12/F/M/18
15

37 Which statement about genetic engineering is correct?

A It involves choosing which individual organisms are used for breeding.


B It is always done using genes from the same species.
C It produces a new combination of genes.
D It produces exact copies of individual organisms.

38 Deforestation has undesirable effects on the environment.

Which effect makes the greatest contribution to global warming?

A extinction of species
B increased carbon dioxide
C increased flooding
D loss of soil by erosion

39 Methane is a gas that contributes to the greenhouse effect.

Which human activity is a source of methane?

A deforestation
B growing rice
C using fertilisers
D using herbicides

40 Which stage in the treatment of sewage removes large floating objects, such as plastic bags?

A aeration
B discharge
C screening
D sedimentation

295
© UCLES 2018 0610/12/F/M/18
16

BLANK PAGE

Permission to reproduce items where third-party owned material protected by copyright is included has been sought and cleared where possible. Every
reasonable effort has been made by the publisher (UCLES) to trace copyright holders, but if any items requiring clearance have unwittingly been included, the
publisher will be pleased to make amends at the earliest possible opportunity.

To avoid the issue of disclosure of answer-related information to candidates, all copyright acknowledgements are reproduced online in the Cambridge
International Examinations Copyright Acknowledgements Booklet. This is produced for each series of examinations and is freely available to download at
www.cie.org.uk after the live examination series.

296
Cambridge International Examinations is part of the Cambridge Assessment Group. Cambridge Assessment is the brand name of University of Cambridge Local
Examinations Syndicate (UCLES), which is itself a department of the University of Cambridge.

© UCLES 2018 0610/12/F/M/18


Cambridge International Examinations
Cambridge International General Certificate of Secondary Education

BIOLOGY 0610/21
Paper 2 Multiple Choice (Extended) October/November 2017
45 minutes
Additional Materials: Multiple Choice Answer Sheet
Soft clean eraser
*6977353268*

Soft pencil (type B or HB is recommended)

READ THESE INSTRUCTIONS FIRST

Write in soft pencil.


Do not use staples, paper clips, glue or correction fluid.
Write your name, Centre number and candidate number on the Answer Sheet in the spaces provided
unless this has been done for you.
DO NOT WRITE IN ANY BARCODES.

There are forty questions on this paper. Answer all questions. For each question there are four possible
answers A, B, C and D.
Choose the one you consider correct and record your choice in soft pencil on the separate Answer Sheet.

Read the instructions on the Answer Sheet very carefully.

Each correct answer will score one mark. A mark will not be deducted for a wrong answer.
Any rough working should be done in this booklet.
Electronic calculators may be used.

The syllabus is approved for use in England, Wales and Northern Ireland as a Cambridge International Level 1/Level 2 Certificate.

This document consists of 15 printed pages and 1 blank page.

IB17 11_0610_21/4RP
© UCLES 2017 [Turn297
over
2

1 Which term is defined as all the chemical reactions that occur in cells?

A photosynthesis
B protein synthesis
C respiration
D metabolism

2 The diagram shows a section of DNA from a chimpanzee.

A G C T A C A G A G

Which diagram shows a section of DNA from the organism that is most closely related to the
chimpanzee?

A A G C T A C A G A T

B A G C T A C A G T T

C A T C A A C A G T T

D A T C T A C A G T T

3 The length of a mitochondrion in a photomicrograph is 15 mm.

The actual length of the mitochondrion is 3 µm.

What is the magnification of the photomicrograph?

A ×5 B ×45 C ×5000 D ×45 000

298
© UCLES 2017 0610/21/O/N/17
3

4 The diagrams show a leaf and its internal structure.

What are the levels of organisation of the labelled structures?

1 2 3

A cell tissue organ system


B organ cell tissue
C organ system tissue cell
D tissue cell organ

5 Different factors affect the rate of diffusion of molecules across a membrane.

Which row represents changes to factors that will increase the rate of diffusion?

concentration
thickness surface area
gradient across temperature
of membrane of membrane
a membrane

A decrease decrease increase increase


B decrease increase increase decrease
C increase decrease increase increase
D increase increase decrease decrease

299
© UCLES 2017 0610/21/O/N/17 [Turn over
4

6 A red blood cell and a palisade mesophyll cell are placed in a solution which has a higher water
potential than the cells.

What will happen to each cell?

palisade
red blood cell
mesophyll cell

A bursts bursts
B bursts gains mass
C loses mass gains mass
D loses mass loses mass

7 The diagram shows a protease molecule catalysing the break down of a protein molecule.

What are the parts labelled P, Q, R and S?

enzyme product substrate active site

A P Q R S
B R S P Q
C S P Q R
D S R Q P

300
© UCLES 2017 0610/21/O/N/17
5

8 The diagram shows the structure of part of a DNA molecule.

What does X represent?

A amino acid
B base
C carbon
D protein

9 Which statement about enzymes is correct?

A Enzymes become part of the product.


B Lowering the pH always slows down the reaction rate.
C Raising the temperature always increases the reaction rate.
D The specificity of an enzyme depends on the shape of its active site.

10 Which enzyme is used to produce clear apple juice?

A amylase
B lipase
C pectinase
D protease

11 Which structure would be found in large numbers in cells that have a high energy requirement?

A chloroplast
B endoplasmic reticulum
C large vacuole
D mitochondrion

301
© UCLES 2017 0610/21/O/N/17 [Turn over
6

12 What must be increased in the diet of a person suffering from constipation?

A fats
B fibre
C iron
D protein

13 Which disease can be caused by a deficiency of iron in the diet?

A anaemia
B kwashiorkor
C marasmus
D rickets

14 The diagram shows the human alimentary canal, with a string marked in metres beside it.

oesophagus
0

3
string marked
in metres
4

9 anus

How long is the small intestine?

A 2m B 6m C 8m D 9m

302
© UCLES 2017 0610/21/O/N/17
7

15 The graph shows the concentration of oxygen in the air spaces of a green leaf of a plant during a
12-hour period.

concentration
of oxygen in
air spaces X

time

Which statement about carbon dioxide in the air spaces during time X is correct?

A Carbon dioxide is being produced because the rate of photosynthesis is greater than the rate
of respiration.
B Carbon dioxide is being produced because the rate of respiration is greater than the rate of
photosynthesis.
C Carbon dioxide is being used because the rate of photosynthesis is greater than the rate of
respiration.
D Carbon dioxide is being used because the rate of respiration is greater than the rate of
photosynthesis.

16 What is a description of transpiration?

A exchange of gases between the leaf and the atmosphere


B loss of water vapour from the leaves and stems of a plant
C movement of water from the roots to the leaves
D movement of water through the cells of the leaf

303
© UCLES 2017 0610/21/O/N/17 [Turn over
8

17 The diagram shows the vessels associated with the flow of fluids to and from body cells.

Which is the lymph vessel?

D B

18 The diagram shows a cross-section through a human blood vessel.

elastic and fibrous


connective tissue

muscle layer

Which type of blood vessel does the diagram show?

A an artery
B a capillary
C a vein
D a ventricle

19 Which are both chemical barriers to the transmission of pathogens?

A mucus and stomach acid


B mucus and white blood cells
C skin and hairs in the nose
D skin and stomach acid

304
© UCLES 2017 0610/21/O/N/17
9

20 The table shows some of the changes that occur during breathing.

from contracted from relaxed


to relaxed to contracted

diaphragm P X
external intercostals Q Y
internal intercostals R Z

Which changes occur to cause inspiration?

A P, Q and Z B X, Q and R C X, Y and R D X, Y and Z

21 In an experiment to investigate anaerobic respiration, two bottles are set up in a warm room, as
shown.

P Q
balloon balloon

sugar, yeast yeast


and water and water

What would happen to each balloon after one day?

A B

P Q P Q

C D

P Q P Q

305
© UCLES 2017 0610/21/O/N/17 [Turn over
10

22 The graph shows the lactic acid concentration in blood during and after exercise.

1.0
Z

blood lactic acid


concentration 0.5
/ mg dm–3

exercise recovery
0
0 5 10 15 20 25 30
time / minutes

The continuation of which process accounts for the shape of the graph at Z?

A deep breathing
B high heart rate
C high rate of breathing
D movement of lactic acid from the muscles

23 What is the most important function of sweating?

A to remove excess heat from the body


B to remove excess salts from the body
C to remove excess urea from the body
D to remove excess water from the body

24 Four effects of a specific hormone are listed.

● increased blood pressure


● increased blood glucose concentration
● increased rate of respiration
● reduced blood flow to the gut

What is this hormone?

A adrenaline
B glucagon
C insulin
D testosterone

306
© UCLES 2017 0610/21/O/N/17
11

25 The diagram shows a person sweating in hot weather.

What part is played by sweat glands during the process of sweating?

A effector
B receptor
C sense organ
D stimulus

26 The diagram shows a synapse in a reflex arc.

P Q

What are the identities of the two neurones and in which direction does the neurotransmitter
pass?

direction of passage
neurone P neurone Q
of neurotransmitter

A motor relay P→Q


B motor sensory P→Q
C relay motor Q→P
D relay sensory Q→P

307
© UCLES 2017 0610/21/O/N/17 [Turn over
12

27 The immune system recognises pathogens and attacks them.

Which feature of pathogens triggers this response?

A antibodies
B antibiotics
C antigens
D memory cells

28 Which environmental factor is not always a requirement for seed germination?

A light
B oxygen
C suitable temperature
D water

29 Which row shows the adaptive features of sperm cells?

flagellum jelly coat mitochondria

A absent absent present


B absent present absent
C present absent present
D present present absent

30 In some mammals the allele for brown coat colour is dominant to the allele for white coat colour.

Which percentage of offspring will be white if a cross is made between two heterozygous
mammals?

A 0% B 25% C 50% D 100%

31 Which term is defined as a length of DNA that codes for a protein?

A amino acid
B chromosome
C gene
D mutation

308
© UCLES 2017 0610/21/O/N/17
13

32 The following are involved in protein synthesis.

1 amino acids assembled in order


2 mRNA moves to the cytoplasm
3 mRNA passing through a ribosome
4 DNA in the nucleus

In which order do they become involved when proteins are made?

A 1→3→2→4

B 3→2→1→4

C 4→2→3→1

D 4→3→2→1

33 What makes tongue rolling an example of discontinuous variation?

A A person can roll their tongue only when they are young.
B There are many different types of tongue rollers.
C Tongue rolling has to be learnt.
D Tongue rolling is something that a person either can or cannot do.

34 The distribution of the sickle-cell allele in human populations varies in different areas of the world.

What is an explanation for this difference?

A People that are heterozygous for the sickle-cell allele have a resistance to cholera.
B People that are heterozygous for the sickle-cell allele have a resistance to malaria.
C People that are heterozygous for the sickle-cell allele are more likely to suffer from anaemia.
D People with sickle-cell anaemia have more alleles.

35 Nitrogen in the air cannot be used by plants until it is in the form of nitrates.

Which two processes convert nitrogen from the air into nitrates?

A decomposition of faeces and nitrification


B denitrification and lightning
C nitrogen fixation by bacteria and denitrification
D nitrogen fixation by bacteria and lightning

309
© UCLES 2017 0610/21/O/N/17 [Turn over
14

36 What is shown by the widest block in a pyramid of numbers for a grassland ecosystem?

A all the consumers in the pyramid


B the carnivores in the pyramid
C the organisms at the top of the pyramid
D the producers in the pyramid

37 The diagrams show the depth and rate of breathing in a person at 12.00 pm and 1.00 pm.

12.00 pm 1.00 pm

0.5 dm3
volume
volume

0 10 20 30 40 50 60 70 0 10 20 30 40 50 60 70
time / seconds time / seconds

What happens to the person’s breathing between 12.00 pm and 1.00 pm?

depth rate
of breathing of breathing

A decreases decreases
B decreases increases
C increases decreases
D increases increases

38 Ligase enzymes are used in genetic engineering to

A cut open plasmid DNA.


B insert plasmids into bacteria.
C isolate the DNA making up a human gene.
D join human DNA to plasmid DNA.

310
© UCLES 2017 0610/21/O/N/17
15

39 What is not a reason for conservation programmes?

A to introduce new species


B to maintain nutrient cycles
C to maintain resources
D to protect vulnerable environments

40 Some examples of the waste products of human activity are discarded household rubbish,
excess fertiliser, industrial chemicals and untreated sewage.

Which of these can both cause increased growth of aquatic plants?

A chemical waste and discarded household rubbish


B discarded household rubbish and excess fertiliser
C excess fertiliser and untreated sewage
D untreated sewage and chemical waste

311
© UCLES 2017 0610/21/O/N/17
16

BLANK PAGE

Permission to reproduce items where third-party owned material protected by copyright is included has been sought and cleared where possible. Every
reasonable effort has been made by the publisher (UCLES) to trace copyright holders, but if any items requiring clearance have unwittingly been included, the
publisher will be pleased to make amends at the earliest possible opportunity.

To avoid the issue of disclosure of answer-related information to candidates, all copyright acknowledgements are reproduced online in the Cambridge
International Examinations Copyright Acknowledgements Booklet. This is produced for each series of examinations and is freely available to download at
www.cie.org.uk after the live examination series.

312
Cambridge International Examinations is part of the Cambridge Assessment Group. Cambridge Assessment is the brand name of University of Cambridge Local
Examinations Syndicate (UCLES), which is itself a department of the University of Cambridge.

© UCLES 2017 0610/21/O/N/17


Cambridge International Examinations
Cambridge International General Certificate of Secondary Education

BIOLOGY 0610/22
Paper 2 Multiple Choice (Extended) October/November 2017
45 minutes
Additional Materials: Multiple Choice Answer Sheet
Soft clean eraser
*3623538707*

Soft pencil (type B or HB is recommended)

READ THESE INSTRUCTIONS FIRST

Write in soft pencil.


Do not use staples, paper clips, glue or correction fluid.
Write your name, Centre number and candidate number on the Answer Sheet in the spaces provided
unless this has been done for you.
DO NOT WRITE IN ANY BARCODES.

There are forty questions on this paper. Answer all questions. For each question there are four possible
answers A, B, C and D.
Choose the one you consider correct and record your choice in soft pencil on the separate Answer Sheet.

Read the instructions on the Answer Sheet very carefully.

Each correct answer will score one mark. A mark will not be deducted for a wrong answer.
Any rough working should be done in this booklet.
Electronic calculators may be used.

The syllabus is approved for use in England, Wales and Northern Ireland as a Cambridge International Level 1/Level 2 Certificate.

This document consists of 14 printed pages and 2 blank pages.

IB17 11_0610_22/5RP
© UCLES 2017 [Turn313
over
2

1 To which group do both the organisms shown in the diagram belong?

leaf-like fronds which have


structures containing spores
on their lower surface

A dicotyledons
B ferns
C fungi
D monocotyledons

2 The diagram shows a section of DNA from a chimpanzee.

A G C T A C A G A G

Which diagram shows a section of DNA from the organism that is most closely related to the
chimpanzee?

A A G C T A C A G A T

B A G C T A C A G T T

C A T C A A C A G T T

D A T C T A C A G T T

314
© UCLES 2017 0610/22/O/N/17
3

3 What is a leaf?

A a cell
B an organ
C an organ system
D a tissue

4 The diagram shows a magnified image of a human liver cell with a mitochondrion labelled. The
actual size of the liver cell is 20 µm.

The image size of the liver cell is 40 mm and the image size of the mitochondrion is 4 mm.

mitochondrion

20 µm

What is the actual size of the mitochondrion shown in the diagram?

A 0.002 mm B 0.02 mm C 0.2 mm D 2 mm

5 What would increase the rate of diffusion of oxygen into an animal cell?

A decreasing the concentration gradient between the inside of the cell and the outside
B decreasing the temperature of the cell and its surroundings
C increasing the distance that the oxygen molecules have to travel
D increasing the surface area of the cell membrane

315
© UCLES 2017 0610/22/O/N/17 [Turn over
4

6 A red blood cell and a palisade mesophyll cell are placed in a solution which has a higher water
potential than the cells.

What will happen to each cell?

palisade
red blood cell
mesophyll cell

A bursts bursts
B bursts gains mass
C loses mass gains mass
D loses mass loses mass

7 Where in the alimentary canal is the enzyme trypsin found and what are the products of the
reaction it catalyses?

where trypsin
products
is found

A duodenum amino acids


B duodenum fatty acids
C ileum proteins
D stomach amino acids

8 The diagram shows a cell before and during mitosis.

At which stage are the chromosomes copied?

B
A

316
© UCLES 2017 0610/22/O/N/17
5

9 Why does excessive heat decrease enzyme activity?

A It changes the shape of the active site.


B It changes the shape of the substrate and product molecules.
C It increases the force of collisions between substrate and product molecules.
D It increases the kinetic energy of the substrate molecules.

10 Four test-tubes were set up as shown in the table.

In which test-tube would starch be broken down the fastest?

2 cm3 starch
1 cm3 of 1 cm3 of boiled temperature
suspension
amylase added amylase added / °C
added

A    5 key
B    35 = present
C    5 = absent
D    35

11 The average number of chloroplasts in four different types of cell taken from a plant is shown.

Which is a root hair cell?

A 0 B 47 C 370 D 920

12 What must be increased in the diet of a person suffering from constipation?

A fats
B fibre
C iron
D protein

13 Which stage of nutrition takes place when food molecules become part of a body cell?

A absorption
B assimilation
C digestion
D ingestion

317
© UCLES 2017 0610/22/O/N/17 [Turn over
6

14 The diagram shows the human alimentary canal, with a string marked in metres beside it.

oesophagus
0

3
string marked
in metres
4

9 anus

How long is the small intestine?

A 2m B 6m C 8m D 9m

15 What is the function of translocation?

A to move leaves towards the light for photosynthesis


B to move water into leaves for photosynthesis
C to transport amino acids for the growth of new leaves
D to transport starch to all parts of a plant

16 What is a description of transpiration?

A exchange of gases between the leaf and the atmosphere


B loss of water vapour from the leaves and stems of a plant
C movement of water from the roots to the leaves
D movement of water through the cells of the leaf

318
© UCLES 2017 0610/22/O/N/17
7

17 What happens as the blood flows from the atria into the ventricles of the heart?

atrioventricular muscle wall muscle wall semi-lunar


valves of the atria of the ventricles valves

A close relax contract open


B close relax relax open
C open contract contract close
D open contract relax close

18 The diagram shows a cross-section through a human blood vessel.

elastic and fibrous


connective tissue

muscle layer

Which type of blood vessel does the diagram show?

A an artery
B a capillary
C a vein
D a ventricle

19 Which is a mechanical barrier to pathogens?

A acid in the stomach


B hairs in the nose
C mucus in the trachea
D phagocytosis in the blood

319
© UCLES 2017 0610/22/O/N/17 [Turn over
8

20 The table shows some of the changes that occur during breathing.

from contracted from relaxed


to relaxed to contracted

diaphragm P X
external intercostals Q Y
internal intercostals R Z

Which changes occur to cause inspiration?

A P, Q and Z B X, Q and R C X, Y and R D X, Y and Z

21 In an experiment to investigate anaerobic respiration, two bottles are set up in a warm room, as
shown.

P Q
balloon balloon

sugar, yeast yeast


and water and water

What would happen to each balloon after one day?

A B

P Q P Q

C D

P Q P Q

320
© UCLES 2017 0610/22/O/N/17
9

22 Vigorous exercise can cause an oxygen debt.

Which process removes the oxygen debt?

A aerobic respiration of lactic acid in the liver


B a decrease in breathing rate
C a decrease in heart rate
D an increase in blood supply to the skin

23 What is the most important function of sweating?

A to remove excess heat from the body


B to remove excess salts from the body
C to remove excess urea from the body
D to remove excess water from the body

24 Which row shows the function of rod cells?

have greater
give colour vision
sensitivity to light

A  
B  
C  
D  

321
© UCLES 2017 0610/22/O/N/17 [Turn over
10

25 The diagram shows a person sweating in hot weather.

What part is played by sweat glands during the process of sweating?

A effector
B receptor
C sense organ
D stimulus

26 The diagram shows a synapse in a reflex arc.

P Q

What are the identities of the two neurones and in which direction does the neurotransmitter
pass?

direction of passage
neurone P neurone Q
of neurotransmitter

A motor relay P→Q


B motor sensory P→Q
C relay motor Q→P
D relay sensory Q→P

322
© UCLES 2017 0610/22/O/N/17
11

27 The graph shows the number of deaths from MRSA infection per 100 000 people in a population
of over-65-year-olds from 2005 to 2012.

13

12

number of deaths
11
from MRSA infection
in over-65-year-olds
per 100 000 people 10

8
2005 2006 2007 2008 2009 2010 2011 2012
year

What is a possible explanation for the changes shown in the graph?

A a decrease in the size of the population of over-65-year-olds


B antibiotics do not affect viruses
C more effective antibiotics are being used to treat infected people
D over-65-year-olds are immune to MRSA infection

28 After fertilisation, how many chromosomes are in the zygote?

A half as many as in an ovum


B the same as in an ovum
C the same as in a sperm
D twice as many as in a sperm

29 Which environmental factor is not always a requirement for seed germination?

A light
B oxygen
C suitable temperature
D water

323
© UCLES 2017 0610/22/O/N/17 [Turn over
12

30 In some plants, H is the dominant allele for hairy stems and h is the recessive allele for smooth
stems.

A pair of these plants produce 37 offspring, 18 with hairy stems and 19 with smooth stems.

What are the most likely genotypes of the parents?

A HH × HH B Hh × Hh C Hh × hh D hh × hh

31 The family tree shows the inheritance of the ability to smell flowers called freesias. The allele for
the ability to smell freesias is dominant.

Which individual’s symbol is not correct?

key
A female able to
smell freesias
female unable to
smell freesias
male able to
smell freesias
male unable to
D C B
smell freesias

32 The diagram shows a cell from an organism at the end of meiosis.

What is the diploid number for cells from this organism?

A 3 B 6 C 12 D 24

33 When growing millet, farmers choose seeds from high-yielding plants.

Which biological practice is this an example of?

A adaptation
B artificial selection
C evolution
D natural selection

324
© UCLES 2017 0610/22/O/N/17
13

34 Which graph shows the growth of a population where there are no limiting factors?

A B

population population
size size

0 0
0 time 0 time

C D

population population
size size

0 0
0 time 0 time

35 What is defined as ‘all of the populations of different species in an ecosystem’?

A community
B environment
C habitat
D trophic level

36 Which pyramid of numbers has more herbivores than producers?

A B C D

325
© UCLES 2017 0610/22/O/N/17 [Turn over
14

37 Why are bacteria useful in biotechnology and genetic engineering?

A Bacteria do not have cell vacuoles.


B Bacteria do not have mitochondria.
C Bacteria have cell walls.
D Bacteria share their genetic code with all other organisms.

38 Ligase enzymes are used in genetic engineering to

A cut open plasmid DNA.


B insert plasmids into bacteria.
C isolate the DNA making up a human gene.
D join human DNA to plasmid DNA.

39 The action of which type of bacteria would cause soil to be lacking in nitrates?

A aerobic
B denitrifying
C nitrifying
D nitrogen fixing

40 What is used to help sustain fish stocks?

education legal quotas eating more fish restocking

A    
B    
C    
D    

326
© UCLES 2017 0610/22/O/N/17
Cambridge International Examinations
Cambridge International General Certificate of Secondary Education

BIOLOGY 0610/23
Paper 2 Multiple Choice (Extended) October/November 2017
45 minutes
Additional Materials: Multiple Choice Answer Sheet
Soft clean eraser
*0288685329*

Soft pencil (type B or HB is recommended)

READ THESE INSTRUCTIONS FIRST

Write in soft pencil.


Do not use staples, paper clips, glue or correction fluid.
Write your name, Centre number and candidate number on the Answer Sheet in the spaces provided
unless this has been done for you.
DO NOT WRITE IN ANY BARCODES.

There are forty questions on this paper. Answer all questions. For each question there are four possible
answers A, B, C and D.
Choose the one you consider correct and record your choice in soft pencil on the separate Answer Sheet.

Read the instructions on the Answer Sheet very carefully.

Each correct answer will score one mark. A mark will not be deducted for a wrong answer.
Any rough working should be done in this booklet.
Electronic calculators may be used.

The syllabus is approved for use in England, Wales and Northern Ireland as a Cambridge International Level 1/Level 2 Certificate.

This document consists of 17 printed pages and 3 blank pages.

IB17 11_0610_23/3RP
© UCLES 2017 [Turn327
over
2

1 The plant Mimosa pudica has leaves that fold in when touched.

This demonstrates movement and which other characteristic?

A excretion
B growth
C nutrition
D sensitivity

2 The diagram shows a section of DNA from a chimpanzee.

A G C T A C A G A G

Which diagram shows a section of DNA from the organism that is most closely related to the
chimpanzee?

A A G C T A C A G A T

B A G C T A C A G T T

C A T C A A C A G T T

D A T C T A C A G T T

328
© UCLES 2017 0610/23/O/N/17
3

3 The diagrams show structures associated with a leaf.

thick cellulose wall

nucleus

vacuole

cytoplasm
leaf leaf stalk detail

What is the level of organisation of the part shown in detail?

A cell
B organ
C organ system
D tissue

4 The diagram shows a pollen grain of a rice plant. The size of the image is 40 mm

40 µm

The actual length of the pollen grain is 40 µm.

By how many times has the diagram been magnified?

A ×1 B ×10 C ×100 D ×1000

329
© UCLES 2017 0610/23/O/N/17 [Turn over
4

5 Three equally sized pieces of potato were put into different concentrations of sucrose solution.
One piece of potato was put into distilled water.

The concentrations of sucrose solution were 0.2 g dm–3, 0.4 g dm–3 and 0.6 g dm–3.

The graph shows the change in mass of the potato pieces over a period of 60 minutes.

Which piece of potato was put into distilled water?

2
A
change in mass of 1 B
the potato pieces / g
0 C
0 time / minutes 60
–1 D

6 A red blood cell and a palisade mesophyll cell are placed in a solution which has a higher water
potential than the cells.

What will happen to each cell?

palisade
red blood cell
mesophyll cell

A bursts bursts
B bursts gains mass
C loses mass gains mass
D loses mass loses mass

330
© UCLES 2017 0610/23/O/N/17
5

7 The diagram shows an evolutionary tree based on a computer-generated study of different sea
mammals.

fin whale

humpback whale

sperm whale

beaked whale
common ancestor
dolphin

porpoise

According to this evolutionary tree, which whale has DNA base sequences that are least shared
with the other whales?

A beaked whale
B fin whale
C humpback whale
D sperm whale

8 What is the colour change shown by Benedict’s solution when heated with a reducing sugar?

A blue to purple
B blue to red
C brown to blue-black
D red to yellow

331
© UCLES 2017 0610/23/O/N/17 [Turn over
6

9 Into which part of the alimentary canal is the enzyme that digests starch secreted?

10 The diagram shows an enzyme with its substrate and product molecules.

Q
P

Which form an enzyme-substrate complex?

A P and Q B Q and R C R and S D S and P

332
© UCLES 2017 0610/23/O/N/17
7

11 The diagram shows the apparatus at the beginning of an investigation into temperature change
during the germination of seeds. The temperature at the start of the investigation was 25 °C in
both flasks.

After two days the temperature in flask 1 is 25 °C. The temperature in flask 2 is 28 °C.

vacuum
flask

dead and living seeds


disinfected
seeds

cotton wool

thermometer

flask 1 flask 2

Which characteristic of living organisms is shown in this experiment?

A excretion
B growth
C reproduction
D respiration

12 What must be increased in the diet of a person suffering from constipation?

A fats
B fibre
C iron
D protein

333
© UCLES 2017 0610/23/O/N/17 [Turn over
8

13 Which substrate, enzyme and product are correctly named?

substrate enzyme product

A amino acids trypsin protein


B amylase maltose glucose
C lipase lipid fatty acids and glycerol
D maltose maltase glucose

14 The diagram shows the human alimentary canal, with a string marked in metres beside it.

oesophagus
0

3
string marked
in metres
4

9 anus

How long is the small intestine?

A 2m B 6m C 8m D 9m

334
© UCLES 2017 0610/23/O/N/17
9

15 What is not a use of water by plants?

A acting as a solvent
B cooling the plant
C dissolving cellulose cell walls
D raw material in photosynthesis

16 What is a description of transpiration?

A exchange of gases between the leaf and the atmosphere


B loss of water vapour from the leaves and stems of a plant
C movement of water from the roots to the leaves
D movement of water through the cells of the leaf

17 The diagram represents the circulatory system of a mammal.

In which chamber of the heart are the muscle walls thickest?

rest of
lungs A B body

D C

heart

335
© UCLES 2017 0610/23/O/N/17 [Turn over
10

18 The diagram shows a cross-section through a human blood vessel.

elastic and fibrous


connective tissue

muscle layer

Which type of blood vessel does the diagram show?

A an artery
B a capillary
C a vein
D a ventricle

19 What can be passed from one person to another during blood transfusion?

A cholera
B chronic obstructive pulmonary disease (COPD)
C HIV
D scurvy

20 The table shows some of the changes that occur during breathing.

from contracted from relaxed


to relaxed to contracted

diaphragm P X
external intercostals Q Y
internal intercostals R Z

Which changes occur to cause inspiration?

A P, Q and Z B X, Q and R C X, Y and R D X, Y and Z

336
© UCLES 2017 0610/23/O/N/17
11

21 In an experiment to investigate anaerobic respiration, two bottles are set up in a warm room, as
shown.

P Q
balloon balloon

sugar, yeast yeast


and water and water

What would happen to each balloon after one day?

A B

P Q P Q

C D

P Q P Q

337
© UCLES 2017 0610/23/O/N/17 [Turn over
12

22 Two pieces of an aquatic plant were placed into two different test-tubes, P and Q.

Each test-tube contained hydrogencarbonate indicator and was sealed and kept at 20 °C.

Test-tube P was kept in the light and test-tube Q was kept in the dark.

The table shows the effect of carbon dioxide on the colour of the hydrogencarbonate indicator.

less carbon more carbon


dioxide dioxide

dark red orange

What would the colour of the indicator be after 12 hours?

P Q

A dark red orange


B dark red dark red
C orange dark red
D orange orange

23 What is the most important function of sweating?

A to remove excess heat from the body


B to remove excess salts from the body
C to remove excess urea from the body
D to remove excess water from the body

24 A student begins to lose control of her bicycle while travelling down a hill at speed.

The concentration of which substance will begin to increase rapidly in her blood?

A adrenaline
B insulin
C oestrogen
D testosterone

338
© UCLES 2017 0610/23/O/N/17
13

25 The diagram shows a person sweating in hot weather.

What part is played by sweat glands during the process of sweating?

A effector
B receptor
C sense organ
D stimulus

26 The diagram shows a synapse in a reflex arc.

P Q

What are the identities of the two neurones and in which direction does the neurotransmitter
pass?

direction of passage
neurone P neurone Q
of neurotransmitter

A motor relay P→Q


B motor sensory P→Q
C relay motor Q→P
D relay sensory Q→P

27 Which process occurring at a synapse is prevented by the presence of heroin?

A the binding of a neurotransmitter with receptors


B the diffusion of a neurotransmitter across the gap
C the formation of a neurotransmitter
D the stimulation of vesicles by an impulse

339
© UCLES 2017 0610/23/O/N/17 [Turn over
14

28 The diagram shows half a flower.

Q
R

After pollination, where would pollen grains be found?

A P and Q B Q and R C R and S D S and P

29 Which environmental factor is not always a requirement for seed germination?

A light
B oxygen
C suitable temperature
D water

30 A pure-breeding white rat was crossed with a pure-breeding black rat. All their offspring were
black.

One of the offspring was bred with a pure-breeding white rat.

What is the most likely percentage of black rats in the offspring?

A 25 B 50 C 75 D 100

31 An organism is heterozygous for a gene with the alleles T and t.

Which diagram represents a diploid cell from this organism?

A B C D

T t T T t T

340
© UCLES 2017 0610/23/O/N/17
15

32 Red-green colour blindness is a sex-linked characteristic caused by a recessive allele.

Which prediction can be made about the children of a woman who is colour-blind and a man with
normal vision?

A Boys will be colour-blind, girls will have a 50% chance of being colour-blind.
B Boys will be colour-blind, girls will have normal vision.
C Girls will be colour-blind, boys will have a 50% chance of being colour-blind.
D Girls will be colour-blind, boys will have normal vision.

33 The diagram shows a vertical section through a leaf.

upper epidermis

sub-stomatal
air chamber

spongy
mesophyll cell

lower epidermis

Which diagram shows the plant that the leaf was taken from?

A B C D

water

34 What is a mutation?

A a change in appearance
B a change in a gene
C a change in behaviour
D a change in the environment

341
© UCLES 2017 0610/23/O/N/17 [Turn over
16

35 The biomass at each trophic level in an ecosystem is measured. The results are shown in the
table.

Which trophic level contains herbivores?

trophic
mass / g m–3
level

A 0.1
B 0.6
C 1.2
D 17.9

36 The diagram shows a food web.

plants herbivores carnivores

decomposers

What do the arrows represent?

A the absorption of oxygen


B the absorption of water
C the flow of energy
D the release of carbon dioxide

37 Ligase enzymes are used in genetic engineering to

A cut open plasmid DNA.


B insert plasmids into bacteria.
C isolate the DNA making up a human gene.
D join human DNA to plasmid DNA.

38 With which kingdoms do bacteria share the same genetic code?

A animal, plant, fungus and protoctist


B animal, plant and fungus only
C animal and plant only
D animal only

342
© UCLES 2017 0610/23/O/N/17
17

39 What is a direct result of deforestation?

A decreased leaching of mineral salts


B increased loss of soil
C increased production of methane
D increased recycling of important minerals

40 The table shows the ability of three species of fish and their eggs to survive in water at different
pH levels.

If the eggs do not survive offspring cannot be produced.

pH
6.5 6.0 5.5 5.0 4.5 4.0

trout       key
sea bass       = survive
perch       = do not survive
fish eggs      

A lake at pH 6.0 contains breeding populations of all three fish.

If acid rain causes the pH to fall to 5.0, which outcome would be likely to occur?

A Trout and perch will survive and produce offspring.


B Trout and perch will survive but only perch will produce offspring.
C Trout and perch will survive but produce no offspring.
D Trout, sea bass and perch will survive but produce no offspring.

343
© UCLES 2017 0610/23/O/N/17
Cambridge International Examinations
Cambridge International General Certificate of Secondary Education

BIOLOGY 0610/11
Paper 1 Multiple Choice (Core) October/November 2017
45 minutes
Additional Materials: Multiple Choice Answer Sheet
Soft clean eraser
*0846476635*

Soft pencil (type B or HB is recommended)

READ THESE INSTRUCTIONS FIRST

Write in soft pencil.


Do not use staples, paper clips, glue or correction fluid.
Write your name, Centre number and candidate number on the Answer Sheet in the spaces provided
unless this has been done for you.
DO NOT WRITE IN ANY BARCODES.

There are forty questions on this paper. Answer all questions. For each question there are four possible
answers A, B, C and D.
Choose the one you consider correct and record your choice in soft pencil on the separate Answer Sheet.

Read the instructions on the Answer Sheet very carefully.

Each correct answer will score one mark. A mark will not be deducted for a wrong answer.
Any rough working should be done in this booklet.
Electronic calculators may be used.

The syllabus is approved for use in England, Wales and Northern Ireland as a Cambridge International Level 1/Level 2 Certificate.

This document consists of 15 printed pages and 1 blank page.

IB17 11_0610_11/3RP
© UCLES 2017 [Turn344
over
2

1 Which process in plant cells uses chlorophyll?

A growth
B nutrition
C movement
D respiration

2 The table shows the scientific names of four members of the cat family.

common name scientific name

leopard Panthera pardus


lion Panthera leo
ocelot Leopardus pardalis
tiger Panthera tigris

Which statement is correct?

A All four cats are members of the same species.


B The leopard and the ocelot are members of the same genus.
C The leopard, lion and tiger are members of the same genus.
D The leopard, lion and tiger are members of the same species.

3 The diagram shows a type of tooth.

crown

root

Use the key to identify the tooth.

1 the root is divided into two parts ............................................................... go to 2


the root is not divided into two parts ......................................................... go to 3
2 the height of the crown is greater than the length of the root ................... A
the height of the crown is less than the length of the root ........................ B
3 flattened crown ......................................................................................... C
ridged crown ............................................................................................. D

345
© UCLES 2017 0610/11/O/N/17
3

4 The diagram shows a student’s drawing of guard cells.

Which label is not correct?

A cell membrane

B cytoplasm

C vacuole

D cell wall

5 The diagrams show a leaf and its internal structure.

What are the levels of organisation of the labelled structures?

1 2 3

A cell tissue organ system


B organ cell tissue
C organ system tissue cell
D tissue cell organ

346
© UCLES 2017 0610/11/O/N/17 [Turn over
4

6 The diagram shows an insect as seen using the low power lens of a microscope.

The actual diameter of the circle is 0.3 cm.

What is the approximate size of this insect in millimetres?

A 0.1 mm B 1.0 mm C 2.0 mm D 3.0 mm

7 The diagram shows part of a section through a leaf.

Which arrow represents the diffusion of oxygen during photosynthesis?

B A
leaf cells
xylem

8 What helps to support plants?

A pressure inwards on the cell vacuoles


B pressure inwards on the chloroplasts
C pressure outwards on the cell walls
D pressure outwards on the nuclei

347
© UCLES 2017 0610/11/O/N/17
5

9 Which of these is digested by protease?

A key
B amino acid
glucose
C
chemical bond
D

10 Which enzyme is used to produce clear apple juice?

A amylase
B lipase
C pectinase
D protease

11 What is the optimum pH for stomach enzymes?

A pH 2 B pH 7 C pH 9 D pH 12

12 What are the raw materials necessary for photosynthesis?

A carbon dioxide and water


B light and a suitable temperature
C oxygen and carbon dioxide
D water and a suitable temperature

13 In plants, which substance contains magnesium ions?

A cellulose
B chlorophyll
C haemoglobin
D starch

14 What must be increased in the diet of a person suffering from constipation?

A fats
B fibre
C iron
D protein

348
© UCLES 2017 0610/11/O/N/17 [Turn over
6

15 Which chemical reaction takes place in the mouth?

A Fats are digested into fatty acids and glycerol.


B Fats are digested into simpler sugars.
C Starch is digested into simpler sugars.
D Starch is digested into amino acids.

16 The diagram shows the human alimentary canal, with a string marked in metres beside it.

oesophagus
0

3
string marked
in metres
4

9 anus

How long is the small intestine?

A 2m B 6m C 8m D 9m

17 What is a description of transpiration?

A exchange of gases between the leaf and the atmosphere


B loss of water vapour from the leaves and stems of a plant
C movement of water from the roots to the leaves
D movement of water through the cells of the leaf

349
© UCLES 2017 0610/11/O/N/17
7

18 Which tissue transports water from the roots to the leaves in a plant?

A cortex
B epidermis
C mesophyll
D xylem

19 On which organ is an ECG performed?

A brain
B colon
C ear
D heart

20 The diagram shows a cross-section through a human blood vessel.

elastic and fibrous


connective tissue

muscle layer

Which type of blood vessel does the diagram show?

A an artery
B a capillary
C a vein
D a ventricle

21 Which are both chemical barriers to the transmission of pathogens?

A mucus and stomach acid


B mucus and white blood cells
C skin and hairs in the nose
D skin and stomach acid

350
© UCLES 2017 0610/11/O/N/17 [Turn over
8

22 The table shows the approximate composition of inspired and expired air.

percentage of gas percentage of gas


gas
in inspired air in expired air

P 78.10 78.10
Q 20.90 16.00
R variable variable
S 0.04 4.00

What is the name of gas S?

A carbon dioxide
B nitrogen
C oxygen
D water vapour

351
© UCLES 2017 0610/11/O/N/17
9

23 In an experiment to investigate anaerobic respiration, two bottles are set up in a warm room, as
shown.

P Q
balloon balloon

sugar, yeast yeast


and water and water

What would happen to each balloon after one day?

A B

P Q P Q

C D

P Q P Q

352
© UCLES 2017 0610/11/O/N/17 [Turn over
10

24 The diagram represents the exchange of gases during breathing and during respiration in the
body.

oxygen
oxygen in blood

X Y

carbon
carbon dioxide
dioxide
in blood

What is represented by X and by Y?

X Y

A lungs air
B lungs body cells
C body cells air
D body cells lungs

25 The table shows information about urea.

Which row is correct?

substance that organ that organ that


urea is made from makes urea excretes urea

A amino acids kidney bladder


B amino acids liver kidney
C fatty acids kidney bladder
D fatty acids liver kidney

26 What is the most important function of sweating?

A to remove excess heat from the body


B to remove excess salts from the body
C to remove excess urea from the body
D to remove excess water from the body

353
© UCLES 2017 0610/11/O/N/17
11

27 What shows the order in which these structures are involved in a reflex action?

A effector → motor neurone → relay neurone → sensory neurone → receptor

B effector → sensory neurone → motor neurone → relay neurone→ receptor

C receptor → sensory neurone → relay neurone → motor neurone → effector

D receptor → motor neurone → sensory neurone → relay neurone→ effector

28 The diagram shows a person sweating in hot weather.

What part is played by sweat glands during the process of sweating?

A effector
B receptor
C sense organ
D stimulus

29 The diagram shows the gas exchange surface of a person who has just smoked a cigarette.

Substance P can cause cancer.

P
moisture
alveolus wall

What is substance P?

A carbon dioxide
B carbon monoxide
C nicotine
D tar

354
© UCLES 2017 0610/11/O/N/17 [Turn over
12

30 Which method of birth control works by preventing an egg from being released?

A condom
B contraceptive pill
C monitoring body temperature
D vasectomy

31 Which environmental factor is not always a requirement for seed germination?

A light
B oxygen
C suitable temperature
D water

32 Four processes involved in labour and birth are shown.

1 cutting the umbilical cord

2 contraction of muscles in the uterus wall

3 dilation of the cervix

4 passage of the baby through the vagina

In which sequence do these events normally occur?

A 2→3→1→4

B 2→3→4→1

C 3→2→1→4

D 3→2→4→1

33 In some mammals the allele for brown coat colour is dominant to the allele for white coat colour.

Which percentage of offspring will be white if a cross is made between two heterozygous
mammals?

A 0% B 25% C 50% D 100%

355
© UCLES 2017 0610/11/O/N/17
13

34 Which term is defined as a length of DNA that codes for a protein?

A amino acid
B chromosome
C gene
D mutation

35 What makes tongue rolling an example of discontinuous variation?

A A person can roll their tongue only when they are young.
B There are many different types of tongue rollers.
C Tongue rolling has to be learnt.
D Tongue rolling is something that a person either can or cannot do.

36 What is shown by the widest block in a pyramid of numbers for a grassland ecosystem?

A all the consumers in the pyramid


B the carnivores in the pyramid
C the organisms at the top of the pyramid
D the producers in the pyramid

37 A food chain is shown.

potato plant → slug → hedgehog

By which process is energy transferred from the potato plant to the slug?

A egestion
B excretion
C ingestion
D photosynthesis

356
© UCLES 2017 0610/11/O/N/17 [Turn over
14

38 A farmer put some fertiliser on his field. Some of the fertiliser drained into a nearby lake.

land with fertiliser

lake
plants

What is the effect of the fertiliser on the growth of the crop plants in the field and the plants in the
lake?

crop plant lake plant


growth growth

A decreased decreased
B decreased increased
C increased decreased
D increased increased

39 The diagrams show the depth and rate of breathing in a person at 12.00 pm and 1.00 pm.

12.00 pm 1.00 pm

0.5 dm3
volume
volume

0 10 20 30 40 50 60 70 0 10 20 30 40 50 60 70
time / seconds time / seconds

What happens to the person’s breathing between 12.00 pm and 1.00 pm?

depth rate
of breathing of breathing

A decreases decreases
B decreases increases
C increases decreases
D increases increases

357
© UCLES 2017 0610/11/O/N/17
15

40 The table shows the amount of carbon dioxide in the atmosphere in three different years.

year 1930 1980 1990


carbon dioxide / parts per million 300 330 370

What is the most likely cause of this change?

A destruction of rainforests
B increased use of fertilisers containing nitrogen
C pollution of air by sulfur dioxide
D rise in the sea level

358
© UCLES 2017 0610/11/O/N/17
Cambridge International Examinations
Cambridge International General Certificate of Secondary Education

BIOLOGY 0610/12
Paper 1 Multiple Choice (Core) October/November 2017
45 minutes
Additional Materials: Multiple Choice Answer Sheet
Soft clean eraser
*9230046908*

Soft pencil (type B or HB is recommended)

READ THESE INSTRUCTIONS FIRST

Write in soft pencil.


Do not use staples, paper clips, glue or correction fluid.
Write your name, Centre number and candidate number on the Answer Sheet in the spaces provided
unless this has been done for you.
DO NOT WRITE IN ANY BARCODES.

There are forty questions on this paper. Answer all questions. For each question there are four possible
answers A, B, C and D.
Choose the one you consider correct and record your choice in soft pencil on the separate Answer Sheet.

Read the instructions on the Answer Sheet very carefully.

Each correct answer will score one mark. A mark will not be deducted for a wrong answer.
Any rough working should be done in this booklet.
Electronic calculators may be used.

The syllabus is approved for use in England, Wales and Northern Ireland as a Cambridge International Level 1/Level 2 Certificate.

This document consists of 14 printed pages and 2 blank pages.

IB17 11_0610_12/3RP
© UCLES 2017 [Turn359
over
2

1 Which process do living organisms use to remove excess carbon dioxide from their bodies?

A excretion
B movement
C nutrition
D respiration

2 When a donkey is bred with a zebra, a zebronkey is produced. Zebronkeys are not fertile
animals.

Which statement is correct?

A Donkeys and zebras belong to different species because their offspring are not fertile.
B Donkeys and zebras belong to different species because they do not look exactly the same.
C Donkeys and zebras belong to the same species because they look alike.
D Donkeys and zebras belong to the same species because they produce offspring.

3 The diagram shows a leaf.

Use the key to identify the leaf.

1 leaf with several small leaflets .................................. go to 2


leaf with one large leaf blade .................................... go to 3
2 leaflets are broad and flat ......................................... A
leaflets are narrow and hair-like ............................... B
3 leaf with a smooth edge ........................................... C
leaf with a toothed edge ........................................... D

360
© UCLES 2017 0610/12/O/N/17
3

4 The diagram shows a cell from the leaf of a plant.

Which part contains the highest amount of magnesium?

B
C
D

5 What is a leaf?

A a cell
B an organ
C an organ system
D a tissue

6 An experiment is set up as shown.

water

soluble blue crystals

After several hours, the water turns blue.

Which process causes this colour change to take place?

A absorption
B active transport
C diffusion
D osmosis

361
© UCLES 2017 0610/12/O/N/17 [Turn over
4

7 A cell has an actual length of 0.2 mm. A student wants to make a drawing of this cell so that it
appears two hundred times bigger.

How long should the student draw this cell?

A 4 mm B 0.04 cm C 40 mm D 40 cm

8 What are features of osmosis?

requires a partially
diffusion is
requires cell walls permeable
involved
membrane

A   
B   
C   
D   

9 Which of these is digested by protease?

A key
B amino acid
glucose
C
chemical bond
D

10 Which chemical is used to test for the presence of protein in a food sample?

A Benedict’s solution
B biuret solution
C DCPIP
D iodine solution

362
© UCLES 2017 0610/12/O/N/17
5

11 Four test-tubes were set up as shown in the table.

In which test-tube would starch be broken down the fastest?

2 cm3 starch
1 cm3 of 1 cm3 of boiled temperature
suspension
amylase added amylase added / °C
added

A    5 key
B    35 = present
C    5 = absent
D    35

12 The average number of chloroplasts in four different types of cell taken from a plant is shown.

Which is a root hair cell?

A 0 B 47 C 370 D 920

13 What must be increased in the diet of a person suffering from constipation?

A fats
B fibre
C iron
D protein

363
© UCLES 2017 0610/12/O/N/17 [Turn over
6

14 The diagram shows the human alimentary canal, with a string marked in metres beside it.

oesophagus
0

3
string marked
in metres
4

9 anus

How long is the small intestine?

A 2m B 6m C 8m D 9m

15 What could lead to obesity?

A energy consumption exceeding energy expenditure


B increased consumption of dietary fibre
C reduced consumption of fat
D reduced consumption of sugar

16 Which stage of nutrition takes place when food molecules become part of a body cell?

A absorption
B assimilation
C digestion
D ingestion

364
© UCLES 2017 0610/12/O/N/17
7

17 The diagram shows a transverse section through a plant organ.

What is M?

A phloem in a root
B phloem in a stem
C xylem in a root
D xylem in a stem

18 What is a description of transpiration?

A exchange of gases between the leaf and the atmosphere


B loss of water vapour from the leaves and stems of a plant
C movement of water from the roots to the leaves
D movement of water through the cells of the leaf

19 The diagram shows a cross-section through a human blood vessel.

elastic and fibrous


connective tissue

muscle layer

Which type of blood vessel does the diagram show?

A an artery
B a capillary
C a vein
D a ventricle

365
© UCLES 2017 0610/12/O/N/17 [Turn over
8

20 The diagram shows a section through a mammalian heart.

Which part carries blood directly from the lungs?

B C

A D

21 Which is a mechanical barrier to pathogens?

A acid in the stomach


B hairs in the nose
C mucus in the trachea
D phagocytosis in the blood

22 What is the approximate percentage of oxygen in exhaled air?

A 0.04 B 4.00 C 16.00 D 21.00

366
© UCLES 2017 0610/12/O/N/17
9

23 In an experiment to investigate anaerobic respiration, two bottles are set up in a warm room, as
shown.

P Q
balloon balloon

sugar, yeast yeast


and water and water

What would happen to each balloon after one day?

A B

P Q P Q

C D

P Q P Q

367
© UCLES 2017 0610/12/O/N/17 [Turn over
10

24 The diagram represents the exchange of gases during breathing and during respiration in the
body.

oxygen
oxygen in blood

X Y

carbon
carbon dioxide
dioxide
in blood

What is represented by X and by Y?

X Y

A lungs air
B lungs body cells
C body cells air
D body cells lungs

25 What is the most important function of sweating?

A to remove excess heat from the body


B to remove excess salts from the body
C to remove excess urea from the body
D to remove excess water from the body

26 What shows the order in which these structures are involved in a reflex action?

A effector → motor neurone → relay neurone → sensory neurone → receptor

B effector → sensory neurone → motor neurone → relay neurone→ receptor

C receptor → sensory neurone → relay neurone → motor neurone → effector

D receptor → motor neurone → sensory neurone → relay neurone→ effector

27 Which acts as a sense organ?

A the gall bladder


B the kidney
C the skin
D the ureter

368
© UCLES 2017 0610/12/O/N/17
11

28 The diagram shows a person sweating in hot weather.

What part is played by sweat glands during the process of sweating?

A effector
B receptor
C sense organ
D stimulus

29 Which disease is caused by bacteria?

A AIDS
B cholera
C obesity
D scurvy

30 Where does fertilisation occur in a flowering plant?

A in the ovary
B in the pollen grain
C in the style
D on the stigma

31 Which method of birth control works by preventing an egg from being released?

A condom
B contraceptive pill
C monitoring body temperature
D vasectomy

369
© UCLES 2017 0610/12/O/N/17 [Turn over
12

32 Which environmental factor is not always a requirement for seed germination?

A light
B oxygen
C suitable temperature
D water

33 In some plants, H is the dominant allele for hairy stems and h is the recessive allele for smooth
stems.

A pair of these plants produce 37 offspring, 18 with hairy stems and 19 with smooth stems.

What are the most likely genotypes of the parents?

A HH × HH B Hh × Hh C Hh × hh D hh × hh

34 The family tree shows the inheritance of the ability to smell flowers called freesias. The allele for
the ability to smell freesias is dominant.

Which individual’s symbol is not correct?

key
A female able to
smell freesias
female unable to
smell freesias
male able to
smell freesias
male unable to
D C B
smell freesias

35 What is an example of continuous variation?

A blood group
B gender
C height
D tongue rolling

370
© UCLES 2017 0610/12/O/N/17
13

36 Which pyramid of numbers has more herbivores than producers?

A B C D

37 The diagram shows the flow of energy along a food chain.

green plant rabbit fox

Which process in the rabbit will allow energy to be passed on to the fox?

A excretion
B growth
C movement
D sensitivity

38 The diagram shows part of the carbon cycle.

carbon dioxide
2
in atmosphere
1
plant and animal
4 remains in organic compounds
soil (humus) in plants

organic compounds 3
in animals

Which two numbered arrows represent the process of respiration?

A 1 and 2 B 1 and 3 C 2 and 4 D 3 and 4

371
© UCLES 2017 0610/12/O/N/17 [Turn over
14

39 When making commercial apple juice, the fruit is crushed to separate cells and to release the cell
contents. Chemical Q is found between the cells. It holds the cells together but it makes the
extracted juice cloudy.

Which process is used to produce a clear juice?

A adding more water to dissolve chemical Q


B adding pectinase to digest chemical Q
C boiling the juice to destroy chemical Q
D crushing the apples to release chemical Q

40 The table shows the amount of carbon dioxide in the atmosphere in three different years.

year 1930 1980 1990


carbon dioxide / parts per million 300 330 370

What is the most likely cause of this change?

A destruction of rainforests
B increased use of fertilisers containing nitrogen
C pollution of air by sulfur dioxide
D rise in the sea level

372
© UCLES 2017 0610/12/O/N/17
Cambridge International Examinations
Cambridge International General Certificate of Secondary Education

BIOLOGY 0610/13
Paper 1 Multiple Choice (Core) October/November 2017
45 minutes
Additional Materials: Multiple Choice Answer Sheet
Soft clean eraser
*7127785257*

Soft pencil (type B or HB is recommended)

READ THESE INSTRUCTIONS FIRST

Write in soft pencil.


Do not use staples, paper clips, glue or correction fluid.
Write your name, Centre number and candidate number on the Answer Sheet in the spaces provided
unless this has been done for you.
DO NOT WRITE IN ANY BARCODES.

There are forty questions on this paper. Answer all questions. For each question there are four possible
answers A, B, C and D.
Choose the one you consider correct and record your choice in soft pencil on the separate Answer Sheet.

Read the instructions on the Answer Sheet very carefully.

Each correct answer will score one mark. A mark will not be deducted for a wrong answer.
Any rough working should be done in this booklet.
Electronic calculators may be used.

The syllabus is approved for use in England, Wales and Northern Ireland as a Cambridge International Level 1/Level 2 Certificate.

This document consists of 16 printed pages.

IB17 11_0610_13/4RP
© UCLES 2017 [Turn373
over
2

1 Which process releases water and energy?

A aerobic respiration
B osmosis
C photosynthesis
D protein synthesis

2 The dire wolf is an extinct species of wolf.

What is the correct scientific name for this wolf?

A Canis Dirus
B canis dirus
C Canis dirus
D canis Dirus

3 Which of the animals shown is a crustacean?

A B C D

4 The diagram shows a type of plant cell.

nucleus

cytoplasm

cell wall

vacuole

Where does this type of cell come from?

A a root
B cuticle
C palisade mesophyll
D spongy mesophyll
374
© UCLES 2017 0610/13/O/N/17
3

5 Which structures are found in a white blood cell?

cell cell large


chloroplast cytoplasm nucleus
membrane wall vacuole

A      
B      
C      
D      

6 The diagram shows the structure of a plant cell.

This cell is part of a tissue which

A absorbs carbon dioxide from the air.


B absorbs ions from the soil.
C transports sucrose from leaves.
D transports water in stems.

7 A student made the following statements about the movement of ions by active transport.

1 It is the net movement of particles from a low concentration to a high concentration.


2 It is the net movement of particles from a high concentration to a low concentration.
3 It requires the use of energy.
4 It can only take place in living, respiring cells.

Which statements are correct?

A 1, 3 and 4 B 1 and 4 only C 2 and 4 D 2 only

375
© UCLES 2017 0610/13/O/N/17 [Turn over
4

8 Which diagram shows the appearance of a plant cell after it is placed in pure water?

A B

C D

9 What is the colour change shown by Benedict’s solution when heated with a reducing sugar?

A blue to purple
B blue to red
C brown to blue-black
D red to yellow

10 Into which part of the alimentary canal is the enzyme that digests starch secreted?

376
© UCLES 2017 0610/13/O/N/17
5

11 Which of these is digested by protease?

A key
B amino acid
glucose
C
chemical bond
D

12 What must be increased in the diet of a person suffering from constipation?

A fats
B fibre
C iron
D protein

13 The diagram shows the human alimentary canal, with a string marked in metres beside it.

oesophagus
0

3
string marked
in metres
4

9 anus

How long is the small intestine?

A 2m B 6m C 8m D 9m

377
© UCLES 2017 0610/13/O/N/17 [Turn over
6

14 A person has swollen, bleeding gums and slow wound healing.

This could be caused by a lack of which nutrient in a diet?

A calcium
B fibre
C iron
D vitamin C

15 Which function is performed by the duodenum?

A assimilation
B digestion
C egestion
D ingestion

16 Solid food enters the mouth at P and enters the oesophagus at Q.

How does the food at Q differ from the food at P?

A It contains less fibre.


B It contains less vitamin D.
C It contains less protein.
D It contains less starch.

17 What is a description of transpiration?

A exchange of gases between the leaf and the atmosphere


B loss of water vapour from the leaves and stems of a plant
C movement of water from the roots to the leaves
D movement of water through the cells of the leaf

378
© UCLES 2017 0610/13/O/N/17
7

18 Which words correctly complete the following two sentences?

During transpiration, water moves from the ......1...... in a leaf and passes into ......2...... cells.

It then leaves the surface of these cells by ......3...... and ......4...... out of the stomata.

1 2 3 4

A phloem xylem osmosis evaporates


B stomata guard active transport flows
C xylem mesophyll diffusion drains
D xylem mesophyll evaporation diffuses

19 The diagram shows a cross-section through a human blood vessel.

elastic and fibrous


connective tissue

muscle layer

Which type of blood vessel does the diagram show?

A an artery
B a capillary
C a vein
D a ventricle

379
© UCLES 2017 0610/13/O/N/17 [Turn over
8

20 The photomicrograph shows some blood cells.

What is the function of cell P?

A It carries carbon dioxide.


B It carries oxygen.
C It helps to clot blood.
D It produces antibodies.

21 What can be passed from one person to another during blood transfusion?

A cholera
B chronic obstructive pulmonary disease (COPD)
C HIV
D scurvy

380
© UCLES 2017 0610/13/O/N/17
9

22 The diagram shows some structures in the human thorax (chest).

From which part does most oxygen pass directly into the blood?

381
© UCLES 2017 0610/13/O/N/17 [Turn over
10

23 In an experiment to investigate anaerobic respiration, two bottles are set up in a warm room, as
shown.

P Q
balloon balloon

sugar, yeast yeast


and water and water

What would happen to each balloon after one day?

A B

P Q P Q

C D

P Q P Q

382
© UCLES 2017 0610/13/O/N/17
11

24 The diagram represents the exchange of gases during breathing and during respiration in the
body.

oxygen
oxygen in blood

X Y

carbon
carbon dioxide
dioxide
in blood

What is represented by X and by Y?

X Y

A lungs air
B lungs body cells
C body cells air
D body cells lungs

25 What is the most important function of sweating?

A to remove excess heat from the body


B to remove excess salts from the body
C to remove excess urea from the body
D to remove excess water from the body

26 A student begins to lose control of her bicycle while travelling down a hill at speed.

The concentration of which substance will begin to increase rapidly in her blood?

A adrenaline
B insulin
C oestrogen
D testosterone

27 What shows the order in which these structures are involved in a reflex action?

A effector → motor neurone → relay neurone → sensory neurone → receptor

B effector → sensory neurone → motor neurone → relay neurone→ receptor

C receptor → sensory neurone → relay neurone → motor neurone → effector

D receptor → motor neurone → sensory neurone → relay neurone→ effector

383
© UCLES 2017 0610/13/O/N/17 [Turn over
12

28 The diagram shows a person sweating in hot weather.

What part is played by sweat glands during the process of sweating?

A effector
B receptor
C sense organ
D stimulus

29 Which row states the possible harmful effects of tobacco smoke?

coronary
cancer liver damage heart
disease

A   
B   
C   
D   

30 Which method of birth control works by preventing an egg from being released?

A condom
B contraceptive pill
C monitoring body temperature
D vasectomy

384
© UCLES 2017 0610/13/O/N/17
13

31 Which environmental factor is not always a requirement for seed germination?

A light
B oxygen
C suitable temperature
D water

32 The diagram shows half a flower.

Q
R

After pollination, where would pollen grains be found?

A P and Q B Q and R C R and S D S and P

33 A pure-breeding white rat was crossed with a pure-breeding black rat. All their offspring were
black.

One of the offspring was bred with a pure-breeding white rat.

What is the most likely percentage of black rats in the offspring?

A 25 B 50 C 75 D 100

34 A tall pea plant is crossed with a short pea plant.

All the offspring plants are tall.

What are the genotypes of the tall parent plant and the offspring?

tall parent offspring

A heterozygous heterozygous
B heterozygous homozygous
C homozygous heterozygous
D homozygous homozygous

385
© UCLES 2017 0610/13/O/N/17 [Turn over
14

35 What is a mutation?

A a change in appearance
B a change in a gene
C a change in behaviour
D a change in the environment

36 Which two processes both result in increased water vapour in the atmosphere?

A condensation and precipitation


B condensation and transpiration
C evaporation and transpiration
D precipitation and evaporation

37 The diagram shows a food web.

plants herbivores carnivores

decomposers

What do the arrows represent?

A the absorption of oxygen


B the absorption of water
C the flow of energy
D the release of carbon dioxide

386
© UCLES 2017 0610/13/O/N/17
15

38 The diagram shows part of a food web.

snakes badgers

blackbirds frogs hedgehogs

worms slugs

plants

What is most likely to increase the size of the frog population?

A fewer hedgehogs
B fewer slugs
C more badgers
D more blackbirds

39 The diagram shows a bacterial cell that will be used to produce human insulin.

bacterial cell

bacterial
DNA
Q

What is inserted into gap Q?

A a gene from a healthy human


B cells from a human pancreas
C DNA from another bacterium
D molecules of human insulin

387
© UCLES 2017 0610/13/O/N/17 [Turn over
16

40 The table shows the amount of carbon dioxide in the atmosphere in three different years.

year 1930 1980 1990


carbon dioxide / parts per million 300 330 370

What is the most likely cause of this change?

A destruction of rainforests
B increased use of fertilisers containing nitrogen
C pollution of air by sulfur dioxide
D rise in the sea level

Permission to reproduce items where third-party owned material protected by copyright is included has been sought and cleared where possible. Every
reasonable effort has been made by the publisher (UCLES) to trace copyright holders, but if any items requiring clearance have unwittingly been included, the
publisher will be pleased to make amends at the earliest possible opportunity.

To avoid the issue of disclosure of answer-related information to candidates, all copyright acknowledgements are reproduced online in the Cambridge
International Examinations Copyright Acknowledgements Booklet. This is produced for each series of examinations and is freely available to download at
www.cie.org.uk after the live examination series.

388
Cambridge International Examinations is part of the Cambridge Assessment Group. Cambridge Assessment is the brand name of University of Cambridge Local
Examinations Syndicate (UCLES), which is itself a department of the University of Cambridge.

© UCLES 2017 0610/13/O/N/17


Cambridge International Examinations
Cambridge International General Certificate of Secondary Education

BIOLOGY 0610/21
Paper 2 Multiple Choice (Extended) May/June 2017
45 minutes
Additional Materials: Multiple Choice Answer Sheet
Soft clean eraser
*0190070946*

Soft pencil (type B or HB is recommended)

READ THESE INSTRUCTIONS FIRST

Write in soft pencil.


Do not use staples, paper clips, glue or correction fluid.
Write your name, Centre number and candidate number on the Answer Sheet in the spaces provided
unless this has been done for you.
DO NOT WRITE IN ANY BARCODES.

There are forty questions on this paper. Answer all questions. For each question there are four possible
answers A, B, C and D.
Choose the one you consider correct and record your choice in soft pencil on the separate Answer Sheet.

Read the instructions on the Answer Sheet very carefully.

Each correct answer will score one mark. A mark will not be deducted for a wrong answer.
Any rough working should be done in this booklet.
Electronic calculators may be used.

The syllabus is approved for use in England, Wales and Northern Ireland as a Cambridge International Level 1/Level 2 Certificate.

This document consists of 15 printed pages and 1 blank page.

IB17 06_0610_21_VI_LIL/FP
© UCLES 2017 [Turn389
over
2

1 Which characteristic do all living organisms show?

A breathing
B excretion
C photosynthesis
D tropism

2 The diagram shows some animal cells, as seen under the microscope.

What will be present at X?

A one cell membrane


B one cell wall
C two cell membranes
D two cell walls

3 Each of the diagrams shows an area of cytoplasm.

Which is from a prokaryote?

A B C D

390
© UCLES 2017 0610/21/M/J/17
3

4 The diagram shows a section through a root.

P
Q

What are the levels of organisation of the labelled structures?

cell organ tissue

A P Q R
B P R Q
C Q R P
D R Q P

5 Which characteristics are correct for both osmosis and diffusion?

require a partially require a are energy


permeable membrane concentration gradient consuming processes

A   
B   
C   
D   

391
© UCLES 2017 0610/21/M/J/17 [Turn over
4

6 Which diagram shows the structure of DNA?

A B C D

A C A T A C A G
A C A T A C A T

T G T A T G T G
C A C G C A C G
G T G C G T G C
A C A T A C C T

G T G C G T G C

7 The graph shows how enzyme activity is affected by temperature.

rate of
reaction

0 20 40 60
temperature / °C

How can the change in activity between 40 °C and 55 °C be explained?

A Heat has killed the enzyme.


B The enzyme has been used up.
C The reactants are moving faster.
D The substrate is less likely to fit into the active site.

392
© UCLES 2017 0610/21/M/J/17
5

8 Four test-tubes are set up as shown.

Which test-tube contains the most carbon dioxide after one hour?

A B C D

black
water polythene
to keep
water out light
snail light light light
aquatic
plant

9 The graph shows daily carbon dioxide uptake and transpiration by the plant Agave americana.

The plant is adapted to live in very dry conditions.

10 0.6
CO2 uptake
8
0.4 transpiration
CO2 uptake 6
/ g H 2O
/ mg per dm2
transpiration per dm2
per hour 4
0.2 per hour
2

0 0
0 4 8 12 16 20 24
time of day / hours

What can be concluded from this graph?

A More stomata are closed during dark periods.


B More stomata are closed during light periods.
C There is no carbon dioxide uptake during dark periods.
D There is no water uptake during light periods.

10 In plants, which ions are used to make amino acids?

A magnesium
B nitrates
C phosphates
D potassium

393
© UCLES 2017 0610/21/M/J/17 [Turn over
6

11 The following symptoms can be caused by a dietary deficiency:

● bone pain
● dental problems
● fragile bones
● skeletal deformities

A lack of which nutrient is most likely to cause these symptoms?

A iron
B protein
C vitamin C
D vitamin D

12 Statements 1– 4 describe stages in the development of cholera.

1 Chloride ions are secreted in the gut.


2 Osmosis causes water to move into the gut.
3 The infected person becomes dehydrated.
4 Toxins are produced by the pathogenic bacteria.

What is the correct sequence of the four stages?

A 1→2→3→4

B 1→4→3→2

C 4→1→2→3

D 4→1→3→2

394
© UCLES 2017 0610/21/M/J/17
7

13 The diagram shows a human tooth.

Which statement best describes the function of this tooth?

A It bites into food.


B It cuts food.
C It grinds food.
D It tears food.

14 The graph shows the effect of pH on the activity of three different enzymes.

Z
X Y

enzyme
activity

0 2 4 6 8 10
pH

The table shows the pH of different parts of the alimentary canal.

part of the
pH
alimentary canal

mouth 7.0
stomach 2.0
small intestine 8.5

Which enzymes in the graph are likely to be protease enzymes?

A X, Y and Z B X and Z only C Y and Z only D Z only

395
© UCLES 2017 0610/21/M/J/17 [Turn over
8

15 The diagrams show cross-sections through a root and a stem.

Y
W Z
V

root stem

Which labels are correct?

V W Y Z

A phloem xylem xylem phloem


B phloem xylem phloem xylem
C xylem phloem xylem phloem
D xylem phloem phloem xylem

16 Which processes are used by root hairs to take up ions and water?

ion uptake water uptake

A active transport osmosis


B diffusion active transport
C osmosis diffusion
D osmosis osmosis

17 Which of the following increases transpiration?

A air around the leaf with high humidity


B air molecules around the leaf with less kinetic energy
C an absence of light falling on the leaf
D water molecules in the leaf with more kinetic energy

396
© UCLES 2017 0610/21/M/J/17
9

18 The diagram shows a human blood cell.

What is its function?

A antibody production
B fibrinogen production
C oxygen transport
D phagocytosis

19 What is a disease-causing organism called?

A antibody
B host
C pathogen
D phagocyte

20 Which is a function of the cartilage in the trachea?

A gas exchange
B prevents collapse of the trachea
C produces mucus
D traps bacteria

21 Which is the equation for anaerobic respiration in yeast?

A C6H12O6 → 2C2H5OH + 2CO2

B C6H12O6 + 6O2 → 6CO2 + 6H2O

C 2CO2 + 2C2H5OH → C6H12O6

D 6CO2 + 6H2O → C6H12O6 + 6O2

397
© UCLES 2017 0610/21/M/J/17 [Turn over
10

22 A longitudinal section of the kidney and some associated structures have been labelled.

Which labelling is correct?

A B

cortex cortex

renal artery renal vein

ureter ureter
medulla medulla

C D

medulla medulla

renal artery ureter

ureter renal artery


cortex cortex

23 The steps to produce a reflex action where the hand is withdrawn from a hot object are shown.

1 Nerve impulses pass from the sensory neurone to the relay neurone.
2 Nerve impulses pass from motor neurone to the effector.
3 Nerve impulses pass from the receptor to the neurone.

What is the correct sequence of steps?

A 1→2→3 B 2→1→3 C 3→1→2 D 3→2→1

398
© UCLES 2017 0610/21/M/J/17
11

24 How many different kinds of cone receptor and rod receptor are there in the human retina?

types of types of
cone receptor rod receptor

A 1 1
B 1 3
C 3 1
D 3 3

25 What is the result of the release of adrenaline?

blood glucose
pulse rate
concentration

A decreased decreased
B decreased increased
C increased decreased
D increased increased

26 Which statement about microorganisms is correct?

A Bacteria are killed by hydrochloric acid in the duodenum.


B Bacteria can become resistant to antibiotics.
C Bacterial infection and viruses are treated with antibiotics.
D Bacteria produce sugars that cause tooth decay.

27 Dianthus flowers can be one of three different colours: red, pink or white.

A red flower is always homozygous and a white flower is always homozygous. Pink flowers are
heterozygous.

If a red and a white flower are crossed, what percentage of the offspring will be pink?

A 0% B 25% C 75% D 100%

28 An alligator has 32 chromosomes in each of the cells of its nose.

How many chromosomes will an alligator zygote contain?

A 16 B 32 C 46 D 64

399
© UCLES 2017 0610/21/M/J/17 [Turn over
12

29 Which hormone is produced by cells in the placenta of a developing fetus?

A FSH
B LH
C testosterone
D progesterone

30 The diagram shows four types of birth control device.

Which is not a barrier method of contraception?

A B C D

31 Which blood component is destroyed in people who are HIV positive?

A lymphocytes
B phagocytes
C platelets
D red blood cells

32 What are stem cells?

A specialised cells that divide by meiosis to produce daughter cells


B specialised cells that divide by mitosis to produce daughter cells
C unspecialised cells that divide by meiosis to produce daughter cells
D unspecialised cells that divide by mitosis to produce daughter cells

400
© UCLES 2017 0610/21/M/J/17
13

33 The list shows the diploid number of chromosomes of four organisms.

fruit fly 8
human 46
potato 48
garden pea 14

What is the correct chromosome number of a male sex cell in each of these organisms?

fruit fly human potato garden pea

A 4 23 24 7
B 4 46 12 7
C 8 46 48 14
D 16 92 96 28

34 Which of these is a population?

1 all the insects in a rainforest

2 all the tigers in a rainforest

3 all the kapok trees in a rainforest

A 1, 2 and 3 B 1 only C 1 and 2 only D 2 and 3 only

35 The number of bacteria resistant to antibiotics is increasing.

What has contributed to this situation?

A choosing the antibiotic carefully to match the strain of bacterium


B not completing the course of prescribed antibiotics
C tracing, testing and treating people who have been in contact with the infected person
D using antibiotics only when essential

401
© UCLES 2017 0610/21/M/J/17 [Turn over
14

36 The processes listed in 1– 4 will affect the concentration of carbon dioxide in the atmosphere.

1 increased deforestation
2 increased forestation
3 decreased fossil fuel combustion
4 increased fossil fuel combustion

Which processes would increase the concentration of carbon dioxide in the atmosphere?

A 1 and 3 B 1 and 4 C 2 and 3 D 2 and 4

37 The graph shows the changes in the populations of predator and prey over a period of time.

Which point on the graph shows a decrease in predator population?

A
C
number of
individuals

D
B

time

38 Which is a reason for using bacteria in biotechnology?

A Bacteria are found inside the human body.


B Bacteria can become resistant to antibiotics.
C Bacteria can make complex molecules.
D Bacteria reproduce slowly.

39 Why is yeast used in breadmaking?

A to produce alcohol
B to produce carbon dioxide
C to use up oxygen
D to use up sugar

402
© UCLES 2017 0610/21/M/J/17
15

40 The graph shows the levels of dissolved oxygen and mineral ions in a river.

At which point does raw sewage enter the river?

direction of river flow

dissolved oxygen

levels

mineral ions

A B C D

distance along river

403
© UCLES 2017 0610/21/M/J/17
Cambridge International Examinations
Cambridge International General Certificate of Secondary Education

BIOLOGY 0610/22
Paper 2 Multiple Choice (Extended) May/June 2017
45 minutes
Additional Materials: Multiple Choice Answer Sheet
Soft clean eraser
*0369736826*

Soft pencil (type B or HB is recommended)

READ THESE INSTRUCTIONS FIRST

Write in soft pencil.


Do not use staples, paper clips, glue or correction fluid.
Write your name, Centre number and candidate number on the Answer Sheet in the spaces provided
unless this has been done for you.
DO NOT WRITE IN ANY BARCODES.

There are forty questions on this paper. Answer all questions. For each question there are four possible
answers A, B, C and D.
Choose the one you consider correct and record your choice in soft pencil on the separate Answer Sheet.

Read the instructions on the Answer Sheet very carefully.

Each correct answer will score one mark. A mark will not be deducted for a wrong answer.
Any rough working should be done in this booklet.
Electronic calculators may be used.

The syllabus is approved for use in England, Wales and Northern Ireland as a Cambridge International Level 1/Level 2 Certificate.

This document consists of 15 printed pages and 1 blank page.

IB17 06_0610_22/3RP
© UCLES 2027 [Turn404
over
2

1 Which characteristic do all living organisms show?

A breathing
B excretion
C photosynthesis
D tropism

2 The diagram shows some animal cells, as seen under the microscope.

What will be present at X?

A one cell membrane


B one cell wall
C two cell membranes
D two cell walls

3 A plant has leaves that have white areas and green areas.

white area
green area

Which cell structures are not present in the white areas?

A cell membranes
B cell walls
C chloroplasts
D vacuoles

405
© UCLES 2017 0610/22/M/J/17
3

4 The diagram shows a section through a root.

P
Q

What are the levels of organisation of the labelled structures?

cell organ tissue

A P Q R
B P R Q
C Q R P
D R Q P

5 Which characteristics are correct for both osmosis and diffusion?

require a partially require a are energy


permeable membrane concentration gradient consuming processes

A   
B   
C   
D   

6 Which is an example of active transport?

A carbon dioxide entering a leaf


B ion uptake by root hair cells
C oxygen moving from the alveoli into the blood
D water uptake by root hair cells

406
© UCLES 2017 0610/22/M/J/17 [Turn over
4

7 An experiment was carried out to study the effect of temperature on the time taken for protein to
be digested by an enzyme.

The table shows the results.

temperature / °C observation

25 4 hours for complete digestion


35 2 hours for complete digestion
45 3 hours for complete digestion
55 no digestion takes place

For these results, at which temperature does the enzyme denature?

A 20 °C B 30 °C C 40 °C D 50 °C

8 Four test-tubes are set up as shown.

Which test-tube contains the most carbon dioxide after one hour?

A B C D

black
water polythene
to keep
water out light
snail light light light
aquatic
plant

407
© UCLES 2017 0610/22/M/J/17
5

9 The diagram shows a cross-section through a leaf.

What is the cell labelled X?

A epidermis
B guard cell
C palisade mesophyll
D spongy mesophyll

10 The nutrient ions present in four different soils are shown.

Which soil would be best for growing healthy, green plants?

nitrate ions magnesium ions

A absent absent
B absent present
C present absent
D present present

11 Which disease is caused by a deficiency of iron in the diet?

A anaemia
B kwashiorkor
C marasmus
D rickets

408
© UCLES 2017 0610/22/M/J/17 [Turn over
6

12 Statements 1– 4 describe stages in the development of cholera.

1 Chloride ions are secreted in the gut.


2 Osmosis causes water to move into the gut.
3 The infected person becomes dehydrated.
4 Toxins are produced by the pathogenic bacteria.

What is the correct sequence of the four stages?

A 1→2→3→4

B 1→4→3→2

C 4→1→2→3

D 4→1→3→2

13 Which type of tooth does the diagram show?

A canine
B incisor
C molar
D premolar

409
© UCLES 2017 0610/22/M/J/17
7

14 The graph shows the effect of pH on the activity of three different enzymes.

Z
X Y

enzyme
activity

0 2 4 6 8 10
pH

The table shows the pH of different parts of the alimentary canal.

part of the
pH
alimentary canal

mouth 7.0
stomach 2.0
small intestine 8.5

Which enzymes in the graph are likely to be protease enzymes?

A X, Y and Z B X and Z only C Y and Z only D Z only

15 Which statement describes assimilation?

A the breakdown of large, insoluble molecules into small soluble molecules


B the movement of small soluble molecules through the wall of the intestine into the blood
C the movement of small soluble molecules into the cells of the body, where they are used,
becoming part of the cells
D the taking of substances into the body through the mouth

410
© UCLES 2017 0610/22/M/J/17 [Turn over
8

16 The diagram shows a cross-section of a plant stem.

What is the function of the tissue labelled V?

A transporting dissolved nutrients and mineral ions


B transporting dissolved nutrients only
C transporting water and mineral ions
D transporting water only

17 Which process occurs during transpiration?

A evaporation of water from the xylem


B loss of water by osmosis from the guard cells
C movement of water vapour through the spongy mesophyll by active transport
D movement of water vapour through the stomata by diffusion

411
© UCLES 2017 0610/22/M/J/17
9

18 The diagrams show an experiment on transpiration.

Four leaves of the same species are balanced on two drinking straws. One or both sides of the
leaves are covered in grease. Any difference in mass causes the heavier end to be lower.

leaf 1 leaf 2
no covering both surfaces
covered

thread

leaf 3 drinking leaf 4


paper clip
upper surface straw lower surface
covered covered

At the start of the experiment the straws were positioned so that the leaves were level.

Which leaves will be lower after an hour?

A 1 and 3 B 1 and 4 C 2 and 3 D 2 and 4

19 Which chamber of the heart has the most muscular wall?

A left atrium
B left ventricle
C right atrium
D right ventricle

20 A deficiency in which of the following may result in a person’s blood failing to clot properly?

A antibodies
B fibrinogen
C haemoglobin
D protease

412
© UCLES 2017 0610/22/M/J/17 [Turn over
10

21 Which is a function of some white blood cells?

A to carry glucose
B to carry oxygen
C to produce antibiotics
D to produce antibodies

22 A longitudinal section of the kidney and some associated structures have been labelled.

Which labelling is correct?

A B

cortex cortex

renal artery renal vein

ureter ureter
medulla medulla

C D

medulla medulla

renal artery ureter

ureter renal artery


cortex cortex

413
© UCLES 2017 0610/22/M/J/17
11

23 The diagram shows a reflex arc.

not to scale

What is X?

A a relay neurone
B a synapse
C the effector
D the receptor

24 Which responses occur in the iris of the eye when a person walks from a dimly lit area to a
brightly lit area?

circular muscle radial muscle

A contract contract
B contract relax
C relax contract
D relax relax

25 Which is the result of the release of adrenaline?

A constriction of pupils
B decrease in breathing rate
C decrease in pulse rate
D increase in blood glucose concentration

414
© UCLES 2017 0610/22/M/J/17 [Turn over
12

26 Penicillin is an antibiotic that kills bacteria by preventing them from forming cell walls.

Why does penicillin not affect viruses?

A Viruses become resistant to penicillin.


B Viruses do not contain cytoplasm.
C Viruses do not have cell walls.
D Viruses reproduce very rapidly.

27 Dianthus flowers can be one of three different colours: red, pink or white.

A red flower is always homozygous and a white flower is always homozygous. Pink flowers are
heterozygous.

If a red and a white flower are crossed, what percentage of the offspring will be pink?

A 0% B 25% C 75% D 100%

28 Which row correctly shows whether the nuclei of the cells are diploid or haploid?

body cells gametes zygote

A diploid diploid haploid


B diploid haploid diploid
C haploid diploid haploid
D haploid haploid diploid

29 What is a feature of self-pollination?

A It improves the capacity of a species to respond to changes in the environment.


B It increases variation in the offspring.
C Pollen grains are transferred from the anther of a flower to the stigma of a flower on a
different plant.
D Pollen grains are transferred from the anther of a flower to the stigma of a flower on the same
plant.

30 Which hormone causes the lining of the uterus to become thick and glandular before ovulation?

A FSH
B LH
C oestrogen
D progesterone

415
© UCLES 2017 0610/22/M/J/17
13

31 Which blood component is destroyed in people who are HIV positive?

A lymphocytes
B phagocytes
C platelets
D red blood cells

32 Which row describes the genetic code in DNA?

what forms the genetic code what the DNA codes for

A sequence of amino acids sequence of bases


B sequence of amino acids sequence of proteins
C sequence of bases sequence of amino acids
D sequence of bases sequence of proteins

33 Cell division by meiosis of a parent cell with 23 pairs of chromosomes will result in

A 2 cells, each with 23 pairs of chromosomes.


B 2 cells, each with 23 single chromosomes.
C 4 cells, each with 23 pairs of chromosomes.
D 4 cells, each with 23 single chromosomes.

34 Colour blindness is a condition that occurs more frequently in men than in women.

Which statement about this condition is correct?

A It affects the cornea.


B It can pass from father to son.
C It is a sex-linked characteristic.
D The gene is on the Y chromosome.

35 Which feature is an example of discontinuous variation?

A blood group
B body mass
C foot size
D height

416
© UCLES 2017 0610/22/M/J/17 [Turn over
14

36 In which processes is light energy converted to chemical energy?

photosynthesis plant respiration

A  
B  
C  
D  

37 The graph shows the changes in the populations of predator and prey over a period of time.

Which point on the graph shows a decrease in predator population?

A
C
number of
individuals

D
B

time

38 Which feature of bacteria makes them useful in genetic engineering?

A They do not have mitochondria.


B They do not have vacuoles.
C They have plasmids.
D They have cell walls.

39 Why is yeast used in breadmaking?

A to produce alcohol
B to produce carbon dioxide
C to use up oxygen
D to use up sugar

417
© UCLES 2017 0610/22/M/J/17
15

40 The graph shows the levels of dissolved oxygen and mineral ions in a river.

At which point does raw sewage enter the river?

direction of river flow

dissolved oxygen

levels

mineral ions

A B C D

distance along river

418
© UCLES 2017 0610/22/M/J/17
Cambridge International Examinations
Cambridge International General Certificate of Secondary Education

BIOLOGY 0610/23
Paper 2 Multiple Choice (Extended) May/June 2017
45 minutes
Additional Materials: Multiple Choice Answer Sheet
Soft clean eraser
*3291229666*

Soft pencil (type B or HB is recommended)

READ THESE INSTRUCTIONS FIRST

Write in soft pencil.


Do not use staples, paper clips, glue or correction fluid.
Write your name, Centre number and candidate number on the Answer Sheet in the spaces provided
unless this has been done for you.
DO NOT WRITE IN ANY BARCODES.

There are forty questions on this paper. Answer all questions. For each question there are four possible
answers A, B, C and D.
Choose the one you consider correct and record your choice in soft pencil on the separate Answer Sheet.

Read the instructions on the Answer Sheet very carefully.

Each correct answer will score one mark. A mark will not be deducted for a wrong answer.
Any rough working should be done in this booklet.
Electronic calculators may be used.

The syllabus is approved for use in England, Wales and Northern Ireland as a Cambridge International Level 1/Level 2 Certificate.

This document consists of 16 printed pages.

IB17 06_0610_23/2RP
© UCLES 2017 [Turn419
over
2

1 Which characteristic do all living organisms show?

A breathing
B excretion
C photosynthesis
D tropism

2 The diagram shows some animal cells, as seen under the microscope.

What will be present at X?

A one cell membrane


B one cell wall
C two cell membranes
D two cell walls

3 Which structures are present in large numbers in cells with high rates of metabolism?

A chromosomes
B mitochondria
C ribosomes
D vacuoles

420
© UCLES 2017 0610/23/M/J/17
3

4 The diagram shows a section through a root.

P
Q

What are the levels of organisation of the labelled structures?

cell organ tissue

A P Q R
B P R Q
C Q R P
D R Q P

5 Which characteristics are correct for both osmosis and diffusion?

require a partially require a are energy


permeable membrane concentration gradient consuming processes

A   
B   
C   
D   

421
© UCLES 2017 0610/23/M/J/17 [Turn over
4

6 The bases on one of the strands of a DNA molecule have the sequence shown.

A-A-T-C-T-G

What is the corresponding sequence of bases on the other strand?

A A-A-T-C-T-G
B C-C-G-A-G-T
C G-G-C-T-C-A
D T-T-A-G-A-C

7 Which statement describes the effect of temperature on enzymes?

A High temperatures denature enzymes making it difficult for substrate molecules to fit into the
active site.
B High temperatures denature enzymes making it easy for substrate molecules to fit into the
active site.
C Low temperatures denature enzymes making it difficult for substrate molecules to fit into the
active site.
D Low temperatures denature enzymes making it easy for substrate molecules to fit into the
active site.

8 Four test-tubes are set up as shown.

Which test-tube contains the most carbon dioxide after one hour?

A B C D

black
water polythene
to keep
water out light
snail light light light
aquatic
plant

422
© UCLES 2017 0610/23/M/J/17
5

9 The diagrams show the structure of four different cells from the leaf of a dicotyledonous plant.

Which cell is a guard cell?

A B

NOT TO
SCALE

C D

10 Which mineral ion is absorbed by plant roots and used in the production of all amino acids?

A carbonate
B hydroxide
C magnesium
D nitrate

11 Which is the equation for photosynthesis?

A C6H12O6 → 2C2H5OH + 2CO2

B C6H12O6 + 6O2 → 6CO2 + H2O

C 2C2H5OH + 2CO2 → C6H12O6

D 6CO2 + 6H2O → C6H12O6 + 6O2

423
© UCLES 2017 0610/23/M/J/17 [Turn over
6

12 Statements 1– 4 describe stages in the development of cholera.

1 Chloride ions are secreted in the gut.


2 Osmosis causes water to move into the gut.
3 The infected person becomes dehydrated.
4 Toxins are produced by the pathogenic bacteria.

What is the correct sequence of the four stages?

A 1→2→3→4

B 1→4→3→2

C 4→1→2→3

D 4→1→3→2

13 The diagram shows a person’s teeth. Some of their teeth are missing.

What would the person find difficult?

A biting apples
B drinking water
C grinding meat
D tearing dry bread

424
© UCLES 2017 0610/23/M/J/17
7

14 The graph shows the effect of pH on the activity of three different enzymes.

Z
X Y

enzyme
activity

0 2 4 6 8 10
pH

The table shows the pH of different parts of the alimentary canal.

part of the
pH
alimentary canal

mouth 7.0
stomach 2.0
small intestine 8.5

Which enzymes in the graph are likely to be protease enzymes?

A X, Y and Z B X and Z only C Y and Z only D Z only

425
© UCLES 2017 0610/23/M/J/17 [Turn over
8

15 The diagram shows a villus. Structures P and Q absorb different products of digestion.

P
Q

Which row identifies the products absorbed by P and Q?

P Q

A amino acids glucose


B fatty acids maltose
C glucose fatty acids
D maltose amino acids

16 The diagram shows a leaf attached to the stem of a plant.

sunlight

What do the arrows at X represent?

A movement of amino acids in phloem


B movement of carbon dioxide in phloem
C movement of mineral ions in xylem
D movement of sucrose in xylem

426
© UCLES 2017 0610/23/M/J/17
9

17 Which row shows what is carried, and in which direction, by the pulmonary vein?

what is carried direction

A deoxygenated blood from the heart


B deoxygenated blood to the heart
C oxygenated blood from the heart
D oxygenated blood to the heart

18 Which component of the blood produces antibodies?

A lymphocytes
B phagocytes
C plasma
D red blood cells

19 Which statement about antibodies is correct?

A Breast milk contains antibodies and protects babies by giving them active immunity.
B Injections of antibodies give passive immunity against the disease scurvy.
C Insect repellents contain antibodies and give mosquitoes passive immunity against malaria.
D Injections of antibodies give passive immunity against some pathogens.

20 Which row shows the correct combination of muscle contractions and the pressure in the thorax
when breathing out?

internal external
pressure
intercostal intercostal diaphragm
in thorax
muscles muscles

A contracted contracted contracted high


B contracted relaxed relaxed high
C relaxed contracted contracted high
D relaxed relaxed relaxed low

427
© UCLES 2017 0610/23/M/J/17 [Turn over
10

21 Which is not involved in removing the oxygen debt after a human has been exercising?

A aerobic respiration of alcohol in the liver


B aerobic respiration of lactic acid in the liver
C continuation of deeper breathing
D continuation of faster heart rate

22 A longitudinal section of the kidney and some associated structures have been labelled.

Which labelling is correct?

A B

cortex cortex

renal artery renal vein

ureter ureter
medulla medulla

C D

medulla medulla

renal artery ureter

ureter renal artery


cortex cortex

428
© UCLES 2017 0610/23/M/J/17
11

23 The diagram shows a synapse.

What do the dots at X represent?

A neurotransmitter
B receptor molecule
C synaptic cleft
D vesicle

24 Which is the light sensitive part of the eye?

A cornea
B iris
C lens
D retina

25 What is meant by the term phototropism?

A absorbing mineral ions using light energy


B directional growth in response to gravity
C directional growth in response to light
D making food using light energy

26 Which row shows the effects of excessive alcohol consumption?

anti-social
reaction time self-control
behaviour

A quicker increased increased


B quicker reduced increased
C slower increased decreased
D slower reduced increased

429
© UCLES 2017 0610/23/M/J/17 [Turn over
12

27 Dianthus flowers can be one of three different colours: red, pink or white.

A red flower is always homozygous and a white flower is always homozygous. Pink flowers are
heterozygous.

If a red and a white flower are crossed, what percentage of the offspring will be pink?

A 0% B 25% C 75% D 100%

28 The diagram shows the ovary of a flower.

pollen grain

ovary

Which process is shown in the diagram?

A cross-pollination
B fertilisation
C meiosis
D self-pollination

29 What is a function of the placenta?

A a barrier to nicotine
B a barrier to the rubella virus
C exchange of blood
D transfer of dissolved nutrients

430
© UCLES 2017 0610/23/M/J/17
13

30 Some men do not produce healthy sperm cells. A couple may choose to use the sperm of
another man, a donor, in order for the woman to get pregnant.

Which procedure involves inserting sperm into the vagina of a woman from a donor?

A artificial insemination (AI)


B birth control
C in vitro fertilisation (IVF)
D vasectomy

31 Which blood component is destroyed in people who are HIV positive?

A lymphocytes
B phagocytes
C platelets
D red blood cells

32 Four of the processes involved in the production of a protein are shown.

1 mRNA attaches to a ribosome.


2 mRNA moves to the cytoplasm.
3 The ribosome assembles amino acids into a protein molecule.
4 An mRNA copy of the gene is made.

In which sequence do these events normally occur?

A 1→2→3→4

B 2→1→3→4

C 4→2→1→3

D 4→3→2→1

33 Which statement about the sex chromosomes is correct?

A Men and women can inherit characteristics from genes carried on the X chromosome.
B Men and women can inherit characteristics from genes carried on the Y chromosome.
C Only men can inherit characteristics from genes carried on the X chromosome.
D Only women can inherit characteristics from genes carried on the Y chromosome.

431
© UCLES 2017 0610/23/M/J/17 [Turn over
14

34 Which row best describes human blood groups?

has no shows shows


affected by
intermediate continuous discontinuous
environment
phenotypes variation variation

A    
B    
C    
D    

35 The diagram represents part of an aquatic food web.

tadpoles
water
beetles
aquatic
plants
fish
small
shrimps
frogs

Which organisms are primary consumers in this food web?

A small shrimps and tadpoles


B tadpoles and frogs
C water beetles and fish
D water beetles and frogs

432
© UCLES 2017 0610/23/M/J/17
15

36 The diagram shows part of the water cycle.

clouds

T
air

S plants water in soil

rivers and lakes

What are processes S and T?

S T

A condensation drainage
B condensation evaporation
C evaporation precipitation
D evaporation transpiration

37 The graph shows the changes in the populations of predator and prey over a period of time.

Which point on the graph shows a decrease in predator population?

A
C
number of
individuals

D
B

time

433
© UCLES 2017 0610/23/M/J/17 [Turn over
16

38 Which is an example of genetic engineering?

A altering the DNA in crop plants so they are resistant to herbicides


B only breeding from crop plants that are resistant to insect pests
C production of insulin in the pancreas
D using yeast to produce ethanol for biofuels

39 Why is yeast used in breadmaking?

A to produce alcohol
B to produce carbon dioxide
C to use up oxygen
D to use up sugar

40 The graph shows the levels of dissolved oxygen and mineral ions in a river.

At which point does raw sewage enter the river?

direction of river flow

dissolved oxygen

levels

mineral ions

A B C D

distance along river

Permission to reproduce items where third-party owned material protected by copyright is included has been sought and cleared where possible. Every
reasonable effort has been made by the publisher (UCLES) to trace copyright holders, but if any items requiring clearance have unwittingly been included, the
publisher will be pleased to make amends at the earliest possible opportunity.

To avoid the issue of disclosure of answer-related information to candidates, all copyright acknowledgements are reproduced online in the Cambridge
International Examinations Copyright Acknowledgements Booklet. This is produced for each series of examinations and is freely available to download at
www.cie.org.uk after the live examination series.

434
Cambridge International Examinations is part of the Cambridge Assessment Group. Cambridge Assessment is the brand name of University of Cambridge Local
Examinations Syndicate (UCLES), which is itself a department of the University of Cambridge.

© UCLES 2017 0610/23/M/J/17


Cambridge International Examinations
Cambridge International General Certificate of Secondary Education

BIOLOGY 0610/11
Paper 1 Multiple Choice (Core) May/June 2017
45 minutes
Additional Materials: Multiple Choice Answer Sheet
Soft clean eraser
*6354409362*

Soft pencil (type B or HB is recommended)

READ THESE INSTRUCTIONS FIRST

Write in soft pencil.


Do not use staples, paper clips, glue or correction fluid.
Write your name, Centre number and candidate number on the Answer Sheet in the spaces provided
unless this has been done for you.
DO NOT WRITE IN ANY BARCODES.

There are forty questions on this paper. Answer all questions. For each question there are four possible
answers A, B, C and D.
Choose the one you consider correct and record your choice in soft pencil on the separate Answer Sheet.

Read the instructions on the Answer Sheet very carefully.

Each correct answer will score one mark. A mark will not be deducted for a wrong answer.
Any rough working should be done in this booklet.
Electronic calculators may be used.

The syllabus is approved for use in England, Wales and Northern Ireland as a Cambridge International Level 1/Level 2 Certificate.

This document consists of 15 printed pages and 1 blank page.

IB17 06_0610_11/2RP
© UCLES 2017 [Turn435
over
2

1 Which characteristic do all living organisms show?

A breathing
B excretion
C photosynthesis
D tropism

2 The diagram shows some animal cells, as seen under the microscope.

What will be present at X?

A one cell membrane


B one cell wall
C two cell membranes
D two cell walls

3 The diagram shows an animal.

What is the animal?

1 animal with a vertebral column ....................................... vertebrate


animal with an exoskeleton ............................................ go to 2
2 no distinct head, thorax and abdomen ........................... A
distinct head, thorax and abdomen ................................ go to 3
3 eye occupies less than one third of the head ................. B
eye occupies more than one third of the head ................ go to 4
4 wings are wider than they are long ................................. C
wings are longer than they are wide ............................... D

436
© UCLES 2017 0610/11/M/J/17
3

4 The diagram shows a section through a root.

P
Q

What are the levels of organisation of the labelled structures?

cell organ tissue

A P Q R
B P R Q
C Q R P
D R Q P

5 Which characteristics are correct for both osmosis and diffusion?

require a partially require a are energy


permeable membrane concentration gradient consuming processes

A   
B   
C   
D   

437
© UCLES 2017 0610/11/M/J/17 [Turn over
4

6 The table shows the results of some food tests.

Which row shows a food containing both protein and starch?

Benedict’s iodine
biuret test ethanol
solution solution

A blue blue clear blue-black


B blue purple clear blue-black
C red blue cloudy brown
D red purple cloudy brown

7 The diagrams show an experiment on enzyme activity.

The test-tubes contain hydrogen peroxide and the enzyme catalase.

In which test-tube is the enzyme inactive?

A B C D

mixture of mixture of mixture of


catalase boiled catalase
and catalase and boiled
hydrogen and hydrogen
peroxide hydrogen peroxide
peroxide
at 4 °C at 37 °C at 37 °C at 37 °C

8 Four test-tubes are set up as shown.

Which test-tube contains the most carbon dioxide after one hour?

A B C D

black
water polythene
to keep
water out light
snail light light light
aquatic
plant

438
© UCLES 2017 0610/11/M/J/17
5

9 The diagram shows a cross-section through a leaf.

Which tissue is the cell labelled X part of?

A epidermis
B palisade mesophyll
C phloem
D spongy mesophyll

10 In plants, which ions are used to make amino acids?

A magnesium
B nitrates
C phosphates
D potassium

11 Which condition can be caused by a deficiency of vitamin C?

A constipation
B cholera
C obesity
D scurvy

12 In which structure does storage and assimilation occur?

A liver
B mouth
C pancreas
D rectum
439
© UCLES 2017 0610/11/M/J/17 [Turn over
6

13 The diagram shows a human tooth.

Which statement best describes the function of this tooth?

A It bites into food.


B It cuts food.
C It grinds food.
D It tears food.

14 To protect some medicines from the effect of gastric juice some tablets are coated with a
starch-like substance.

Which enzyme digests the coating and what is produced by this action?

enzyme products

A amylase fatty acids and glycerol


B amylase simple sugars
C lipase fatty acids and glycerol
D lipase simple sugars

440
© UCLES 2017 0610/11/M/J/17
7

15 The diagrams show cross-sections through a root and a stem.

Y
W Z
V

root stem

Which labels are correct?

V W Y Z

A phloem xylem xylem phloem


B phloem xylem phloem xylem
C xylem phloem xylem phloem
D xylem phloem phloem xylem

16 A piece of blue cobalt(II) chloride paper is clipped to the lower surface of a fresh leaf and is then
covered with plastic, as shown.

After a few minutes, part of the paper turns pink, showing that water is present.

paper remains blue

paper turns
lower surface pink
of leaf

cobalt(II) chloride paper

plastic cover

Which process carried out by leaves causes the paper to turn pink?

A absorption
B photosynthesis
C respiration
D transpiration

441
© UCLES 2017 0610/11/M/J/17 [Turn over
8

17 The diagram shows a section through the mammalian heart.

2
3

4
1

Which labelled structures are arteries?

A 1 and 2 B 1 and 4 C 2 and 3 D 3 and 4

18 What is an example of a transmissible disease?

A COPD
B HIV
C scurvy
D tooth decay

19 What is a disease-causing organism called?

A antibody
B host
C pathogen
D phagocyte

442
© UCLES 2017 0610/11/M/J/17
9

20 The diagram shows part of the human gas exchange system.

Where does the exchange of gases between air and blood take place?

21 Which statements about aerobic respiration are correct?

breaks down releases uses


nutrients energy oxygen

A   
B   
C   
D   

22 The diagram shows some organs in the human body.

In which part are amino acids broken down to form urea?

D
A

443
© UCLES 2017 0610/11/M/J/17 [Turn over
10

23 What is a synapse?

A a junction between two neurones


B a relay neurone
C an effector
D an electrical signal

24 The diagram shows the skin.

Which labelled structure releases sweat?

D A

C
B

25 The diagram shows a plant shoot growing towards light.

light

Which response is shown by the shoot of the plant?

A active transport
B gravitropism
C photosynthesis
D phototropism

444
© UCLES 2017 0610/11/M/J/17
11

26 Which statement about microorganisms is correct?

A Bacteria are killed by hydrochloric acid in the duodenum.


B Bacteria can become resistant to antibiotics.
C Bacterial infection and viruses are treated with antibiotics.
D Bacteria produce sugars that cause tooth decay.

27 Which row correctly describes the type of plant reproduction that involves pollination?

type of plant offspring compared


reproduction with parents

A asexual genetically different


B asexual genetically identical
C sexual genetically different
D sexual genetically identical

28 The diagram shows half a flower.

Which structure is the stigma?

A
B
C
D

29 A typical menstrual cycle consists of 28 days, with day 1 being the start of menstruation.

On which day is the lining of the uterus thickest?

A 1 B 4 C 9 D 21

445
© UCLES 2017 0610/11/M/J/17 [Turn over
12

30 The diagram shows four types of birth control device.

Which is not a barrier method of contraception?

A B C D

31 The diagram shows the production of a human fertilised egg cell which develops into a male.

egg cell

fertilised
egg cell
sperm cell

Which row shows the sex chromosomes in the cells?

sex chromosomes found in


egg cell sperm cell fertilised egg cell

A X X XX
B Y X XY
C X Y XY
D X Y XX

32 Which statements about mitosis are correct?

1 Mitosis produces genetically identical cells.


2 Mitosis produces genetically different cells.
3 Mitosis produces gametes.
4 Mitosis produces body cells.

A 1 and 3 B 1 and 4 C 3 and 4 D 2 and 3

446
© UCLES 2017 0610/11/M/J/17
13

33 The graph shows the masses of two different types of tomato.

type 1 type 2

number of
tomatoes

0 20 40 60 80 100 120 140 160 180 200 220 240


mass / g

What can be concluded from the graph?

A Genes do not affect the mass of tomatoes.


B Type 1 tomatoes show continuous variation.
C Type 2 tomatoes are sometimes smaller than type 1 tomatoes.
D Type 2 tomatoes show discontinuous variation.

34 Which of these is a population?

1 all the insects in a rainforest

2 all the tigers in a rainforest

3 all the kapok trees in a rainforest

A 1, 2 and 3 B 1 only C 1 and 2 only D 2 and 3 only

35 The diagram shows a food chain in a rock pool.

seaweed → whelks → crabs → seagulls

What will happen if the number of secondary consumers increases?

There will be

A fewer crabs.
B fewer seagulls.
C fewer whelks.
D less seaweed.

447
© UCLES 2017 0610/11/M/J/17 [Turn over
14

36 The diagram shows part of the carbon cycle.

1 carbon dioxide
2
3
decomposers producers 4

consumers

In which labelled stages is respiration occurring?

1 2 3 4

A    
B    
C    
D    

37 The graph shows the changes in the populations of predator and prey over a period of time.

Which point on the graph shows a decrease in predator population?

A
C
number of
individuals

D
B

time

38 Which is a reason for using bacteria in biotechnology?

A Bacteria are found inside the human body.


B Bacteria can become resistant to antibiotics.
C Bacteria can make complex molecules.
D Bacteria reproduce slowly.

448
© UCLES 2017 0610/11/M/J/17
15

39 Why is yeast used in breadmaking?

A to produce alcohol
B to produce carbon dioxide
C to use up oxygen
D to use up sugar

40 The graph shows the levels of dissolved oxygen and mineral ions in a river.

At which point does raw sewage enter the river?

direction of river flow

dissolved oxygen

levels

mineral ions

A B C D

distance along river

449
© UCLES 2017 0610/11/M/J/17
Cambridge International Examinations
Cambridge International General Certificate of Secondary Education

BIOLOGY 0610/12
Paper 1 Multiple Choice (Core) May/June 2017
45 minutes
Additional Materials: Multiple Choice Answer Sheet
Soft clean eraser
*4381318949*

Soft pencil (type B or HB is recommended)

READ THESE INSTRUCTIONS FIRST

Write in soft pencil.


Do not use staples, paper clips, glue or correction fluid.
Write your name, Centre number and candidate number on the Answer Sheet in the spaces provided
unless this has been done for you.
DO NOT WRITE IN ANY BARCODES.

There are forty questions on this paper. Answer all questions. For each question there are four possible
answers A, B, C and D.
Choose the one you consider correct and record your choice in soft pencil on the separate Answer Sheet.

Read the instructions on the Answer Sheet very carefully.

Each correct answer will score one mark. A mark will not be deducted for a wrong answer.
Any rough working should be done in this booklet.
Electronic calculators may be used.

The syllabus is approved for use in England, Wales and Northern Ireland as a Cambridge International Level 1/Level 2 Certificate.

This document consists of 16 printed pages.

IB17 06_0610_12/2RP
© UCLES 2017 [Turn450
over
2

1 Which characteristic do all living organisms show?

A breathing
B excretion
C photosynthesis
D tropism

2 The diagram shows some animal cells, as seen under the microscope.

What will be present at X?

A one cell membrane


B one cell wall
C two cell membranes
D two cell walls

3 What is a characteristic of both insects and arachnids?

A eight legs
B exoskeleton
C three pairs of legs
D wings

451
© UCLES 2017 0610/12/M/J/17
3

4 The diagram shows a section through a root.

P
Q

What are the levels of organisation of the labelled structures?

cell organ tissue

A P Q R
B P R Q
C Q R P
D R Q P

5 Which characteristics are correct for both osmosis and diffusion?

require a partially require a are energy


permeable membrane concentration gradient consuming processes

A   
B   
C   
D   

6 Which substances are made by linking together glucose molecules only?

A cellulose, glycogen and starch


B fats, cellulose and proteins
C proteins, oils and glycogen
D starch, fats and oils

452
© UCLES 2017 0610/12/M/J/17 [Turn over
4

7 Which statement about enzymes is correct?

A Enzymes are carbohydrates.


B Enzymes are catalysts.
C Enzymes are living organisms.
D Enzymes are the same shape as the substrate.

8 Four test-tubes are set up as shown.

Which test-tube contains the most carbon dioxide after one hour?

A B C D

black
water polythene
to keep
water out light
snail light light light
aquatic
plant

9 The diagram shows a cross-section through a leaf.

What is the cell labelled X?

A epidermis
B guard cell
C palisade mesophyll
D spongy mesophyll

453
© UCLES 2017 0610/12/M/J/17
5

10 The nutrient ions present in four different soils are shown.

Which soil would be best for growing healthy, green plants?

nitrate ions magnesium ions

A absent absent
B absent present
C present absent
D present present

11 Which occurs during digestion?

protease
A fat fatty acids + glycerol

lipase
B proteins simple sugars

amylase
C starch amino acids

amylase
D starch simple sugars

12 By which process does water escape from stomata in the leaves?

A active transport
B diffusion
C evaporation
D osmosis

454
© UCLES 2017 0610/12/M/J/17 [Turn over
6

13 Which type of tooth does the diagram show?

A canine
B incisor
C molar
D premolar

14 Which statement describes assimilation?

A the breakdown of large, insoluble molecules into small soluble molecules


B the movement of small soluble molecules through the wall of the intestine into the blood
C the movement of small soluble molecules into the cells of the body, where they are used,
becoming part of the cells
D the taking of substances into the body through the mouth

15 The diagram shows a cross-section of a plant stem.

What is the function of the tissue labelled V?

A transporting dissolved nutrients and mineral ions


B transporting dissolved nutrients only
C transporting water and mineral ions
D transporting water only

455
© UCLES 2017 0610/12/M/J/17
7

16 A piece of blue cobalt(II) chloride paper is clipped to the lower surface of a fresh leaf and is then
covered with plastic, as shown.

After a few minutes, part of the paper turns pink, showing that water is present.

paper remains blue

paper turns
lower surface pink
of leaf

cobalt(II) chloride paper

plastic cover

Which process carried out by leaves causes the paper to turn pink?

A absorption
B photosynthesis
C respiration
D transpiration

17 Which chamber of the heart has the most muscular wall?

A left atrium
B left ventricle
C right atrium
D right ventricle

18 Which component of the blood carries the most oxygen?

A plasma
B platelets
C red blood cells
D white blood cells

456
© UCLES 2017 0610/12/M/J/17 [Turn over
8

19 Which is a function of some white blood cells?

A to carry glucose
B to carry oxygen
C to produce antibiotics
D to produce antibodies

20 The diagram shows part of the human gas exchange system.

Where does the exchange of gases between air and blood take place?

21 The diagram shows substances moving into and out of a living cell.

glucose carbon dioxide

oxygen water

Which process is taking place in this cell?

A aerobic respiration
B anaerobic respiration
C photosynthesis
D ventilation

457
© UCLES 2017 0610/12/M/J/17
9

22 The diagram shows some organs in the human body.

In which part are amino acids broken down to form urea?

D
A

23 The diagram shows a reflex arc.

not to scale

What is X?

A a relay neurone
B a synapse
C the effector
D the receptor

458
© UCLES 2017 0610/12/M/J/17 [Turn over
10

24 The diagram shows the skin.

Which labelled area contains fatty tissue?

A
B

25 The diagram shows a seedling that has been placed in a pot of soil.

Which diagram shows what happens after five days?

A B C D

26 Which disease can be caused by smoking tobacco?

A cholera
B COPD
C HIV
D scurvy

459
© UCLES 2017 0610/12/M/J/17
11

27 The diagram shows a spider plant with a daughter spider plant growing from it.

parent plant daughter plant

Which statement is correct?

A Fertilisation was involved in this type of reproduction.


B Only one parent is needed in this type of reproduction.
C The parent and daughter plant are genetically different.
D This is an example of sexual reproduction.

28 The diagram shows half a flower.

Which structure is the stigma?

A
B
C
D

460
© UCLES 2017 0610/12/M/J/17 [Turn over
12

29 The diagram shows the human male reproductive system.

What is structure X?

A prostate gland
B scrotum
C testis
D urethra

30 Which hormone is involved in the development and regulation of male secondary sexual
characteristics?

A adrenaline
B insulin
C oestrogen
D testosterone

31 What is produced when a cell undergoes mitosis once?

A four new cells which are different from each other and from their parent cell
B four new cells which are identical to each other and to their parent cell
C two new cells which are different from each other and from their parent cell
D two new cells which are identical to each other and to their parent cell

461
© UCLES 2017 0610/12/M/J/17
13

32 The diagram shows four cells.

Which cell is produced by meiosis?

A B C D

33 The graph shows the masses of two different types of tomato.

type 1 type 2

number of
tomatoes

0 20 40 60 80 100 120 140 160 180 200 220 240


mass / g

What can be concluded from the graph?

A Genes do not affect the mass of tomatoes.


B Type 1 tomatoes show continuous variation.
C Type 2 tomatoes are sometimes smaller than type 1 tomatoes.
D Type 2 tomatoes show discontinuous variation.

34 A female guinea pig that was pure-breeding for white fur, mated with a male guinea pig that was
pure-breeding for brown fur.

All the offspring had brown fur.

One of the male offspring mated with a female with white fur.

What proportion of white offspring would be expected from this cross?

A none
B one quarter
C one third
D one half

462
© UCLES 2017 0610/12/M/J/17 [Turn over
14

35 The diagram shows a food web.

owls

small birds snakes

small mammals

beetle larvae grasshoppers

grass

Which two organisms in this web are both carnivores?

A beetle larvae and grasshoppers


B beetle larvae and snakes
C owls and grasshoppers
D owls and small birds

36 Fungi can act as decomposers feeding on dead organic matter.

Which statement describes their effect on the carbon cycle?

A releasing carbon dioxide during photosynthesis


B releasing carbon dioxide during respiration
C using carbon dioxide for photosynthesis
D using carbon dioxide for respiration

463
© UCLES 2017 0610/12/M/J/17
15

37 The graph shows the changes in the populations of predator and prey over a period of time.

Which point on the graph shows a decrease in predator population?

A
C
number of
individuals

D
B

time

38 A crop plant has been genetically modified to make it resistant to herbicides.

Which is a possible disadvantage of introducing this new crop plant?

A Loss of weeds reduces competition.


B Some weeds might become resistant to the herbicide.
C The crop plant is unharmed and produces a higher yield.
D The new gene will appear in new generations of the crop.

39 Why is yeast used in breadmaking?

A to produce alcohol
B to produce carbon dioxide
C to use up oxygen
D to use up sugar

464
© UCLES 2017 0610/12/M/J/17 [Turn over
16

40 The graph shows the levels of dissolved oxygen and mineral ions in a river.

At which point does raw sewage enter the river?

direction of river flow

dissolved oxygen

levels

mineral ions

A B C D

distance along river

Permission to reproduce items where third-party owned material protected by copyright is included has been sought and cleared where possible. Every
reasonable effort has been made by the publisher (UCLES) to trace copyright holders, but if any items requiring clearance have unwittingly been included, the
publisher will be pleased to make amends at the earliest possible opportunity.

To avoid the issue of disclosure of answer-related information to candidates, all copyright acknowledgements are reproduced online in the Cambridge
International Examinations Copyright Acknowledgements Booklet. This is produced for each series of examinations and is freely available to download at
www.cie.org.uk after the live examination series.

465
Cambridge International Examinations is part of the Cambridge Assessment Group. Cambridge Assessment is the brand name of University of Cambridge Local
Examinations Syndicate (UCLES), which is itself a department of the University of Cambridge.

© UCLES 2017 0610/12/M/J/17


Cambridge International Examinations
Cambridge International General Certificate of Secondary Education

BIOLOGY 0610/13
Paper 1 Multiple Choice (Core) May/June 2017
45 minutes
Additional Materials: Multiple Choice Answer Sheet
Soft clean eraser
*3088341698*

Soft pencil (type B or HB is recommended)

READ THESE INSTRUCTIONS FIRST

Write in soft pencil.


Do not use staples, paper clips, glue or correction fluid.
Write your name, Centre number and candidate number on the Answer Sheet in the spaces provided
unless this has been done for you.
DO NOT WRITE IN ANY BARCODES.

There are forty questions on this paper. Answer all questions. For each question there are four possible
answers A, B, C and D.
Choose the one you consider correct and record your choice in soft pencil on the separate Answer Sheet.

Read the instructions on the Answer Sheet very carefully.

Each correct answer will score one mark. A mark will not be deducted for a wrong answer.
Any rough working should be done in this booklet.
Electronic calculators may be used.

The syllabus is approved for use in England, Wales and Northern Ireland as a Cambridge International Level 1/Level 2 Certificate.

This document consists of 16 printed pages.

IB17 06_0610_13/3RP
© UCLES 2017 [Turn466
over
2

1 Which characteristic do all living organisms show?

A breathing
B excretion
C photosynthesis
D tropism

2 The diagram shows some animal cells, as seen under the microscope.

What will be present at X?

A one cell membrane


B one cell wall
C two cell membranes
D two cell walls

467
© UCLES 2017 0610/13/M/J/17
3

3 The diagram shows an animal.

Use the key to identify the animal in the diagram.

1 has six jointed legs .......................................... go to 2


has eight jointed legs ....................................... go to 3
2 has antennae longer than its body .................. B Gryllus
has antennae shorter than its body ................. C Musca
3 has stripes on its legs ...................................... A Araneus
has no stripes on its legs ................................. D Pisaurina

468
© UCLES 2017 0610/13/M/J/17 [Turn over
4

4 The diagram shows a section through a root.

P
Q

What are the levels of organisation of the labelled structures?

cell organ tissue

A P Q R
B P R Q
C Q R P
D R Q P

5 Which characteristics are correct for both osmosis and diffusion?

require a partially require a are energy


permeable membrane concentration gradient consuming processes

A   
B   
C   
D   

469
© UCLES 2017 0610/13/M/J/17
5

6 What is the correct test for protein?

name of test heat colour change

A Benedict’s yes blue to red


B biuret yes remains blue
C biuret no blue to purple
D emulsion no appearance of a
white emulsion

7 The diagram represents an enzyme-controlled reaction.

What represents the enzyme?

A D

8 Four test-tubes are set up as shown.

Which test-tube contains the most carbon dioxide after one hour?

A B C D

black
water polythene
to keep
water out light
snail light light light
aquatic
plant

470
© UCLES 2017 0610/13/M/J/17 [Turn over
6

9 The diagrams show the structure of four different cells from the leaf of a dicotyledonous plant.

Which cell is a guard cell?

A B

NOT TO
SCALE

C D

10 Which mineral ion is absorbed by plant roots and used in the production of all amino acids?

A carbonate
B hydroxide
C magnesium
D nitrate

11 What is the pathway taken by water as it travels through a plant?

A mesophyll cells → xylem → root cortex cells → root hair cells

B root cortex cells → root hair cells → mesophyll cells → xylem

C root hair cells → root cortex cells → xylem → mesophyll cells

D xylem cells → mesophyll → root cortex cells → root hair cells

471
© UCLES 2017 0610/13/M/J/17
7

12 Which is the main source of fibre in a human diet?

A dairy products
B meat products
C plant material
D water

13 The diagram shows a person’s teeth. Some of their teeth are missing.

What would the person find difficult?

A biting apples
B drinking water
C grinding meat
D tearing dry bread

14 What are functions of the hydrochloric acid in gastric juice?

giving the optimum pH


killing bacteria
for gastric enzymes

A  
B  
C  
D  

472
© UCLES 2017 0610/13/M/J/17 [Turn over
8

15 The diagrams show transverse sections through a root and a leaf.

W Y
V

root leaf

Which correctly identifies the phloem?

A V and Y B V and Z C W and Y D W and Z

16 A piece of blue cobalt(II) chloride paper is clipped to the lower surface of a fresh leaf and is then
covered with plastic, as shown.

After a few minutes, part of the paper turns pink, showing that water is present.

paper remains blue

paper turns
lower surface pink
of leaf

cobalt(II) chloride paper

plastic cover

Which process carried out by leaves causes the paper to turn pink?

A absorption
B photosynthesis
C respiration
D transpiration

473
© UCLES 2017 0610/13/M/J/17
9

17 The diagram shows the outside of a human heart.

Which is a coronary artery?

B
A
C

left atrium
right atrium
D

left ventricle

18 Which of the blood vessels transporting these substances is an artery?

A carbon dioxide from the body to the heart


B carbon dioxide from the heart to the lungs
C oxygen from the lungs to the left atrium
D urea from the kidney to the vena cava

19 Which body defence is a chemical barrier?

A antibody production
B hairs in the nose
C mucus lining the airways
D skin

474
© UCLES 2017 0610/13/M/J/17 [Turn over
10

20 The diagram shows part of the human gas exchange system.

Where does the exchange of gases between air and blood take place?

21 Which equation represents anaerobic respiration in humans?

A glucose → lactic acid

B glucose → lactic acid + carbon dioxide

C glucose + oxygen → lactic acid

D glucose + oxygen → lactic acid + carbon dioxide

22 The diagram shows some organs in the human body.

In which part are amino acids broken down to form urea?

D
A

475
© UCLES 2017 0610/13/M/J/17
11

23 Which is the light sensitive part of the eye?

A cornea
B iris
C lens
D retina

24 Which response occurs when body temperature rises?

A contraction of hair erector muscles


B increased production of urine
C increased shivering
D increased sweating

25 What is meant by the term phototropism?

A absorbing mineral ions using light energy


B directional growth in response to gravity
C directional growth in response to light
D making food using light energy

26 Which row shows the effects of excessive alcohol consumption?

anti-social
reaction time self-control
behaviour

A quicker increased increased


B quicker reduced increased
C slower increased decreased
D slower reduced increased

27 Which statement applies to sexual reproduction?

A Offspring are genetically identical to the parent.


B Offspring are not produced from gametes.
C Offspring develop from a zygote.
D Offspring develop from one gamete.

476
© UCLES 2017 0610/13/M/J/17 [Turn over
12

28 The diagram shows half a flower.

Which structure is the stigma?

A
B
C
D

29 The diagram shows the human female reproductive system.

What is structure X?

A ovary
B oviduct
C uterus
D vagina

30 Which is a barrier method of birth control?

A abstinence from sex


B monitoring body temperature
C monitoring cervical mucus
D using a femidom

477
© UCLES 2017 0610/13/M/J/17
13

31 What do genes code for?

A fats
B proteins
C starch
D sugars

32 What are the uses of meiosis and mitosis?

asexual sexual
growth
reproduction reproduction

A meiosis meiosis meiosis


B meiosis meiosis mitosis
C mitosis meiosis meiosis
D mitosis mitosis meiosis

33 The graph shows the masses of two different types of tomato.

type 1 type 2

number of
tomatoes

0 20 40 60 80 100 120 140 160 180 200 220 240


mass / g

What can be concluded from the graph?

A Genes do not affect the mass of tomatoes.


B Type 1 tomatoes show continuous variation.
C Type 2 tomatoes are sometimes smaller than type 1 tomatoes.
D Type 2 tomatoes show discontinuous variation.

478
© UCLES 2017 0610/13/M/J/17 [Turn over
14

34 Large areas of forest are cut down in order to clear the land for other uses.

Which effect does this have on the atmosphere?

A Carbon dioxide decreases.


B Carbon dioxide increases.
C Oxygen increases.
D Water vapour increases.

35 The diagram shows part of a food web in a pile of dead leaves.

centipedes

bacteria millipedes mites


fungi

dead leaves

Which terms describe the organisms in the table?

bacteria centipedes leaves

A consumers herbivores producers


B decomposers carnivores decomposers
C decomposers consumers producers
D producers carnivores producers

479
© UCLES 2017 0610/13/M/J/17
15

36 The diagram shows part of the water cycle.

clouds

T
air

S plants water in soil

rivers and lakes

What are processes S and T?

S T

A condensation drainage
B condensation evaporation
C evaporation precipitation
D evaporation transpiration

37 The graph shows the changes in the populations of predator and prey over a period of time.

Which point on the graph shows a decrease in predator population?

A
C
number of
individuals

D
B

time

480
© UCLES 2017 0610/13/M/J/17 [Turn over
16

38 Which is an example of genetic engineering?

A altering the DNA in crop plants so they are resistant to herbicides


B only breeding from crop plants that are resistant to insect pests
C production of insulin in the pancreas
D using yeast to produce ethanol for biofuels

39 Why is yeast used in breadmaking?

A to produce alcohol
B to produce carbon dioxide
C to use up oxygen
D to use up sugar

40 The graph shows the levels of dissolved oxygen and mineral ions in a river.

At which point does raw sewage enter the river?

direction of river flow

dissolved oxygen

levels

mineral ions

A B C D

distance along river

Permission to reproduce items where third-party owned material protected by copyright is included has been sought and cleared where possible. Every
reasonable effort has been made by the publisher (UCLES) to trace copyright holders, but if any items requiring clearance have unwittingly been included, the
publisher will be pleased to make amends at the earliest possible opportunity.

To avoid the issue of disclosure of answer-related information to candidates, all copyright acknowledgements are reproduced online in the Cambridge
International Examinations Copyright Acknowledgements Booklet. This is produced for each series of examinations and is freely available to download at
www.cie.org.uk after the live examination series.

481
Cambridge International Examinations is part of the Cambridge Assessment Group. Cambridge Assessment is the brand name of University of Cambridge Local
Examinations Syndicate (UCLES), which is itself a department of the University of Cambridge.

© UCLES 2017 0610/13/M/J/17


Cambridge International Examinations
Cambridge International General Certificate of Secondary Education

BIOLOGY 0610/22
Paper 2 Multiple Choice (Extended) February/March 2017
45 minutes
Additional Materials: Multiple Choice Answer Sheet
Soft clean eraser
*7903107076*

Soft pencil (type B or HB is recommended)

READ THESE INSTRUCTIONS FIRST

Write in soft pencil.


Do not use staples, paper clips, glue or correction fluid.
Write your name, Centre number and candidate number on the Answer Sheet in the spaces provided
unless this has been done for you.
DO NOT WRITE IN ANY BARCODES.

There are forty questions on this paper. Answer all questions. For each question there are four possible
answers A, B, C and D.
Choose the one you consider correct and record your choice in soft pencil on the separate Answer Sheet.

Read the instructions on the Answer Sheet very carefully.

Each correct answer will score one mark. A mark will not be deducted for a wrong answer.
Any rough working should be done in this booklet.
Electronic calculators may be used.

The syllabus is approved for use in England, Wales and Northern Ireland as a Cambridge International Level 1/Level 2 Certificate.

This document consists of 15 printed pages and 1 blank page.

IB17 03_0610_22/4RP
© UCLES 2017 [Turn482
over
2

1 The drawing shows a ground squirrel.

Which feature identifies this animal as a mammal?

A eye
B four limbs
C fur
D tail

2 The diagram shows how a seed changes after it is planted in soil and watered.

Which characteristics of living things are demonstrated by this sequence?

A excretion and growth


B growth and sensitivity
C nutrition and reproduction
D nutrition and sensitivity

483
© UCLES 2017 0610/22/F/M/17
3

3 The diagram shows part of the classification of the animal kingdom with an example of each
group.

animals

without backbones with backbones


(invertebrates) (vertebrates)

arthropods fish amphibians reptiles birds mammals


carp frog lizard parrot rat

crustaceans myriapods insects arachnids


crab centipede flea spider

Which pair of animals have the most recent common ancestor as suggested by the classification?

A centipede and carp


B flea and frog
C lizard and parrot
D spider and rat

4 The diagram shows a sample of material taken from an organism.

Which level of organisation does the sample show?

A cell
B organ
C organ system
D tissue

5 The length of a specimen in a photograph is 45 mm. Its actual length is 25 mm.

What is the magnification of the photograph?

A ×0.6 B ×1.6 C ×1.8 D ×1125

484
© UCLES 2017 0610/22/F/M/17 [Turn over
4

6 The diagram shows a section through a leaf.

Which arrow shows the direction of diffusion of carbon dioxide on a sunny day?

7 The diagram represents two liquids, separated by a membrane through which osmosis can occur.

membrane
left right
key

molecule of water

molecule of
dissolved sustance

Which statement describes how the molecules will move?

A Molecules of dissolved substance move from left to right.


B Molecules of dissolved substance move from right to left.
C Overall, water molecules move from left to right.
D Overall, water molecules move from right to left.

485
© UCLES 2017 0610/22/F/M/17
5

8 Which element is found in proteins but not carbohydrates?

A carbon
B hydrogen
C nitrogen
D oxygen

9 Many enzymes do not work at temperatures above 60 °C.

Which statement explains this?

A Product molecules are not made because the active site has changed shape.
B Product molecules change shape so they do not fit the active site.
C Substrate molecules are moving too fast.
D Substrate molecules are moving too slowly.

10 Which substances are used for photosynthesis?

A carbon dioxide and glucose


B carbon dioxide and water
C glucose and oxygen
D glucose and water

11 Which term is defined as the taking of substances into the body through the mouth?

A absorption
B assimilation
C digestion
D ingestion

12 Which nutrient is deficient in the diet of a child with kwashiorkor?

A calcium
B iron
C protein
D vitamin D

486
© UCLES 2017 0610/22/F/M/17 [Turn over
6

13 The diagram shows a tooth with signs of decay.

enamel

decay

What has made the hole in the enamel of the tooth?

A acid
B saliva
C sugar
D toothpaste

14 Which is a description of translocation?

A movement of amino acids and sucrose from sink to source


B movement of amino acids and sucrose from source to sink
C movement of water down a water potential gradient
D movement of water up a water potential gradient

15 Which is a function of the lymphatic system?

A deamination of amino acids


B production of lipase
C protection from infection
D transport of oxygen

487
© UCLES 2017 0610/22/F/M/17
7

16 A hospital patient who is feeling unwell is given a blood test.

The results of the blood test show a very low level of platelets.

What effect will this have?

A The blood will be unable to transport nutrients, hormones and carbon dioxide.
B The blood will not be able to carry as much oxygen to the tissues as normal.
C There will be a greater risk of bleeding because the blood will take longer to clot.
D There will be a greater risk of infection because the blood cannot make antibodies.

17 What is a common feature of both active and passive immunity?

A They are acquired by vaccination.


B They are always short-term.
C They involve the activity of memory cells.
D They involve antibodies.

488
© UCLES 2017 0610/22/F/M/17 [Turn over
8

18 The graph shows the rate and depth of a person’s breathing before exercise.

2.5

volume of air in 2.0


the lungs during
breathing / dm3 1.5

1.0
0 4 8
time / s

Which graph shows the rate and depth of breathing of the same person immediately after a
period of exercise?

A B
2.5 2.5

volume of air in 2.0 volume of air in 2.0


the lungs during the lungs during
breathing / dm3 1.5 breathing / dm3 1.5

1.0 1.0
0 4 8 0 4 8
time / s time / s

C D
2.5 2.5

volume of air in 2.0 volume of air in 2.0


the lungs during the lungs during
breathing / dm3 1.5 breathing / dm3 1.5

1.0 1.0
0 4 8 0 4 8
time / s time / s

489
© UCLES 2017 0610/22/F/M/17
9

19 In an experiment, three glass bell jars were set up as shown in the diagram.

air from
bell jar the lungs

green
plant

P Q R
left in sunlight left open to the air breathed out by a
for 8 hours air for 8 hours student for 5 minutes

At the end of the experiment, which bell jar has the most oxygen and which has the least?

most oxygen least oxygen

A P Q
B P R
C Q P
D R P

20 What are the products of anaerobic respiration in muscles?

A ethanol and carbon dioxide


B ethanol only
C lactic acid and carbon dioxide
D lactic acid only

21 Which substance remains in the blood as it passes through the kidney?

A protein
B salts
C urea
D water

490
© UCLES 2017 0610/22/F/M/17 [Turn over
10

22 The diagram shows the mechanisms that control the concentration of glucose in the blood.

hormone X

gland W lowers glucose


concentration
rises

normal blood glucose concentration

falls
raises glucose
concentration gland Y

hormone Z

Which row identifies the glands and hormones labelled W, X, Y and Z?

gland W hormone X gland Y hormone Z

A adrenal gland adrenaline adrenal gland glucagon


B adrenal gland adrenaline pancreas insulin
C pancreas glucagon adrenal gland insulin
D pancreas insulin pancreas glucagon

23 Which row shows the effects of increased adrenaline release?

breathing rate pulse rate pupil

A decreases increases widens


B increases decreases widens
C increases increases narrows
D increases increases widens

491
© UCLES 2017 0610/22/F/M/17
11

24 The graph shows how the rate of photosynthesis of a plant changes with light intensity, at three
different carbon dioxide concentrations. In each case the temperature is 15 °C.

0.10% carbon dioxide

X
rate of 0.04% carbon dioxide
photosynthesis
0.02% carbon dioxide

light intensity

What is the limiting factor for the rate of photosynthesis at point X on the graph?

A carbon dioxide concentration


B light intensity
C surface area of the plant
D temperature

25 What is an advantage of asexual reproduction for a population of flowering plants in the wild?

A A disease is less likely to affect the whole population.


B Large numbers of offspring can be produced quickly.
C Pollen can easily be transferred within the same flower.
D The offspring show genetic variety.

26 During sexual reproduction in plants, what will give rise to the greatest variation in the offspring?

A All of the flowers on the same plant have male and female reproductive organs.
B The anthers and stigmas on the same plant mature at the same time of year.
C There are separate male and female flowers on the same plant.
D There are separate male and female plants.

492
© UCLES 2017 0610/22/F/M/17 [Turn over
12

27 The diagram shows the head of a sperm.

What is the function of structure X?

A It carries genetic information.


B It controls the activities of the sperm cell.
C It is involved in the digestion of the ovum cell membrane.
D It provides energy for movement.

28 Which hormone is given to women undergoing fertility treatment?

A adrenaline
B FSH
C insulin
D oestrogen

29 The diagram represents the fusion of sperm and ovum at fertilisation.

ovum

fertilisation cell X

sperm

Which description of cell X is correct?

A diploid gamete
B diploid zygote
C haploid gamete
D haploid zygote

30 A cell with 16 chromosomes divides twice by mitosis.

How many chromosomes does each of the resulting cells contain?

A 4 B 8 C 16 D 32

493
© UCLES 2017 0610/22/F/M/17
13

31 A man of genotype IAIo and woman of genotype IBIo have a child.

What is the chance that the child will have the same blood group as one of its parents?

A zero B 1 in 4 C 1 in 2 D 3 in 4

32 Which human phenotype is affected by environmental and genetic factors?

A blood group
B body size
C gender
D tongue rolling

33 Which adaptation may be present in a xerophyte?

A leaves with small surface area and large numbers of stomata


B little or no xylem tissue and leaves with large surface area
C stomatal hairs and rolled leaves
D thin or no cuticle and deep roots

34 How does artificial selection differ from natural selection?

A Selection changes the characteristics of living things.


B Selection is based on genetic variation.
C Selection is not based on adaptation to their environment.
D Selection occurs over many generations.

35 The diagram shows the energy present in a food chain.

40 000 kJ 4000 kJ 400 kJ 40 kJ


photosynthesising → krill → herring → bass
phytoplankton

What percentage of energy present in the producer is transferred to the secondary consumer?

A 0.01% B 0.1% C 1% D 10%

494
© UCLES 2017 0610/22/F/M/17 [Turn over
14

36 Which process results in the loss of nitrates from soils?

A deamination
B decomposition
C denitrification
D nitrification

37 During the exponential (log) phase of a sigmoid growth curve, which factor limits population
growth?

A availability of food
B build-up of wastes
C presence of disease
D rate of reproduction

38 Genes are isolated from human DNA using ……1…… enzymes.

A bacterial plasmid is cut with the same enzyme forming ……2…… .

The human DNA is inserted into the bacterial plasmid using the enzyme ……3…… forming a

……4…… plasmid.

Which row correctly completes gaps 1, 2, 3 and 4?

1 2 3 4

A ligase sticky ends protease restriction


B recombinant new DNA ligase daughter
C restriction daughter plasmids ligase diploid
D restriction sticky ends ligase recombinant

39 How does cutting down trees contribute to the greenhouse effect?

A There will be less carbon dioxide absorbed.


B There will be less oxygen absorbed.
C There will be less shade from trees.
D The soil will become dry.

495
© UCLES 2017 0610/22/F/M/17
15

40 Which graph shows the effect of pollution by untreated sewage on the amount of oxygen
dissolved in a river?

A B

dissolved dissolved
oxygen oxygen

distance downstream distance downstream


sewage enters sewage enters
the river the river

C D

dissolved dissolved
oxygen oxygen

distance downstream distance downstream


sewage enters sewage enters
the river the river

496
© UCLES 2017 0610/22/F/M/17
Cambridge International Examinations
Cambridge International General Certificate of Secondary Education

BIOLOGY 0610/12
Paper 1 Multiple Choice (Core) February/March 2017
45 minutes
Additional Materials: Multiple Choice Answer Sheet
Soft clean eraser
*1962770622*

Soft pencil (type B or HB is recommended)

READ THESE INSTRUCTIONS FIRST

Write in soft pencil.


Do not use staples, paper clips, glue or correction fluid.
Write your name, Centre number and candidate number on the Answer Sheet in the spaces provided
unless this has been done for you.
DO NOT WRITE IN ANY BARCODES.

There are forty questions on this paper. Answer all questions. For each question there are four possible
answers A, B, C and D.
Choose the one you consider correct and record your choice in soft pencil on the separate Answer Sheet.

Read the instructions on the Answer Sheet very carefully.

Each correct answer will score one mark. A mark will not be deducted for a wrong answer.
Any rough working should be done in this booklet.
Electronic calculators may be used.

The syllabus is approved for use in England, Wales and Northern Ireland as a Cambridge International Level 1/Level 2 Certificate.

This document consists of 17 printed pages and 3 blank pages.

IB17 03_0610_12/2RP
© UCLES 2017 [Turn497
over
2

1 The drawing shows a ground squirrel.

Which feature identifies this animal as a mammal?

A eye
B four limbs
C fur
D tail

2 The diagram shows how a seed changes after it is planted in soil and watered.

Which characteristics of living things are demonstrated by this sequence?

A excretion and growth


B growth and sensitivity
C nutrition and reproduction
D nutrition and sensitivity

498
© UCLES 2017 0610/12/F/M/17
3

3 The diagram shows a palisade cell from a leaf.

How does a palisade cell differ from the other cells in the same plant?

A It has a cell wall.


B It has a nucleus.
C It has a smaller vacuole.
D It has more chloroplasts.

4 The diagram shows some cells.

cilia

cytoplasm

nucleus

Where are these cells found?

A alimentary canal
B blood
C bronchus
D plant roots

5 The length of a specimen in a photograph is 45 mm. Its actual length is 25 mm.

What is the magnification of the photograph?

A ×0.6 B ×1.6 C ×1.8 D ×1125

499
© UCLES 2017 0610/12/F/M/17 [Turn over
4

6 The diagram shows a section through a leaf.

Which arrow shows the direction of diffusion of carbon dioxide on a sunny day?

7 The diagram represents two liquids, separated by a membrane through which osmosis can occur.

membrane
left right
key

molecule of water

molecule of
dissolved sustance

Which statement describes how the molecules will move?

A Molecules of dissolved substance move from left to right.


B Molecules of dissolved substance move from right to left.
C Overall, water molecules move from left to right.
D Overall, water molecules move from right to left.

500
© UCLES 2017 0610/12/F/M/17
5

8 Which element is found in proteins but not carbohydrates?

A carbon
B hydrogen
C nitrogen
D oxygen

9 Two samples of a human enzyme were used in an experiment. Before they were used

● sample X was heated to 80 °C and then cooled to 37 °C,

● sample Y was cooled to 0 °C and then heated to 37 °C.

How will this affect their activity?

A Sample X and sample Y are no longer active.


B Sample X and sample Y will be equally active.
C Sample X will be more active than sample Y.
D Sample Y will be more active than sample X.

10 Which substances are used for photosynthesis?

A carbon dioxide and glucose


B carbon dioxide and water
C glucose and oxygen
D glucose and water

11 Which term is defined as the taking of substances into the body through the mouth?

A absorption
B assimilation
C digestion
D ingestion

12 What is needed to make the haemoglobin in red blood cells?

A calcium
B iron
C roughage
D vitamin D

501
© UCLES 2017 0610/12/F/M/17 [Turn over
6

13 The diagram shows a tooth with signs of decay.

enamel

decay

What has made the hole in the enamel of the tooth?

A acid
B saliva
C sugar
D toothpaste

502
© UCLES 2017 0610/12/F/M/17
7

14 Four leafy plant stems were placed into measuring cylinders with 100 cm3 of water. A layer of oil
prevented the water in the measuring cylinder from evaporating.

layer of oil

water

The plant stems were exposed to different air humidities and temperatures for 48 hours as shown
in the table.

plant temperature final volume of


humidity
stem / °C water / cm3

1 low 5 75
2 low 25
3 high 5 95
4 high 25 65

The final volume of water in the measuring cylinders is shown for plant stems 1, 3 and 4.

What would be a likely final volume for plant stem 2?

A less than 65 cm3


B between 65 cm3 and 75 cm3
C between 75 cm3 and 95 cm3
D greater than 95 cm3

503
© UCLES 2017 0610/12/F/M/17 [Turn over
8

15 Which blood vessel, if it becomes blocked, could lead directly to a heart attack?

A coronary artery
B pulmonary artery
C pulmonary vein
D vena cava

16 A hospital patient who is feeling unwell is given a blood test.

The results of the blood test show a very low level of platelets.

What effect will this have?

A The blood will be unable to transport nutrients, hormones and carbon dioxide.
B The blood will not be able to carry as much oxygen to the tissues as normal.
C There will be a greater risk of bleeding because the blood will take longer to clot.
D There will be a greater risk of infection because the blood cannot make antibodies.

17 What is a pathogen?

A a bacterium
B a disease-causing organism
C a disease that is transmissible
D a white blood cell that engulfs bacteria

504
© UCLES 2017 0610/12/F/M/17
9

18 The graph shows the rate and depth of a person’s breathing before exercise.

2.5

volume of air in 2.0


the lungs during
breathing / dm3 1.5

1.0
0 4 8
time / s

Which graph shows the rate and depth of breathing of the same person immediately after a
period of exercise?

A B
2.5 2.5

volume of air in 2.0 volume of air in 2.0


the lungs during the lungs during
breathing / dm3 1.5 breathing / dm3 1.5

1.0 1.0
0 4 8 0 4 8
time / s time / s

C D
2.5 2.5

volume of air in 2.0 volume of air in 2.0


the lungs during the lungs during
breathing / dm3 1.5 breathing / dm3 1.5

1.0 1.0
0 4 8 0 4 8
time / s time / s

505
© UCLES 2017 0610/12/F/M/17 [Turn over
10

19 In an experiment, three glass bell jars were set up as shown in the diagram.

air from
bell jar the lungs

green
plant

P Q R
left in sunlight left open to the air breathed out by a
for 8 hours air for 8 hours student for 5 minutes

At the end of the experiment, which bell jar has the most oxygen and which has the least?

most oxygen least oxygen

A P Q
B P R
C Q P
D R P

20 What are the products of anaerobic respiration in muscles?

A ethanol and carbon dioxide


B ethanol only
C lactic acid and carbon dioxide
D lactic acid only

21 Which substance remains in the blood as it passes through the kidney?

A protein
B salts
C urea
D water

506
© UCLES 2017 0610/12/F/M/17
11

22 When the nervous system responds to a stimulus there are several stages to the response.

1 The central nervous system processes the information.


2 The receptors detect the stimulus.
3 A nerve impulse is sent to the central nervous system.
4 A response is produced.
5 A nerve impulse is sent to the muscles.

What is the correct order of the stages?

A 2, 3, 1, 5, 4
B 2, 3, 5, 1, 4
C 3, 2, 1, 5, 4
D 3, 2, 5, 1, 4

23 In which parts of the body are there receptors for blood temperature?

fatty tissue
brain
of the skin

A  
B  
C  
D  

24 Which row shows the effects of increased adrenaline release?

breathing rate pulse rate pupil

A decreases increases widens


B increases decreases widens
C increases increases narrows
D increases increases widens

25 Which statement about the growth response of a plant shoot is correct?

A It grows away from gravity and away from light.


B It grows away from gravity and towards light.
C It grows towards gravity and away from light.
D It grows towards gravity and towards light.

507
© UCLES 2017 0610/12/F/M/17 [Turn over
12

26 A patient is suffering from an infection. Her doctor prescribes an antibiotic.

One week later the infection is still present.

What could be the reason for this?

A It was the correct antibiotic for this infection.


B The pathogen was a virus.
C The pathogen was resistant to the antibodies.
D The patient was resistant to the antibiotic.

27 The diagram shows Hydra growing and releasing an offspring from the side of its body.

offspring

parent
hydra

Which row is correct?

parent and
uses sexual
offspring are
reproduction
genetically identical

A  
B  
C  
D  

508
© UCLES 2017 0610/12/F/M/17
13

28 The photograph shows the structures needed for reproduction in the flower of a dicotyledon.

What is the part labelled P?

A a stigma
B a style
C an anther
D an ovary

29 What is the name of the organ in which a human fetus grows until it is born?

A bladder
B ovary
C urethra
D uterus

30 What would protect against a sexually transmitted infection?

A contraceptive implant
B use of a condom
C use of a diaphragm
D vasectomy

509
© UCLES 2017 0610/12/F/M/17 [Turn over
14

31 Which term describes the transmission of genetic information from one generation to the next?

A genetic engineering
B inheritance
C natural selection
D variation

32 What is a gene?

A a chain of amino acids that codes for a protein


B a length of DNA that codes for a protein
C a length of protein that codes for an allele
D a structure that codes for the production of DNA

33 Which process involves meiosis?

A growth of cells
B production of gametes
C repair of damaged tissues
D replacement of cells

34 The allele for brown coat colour is dominant to the allele for albino coat colour.

A homozygous brown-coated mammal is crossed with a heterozygous brown-coated mammal.

What percentage of the offspring will be albino?

A 0% B 25% C 50% D 100%

35 Which description of an adaptive feature is correct?

A a change in phenotype that is caused by the environment


B a change in the environment that is an advantage to an organism
C an action by an organism that aids its ability to survive and reproduce
D an inherited feature that increases an organism’s ability to survive and reproduce

510
© UCLES 2017 0610/12/F/M/17
15

36 The diagram shows part of the carbon cycle.

carbon dioxide

dead matter plants

animals

What does arrow X represent?

A decay
B nutrition
C photosynthesis
D respiration

37 The diagram shows the water cycle.

Y
Z

pond

Which processes do Y and Z represent?

Y Z

A condensation evaporation
B precipitation evaporation
C transpiration condensation
D transpiration evaporation

511
© UCLES 2017 0610/12/F/M/17 [Turn over
16

38 What is inserted into a bacterium to make the bacterium produce insulin?

A a length of DNA from a human


B a length of DNA from another bacterium
C a molecule of insulin
D an enzyme

39 The graph shows world wheat production from 1950 to 2000.

world
wheat
production

1950 1975 2000

What has contributed to the changes shown in wheat production?

A not using herbicides


B not using insecticides
C using selective breeding
D using smaller areas of land

512
© UCLES 2017 0610/12/F/M/17
17

40 Which graph shows the effect of pollution by untreated sewage on the amount of oxygen
dissolved in a river?

A B

dissolved dissolved
oxygen oxygen

distance downstream distance downstream


sewage enters sewage enters
the river the river

C D

dissolved dissolved
oxygen oxygen

distance downstream distance downstream


sewage enters sewage enters
the river the river

513
© UCLES 2017 0610/12/F/M/17

Das könnte Ihnen auch gefallen